[obm-l] RES: [obm-l] Livros OBM - Universitária

2021-04-30 Por tôpico bouskela
Olá!

 

Certamente, um bom início é ler o livro A Arte De Resolver Problemas (George 
Pólya). Há, também, a revista Eureka!

 

Por último, o site [ AoPS and Beast Academy Math Programs for Advanced Students 
] contém os problemas (e as respectivas soluções) das Olimpíadas de Matemática 
já realizadas.

 

Albert Bouskelá

  bousk...@gmail.com

 

De: owner-ob...@mat.puc-rio.br  Em nome de Wallace 
Albert
Enviada em: terça-feira, 27 de abril de 2021 20:49
Para: obm-l@mat.puc-rio.br
Assunto: [obm-l] Livros OBM - Universitária

 

Pessoal, bom dia!

 

Pretendo iniciar meus estudos pra OBM nível U em breve e gostaria de saber se 
vocês têm alguma recomendação de livros didáticos para que eu possa estudar. No 
site da OBM vejo que o foco é mais voltado às olimpíadas de ensino 
médio/internacionais. Alguém poderia me ajudar? Desde já agradeço!

-- 

Att.

 

Wallace Albert da Silva Antonio

(11) 9-6763-3118


-- 
Esta mensagem foi verificada pelo sistema de antiv�s e 
acredita-se estar livre de perigo. 


-- 
Esta mensagem foi verificada pelo sistema de antiv�rus e
 acredita-se estar livre de perigo.



[obm-l] Re: [obm-l] RES: [obm-l] Re: [obm-l] INFLAÇÂO MÁXIMA

2021-04-24 Por tôpico Pedro Júnior
Sobre a gramática, verdade!
Sobre a Matemática, impecável.
Consegui ver onde eu estava errando.

Obrigado, professor.

Em sex, 23 de abr de 2021 14:24,  escreveu:

> Olá!
>
> Para começar, esta questão deveria ter sido anulada. “… não HAJAM perdas
> reais?” é um assassinato da nossa língua.
>
>
>
> Juros “reais” (JR), de 10%, significam juros acima da inflação (IF).
>
> No período de 1 ano, o ganho bruto de capital (GB) será: GB = 1.000
> (1+10%)(1+IF) - 1.000
>
> Descontando o imposto, o ganho líquido (GL) será: GL =  (1-40%)GB
>
> Condição de contorno: não “hão” (coerência com o linguajar da questão)
> perdas reais: 1.000(1+IF) = 1.000+GL = 1.000 + (1-40%)GB = 1.000 + (1-40%)(
> 1.000 (1+10%)(1+IF) - 1.000 )
>
> Daí: IF = 17,6470…%
>
>
>
> *Albert Bouskelá*
>
> bousk...@gmail.com
>
>
>
> *De:* owner-ob...@mat.puc-rio.br  *Em nome de
> *Daniel Jelin
> *Enviada em:* sexta-feira, 23 de abril de 2021 12:30
> *Para:* obm-l@mat.puc-rio.br
> *Assunto:* [obm-l] Re: [obm-l] INFLAÇÂO MÁXIMA
>
>
>
> Curioso, pra mim deu muito perto, 17,6470...%
>
> Resolvi a seguinte inequação, com x = 1 + (inflação):
>
> 1.1*1000x - (1.1*1000x - 1000)*0.4>=1000x
> 1.1 x - 0.44 x + 0.4 >= x
> x<=0.4/0.34= 1.176470...
>
> Parece simples. O que tá escapando aqui?
>
>
>
> On Fri, Apr 23, 2021 at 11:23 AM Pedro Júnior 
> wrote:
>
> Olá pessoal, acabei me enrolando nesse probleminha da Olimpíada Brasileira
> de Economia. Será que alguém pode me ajudar? Vai junto o gabarito da
> competição, isso foi em 2020.
>
>
>
> *01)* Um título comprado por mil reais promete pagar juros reais de 10%
> a.a. A alíquota de imposto é de 40%. Qual a inflação máxima no período para
> que não hajam perdas reais?
>
> Resp.: 17,62%
>
>
>
> --
>
> Pedro Jerônimo S. de O. Júnior
>
> Professor de Matemática
>
>
>
>
> --
> Esta mensagem foi verificada pelo sistema de antivírus e
> acredita-se estar livre de perigo.
>
>
> --
> Esta mensagem foi verificada pelo sistema de antiv?s e
> acredita-se estar livre de perigo.
>
> --
> Esta mensagem foi verificada pelo sistema de antivírus e
> acredita-se estar livre de perigo.
>

-- 
Esta mensagem foi verificada pelo sistema de antiv�rus e
 acredita-se estar livre de perigo.



[obm-l] RES: [obm-l] Re: [obm-l] INFLAÇÂO MÁXIMA

2021-04-23 Por tôpico bouskela
Olá!

Para começar, esta questão deveria ter sido anulada. “… não HAJAM perdas 
reais?” é um assassinato da nossa língua.

 

Juros “reais” (JR), de 10%, significam juros acima da inflação (IF).

No período de 1 ano, o ganho bruto de capital (GB) será: GB = 1.000 
(1+10%)(1+IF) - 1.000

Descontando o imposto, o ganho líquido (GL) será: GL =  (1-40%)GB

Condição de contorno: não “hão” (coerência com o linguajar da questão) perdas 
reais: 1.000(1+IF) = 1.000+GL = 1.000 + (1-40%)GB = 1.000 + (1-40%)( 1.000 
(1+10%)(1+IF) - 1.000 )

Daí: IF = 17,6470…%

 

Albert Bouskelá

  bousk...@gmail.com

 

De: owner-ob...@mat.puc-rio.br  Em nome de Daniel 
Jelin
Enviada em: sexta-feira, 23 de abril de 2021 12:30
Para: obm-l@mat.puc-rio.br
Assunto: [obm-l] Re: [obm-l] INFLAÇÂO MÁXIMA

 

Curioso, pra mim deu muito perto, 17,6470...%

Resolvi a seguinte inequação, com x = 1 + (inflação):

1.1*1000x - (1.1*1000x - 1000)*0.4>=1000x
1.1 x - 0.44 x + 0.4 >= x
x<=0.4/0.34= 1.176470... 

Parece simples. O que tá escapando aqui?

 

On Fri, Apr 23, 2021 at 11:23 AM Pedro Júnior mailto:pedromatematic...@gmail.com> > wrote:

Olá pessoal, acabei me enrolando nesse probleminha da Olimpíada Brasileira de 
Economia. Será que alguém pode me ajudar? Vai junto o gabarito da competição, 
isso foi em 2020.

 

01) Um título comprado por mil reais promete pagar juros reais de 10% a.a. A 
alíquota de imposto é de 40%. Qual a inflação máxima no período para que não 
hajam perdas reais?

Resp.: 17,62%


 

-- 

Pedro Jerônimo S. de O. Júnior

Professor de Matemática

 


-- 
Esta mensagem foi verificada pelo sistema de antivírus e 
acredita-se estar livre de perigo. 


-- 
Esta mensagem foi verificada pelo sistema de antiv�s e 
acredita-se estar livre de perigo. 


-- 
Esta mensagem foi verificada pelo sistema de antiv�rus e
 acredita-se estar livre de perigo.



RES: [obm-l] Probabilidade

2021-04-08 Por tôpico bouskela
Este é um problema bastante interessante, contudo o seu enunciado, tal como 
está, apresenta uma falha: - É necessário fixar quais são os resultados 
possíveis numa determinada rodada do jogo! Dito assim, o enunciado admite, para 
cada rodada 4 possibilidades: (A=1, B=1); (A=1, B=0); (A=0, B=1); (A=0, B=0).

 

Albert Bouskelá

  bousk...@gmail.com

 

De: owner-ob...@mat.puc-rio.br  Em nome de 
Professor Vanderlei Nemitz
Enviada em: quinta-feira, 8 de abril de 2021 14:34
Para: OBM 
Assunto: Re: [obm-l] Probabilidade

 

Muito legal esse tipo de problema. 

Em que ano caiu, você sabe, Pacini?

 

Em sáb., 3 de abr. de 2021 às 15:22, Pacini Bores mailto:pacini.bo...@globo.com> > escreveu:

Olá pessoal, Encontrei uma resposta que não está entre as opções desta questão 
do Canguru.

" um certo jogo tem um vencedor quando este atinge 3 pontos a frente do 
oponente. Dois jogadores A e B estão jogando e, num determinado momento, A está 
1 ponto a frente de B. Os jogadores  têm probabilidades iguais de obter 1 
ponto. Qual a probabilidade de A vencer o jogo ?

(A) 1/2   (B) 2/3  (C) 3/4   (D) 4/5  (E) 5/6

 

O que vocês acham ?

 Pacini

 


-- 
Esta mensagem foi verificada pelo sistema de antivírus e 
acredita-se estar livre de perigo. 


-- 
Esta mensagem foi verificada pelo sistema de antiv�s e 
acredita-se estar livre de perigo. 


-- 
Esta mensagem foi verificada pelo sistema de antiv�rus e
 acredita-se estar livre de perigo.



RES: [obm-l] Dois problemas

2020-04-26 Por tôpico Rogério Possi Júnior
Prezados.

Segue o problema 1 corrigido.


  1.  (Ibero-1992) Para cada inteiro positivo n, seja a_n o último dígito de 
1+2+3+...+n. Calcule a_1+a_2+...+a_1992.

Sds,

Rogério


Enviado do Email para Windows 10

De: Carlos Victor
Enviado:domingo, 26 de abril de 2020 21:13
Para: obm-l@mat.puc-rio.br
Cc:owner-ob...@mat.puc-rio.br; Rogério Possi 
Júnior
Assunto: Re: [obm-l] Dois problemas


Para o (1), observar que a_n é periódico e  tem período igual a 20, daí 

Abraços

Carlos Victor

Em 26/04/2020 19:21, Rogério Possi Júnior escreveu:
Boa noite.

Quem pode ajudar com esses dois problemas:

1) (Ibero-1992) Para cada inteiro positivo n, seja a_n o último dígito de 
1+2+3+...+n. Calcule a_1+a_2+...+a_n.

2) (UK-1997) N é um número inteiro de 4 dígitos não terminado em zero, e R(N) é 
o número inteiro de 4 dígitos obtido pela reversão dos dígitos de N; por 
exemplo R(3275)=5723. Determine todos os inteiros N ára os quais R(N)=4N+3.

Sds,

Rogério


--
Esta mensagem foi verificada pelo sistema de antivírus e
acredita-se estar livre de perigo.



-- 
Esta mensagem foi verificada pelo sistema de antivírus e
 acredita-se estar livre de perigo.



[obm-l] RES: [obm-l] Uma concepção dos logaritmos

2020-03-09 Por tôpico bouskela
Olá!

O livro “e: A História de um Número” (Eli Maor) contém uma breve (mas
enriquecedora) história dos logaritmos e, é claro, da descoberta do número
“e”.

 

Albert Bouskelá

  bousk...@gmail.com

 

De: owner-ob...@mat.puc-rio.br  Em nome de
Maikel Andril Marcelino
Enviada em: domingo, 8 de março de 2020 16:12
Para: obm-l@mat.puc-rio.br
Assunto: [obm-l] Uma concepção dos logaritmos

 

Boa tarde, pessoal! Estou escrevendo um artigo sobre os logaritmos. Durante
a minha graduação, ninguém me comentou  sobre a origem dos logaritmos, para
resolver multiplicações enormes. 

 

Alguém conhece ou já escreveu algum artigo sobre a concepção que os
professores tem sobre a origem dos logaritmos? 

 

Acredito que 95% dos professores pensam que logaritmos foram criados com o
intuito de resolver equações exponenciais, onde as mesma não são possíveis
de reduzir a mesma base. Para não postar achismo, e nem posso, gostaria de
um referencial teórico.

 

Atenciosamente,

 

Maikel Andril Marcelino

 

(84) 9-9149-8991 (Contato)

(84) 8851-3451 (WhatsApp)


-- 
Esta mensagem foi verificada pelo sistema de antivírus e 
acredita-se estar livre de perigo. 


-- 
Esta mensagem foi verificada pelo sistema de antivírus e
 acredita-se estar livre de perigo.



RES: [obm-l] Re: [obm-l] Soma de raízes quadradas

2020-02-18 Por tôpico Hermann
Escreve para esse email

nicolaumat.puc-rio.br ou nicolau.saldanhagmail.com

dizendo que quer sair da lista

Enviado do Email para Windows 10

De: Lorena Luna
Enviado:terça-feira, 18 de fevereiro de 2020 03:22
Para: obm-l@mat.puc-rio.br
Assunto: [obm-l] Re: [obm-l] Soma de raízes quadradas

CANCELAR LISTA DE E-MAIL (Cancelar recebimento)

Em seg, 17 de fev de 2020 às 13:25, Vanderlei Nemitz  
escreveu:
Boa tarde!
Existe uma fórmula fechada para a soma das raízes quadradas dos n primeiros 
números naturais?

Muito obrigado!

-- 
Esta mensagem foi verificada pelo sistema de antiv�rus e 
acredita-se estar livre de perigo. 

-- 
Esta mensagem foi verificada pelo sistema de antiv�rus e 
acredita-se estar livre de perigo. 


-- 
Esta mensagem foi verificada pelo sistema de antiv�rus e
 acredita-se estar livre de perigo.



RES: [obm-l] Uma soma

2020-01-17 Por tôpico Hermann
Eu não sei fazer, mas pelo teorema das diferenças das diferenças que estudei em 
1996 e não lembro mais, com certeza absoluta é um polinômio de grau 3 em n.
Tipo n(n+a)(n+b)+k algo assim...

Sei que não ajudei muito, mas é de grau 3 com certeza.

Enviado do Email para Windows 10

De: marcone augusto araújo borges
Enviado:quinta-feira, 16 de janeiro de 2020 19:47
Para: obm-l@mat.puc-rio.br
Assunto: [obm-l] Uma soma

Como calcular 1 + (1+2) + (1+2+3) +... +(1+2+...+n)? 
-- 
Esta mensagem foi verificada pelo sistema de antivírus e 
acredita-se estar livre de perigo. 


-- 
Esta mensagem foi verificada pelo sistema de antiv�rus e
 acredita-se estar livre de perigo.



RES: [obm-l]

2019-11-28 Por tôpico Jamil Silva
Qual o raciocínio que leva a esse resultado ?

Enviado do Email para Windows 10


De: owner-ob...@mat.puc-rio.br  em nome de Esdras 
Muniz 
Enviado: Thursday, November 28, 2019 6:18:00 PM
Para: obm-l@mat.puc-rio.br 
Assunto: Re: [obm-l]

Acho que é (2019!)/(2^{1000}×1009!).

Em qui, 28 de nov de 2019 12:41, Jamil Silva 
mailto:jamildasi...@hotmail.com.br>> escreveu:

Qual o menor número que  possui exatamente 2019 partições tal que em todas elas 
as partes sejam números inteiros positivos e consecutivos ?

--
Esta mensagem foi verificada pelo sistema de antivírus e
acredita-se estar livre de perigo.

--
Esta mensagem foi verificada pelo sistema de antiv�rus e
acredita-se estar livre de perigo.

-- 
Esta mensagem foi verificada pelo sistema de antiv�rus e
 acredita-se estar livre de perigo.



RES: [obm-l]

2019-11-28 Por tôpico Jamil Silva
Ok...vou  pensar  tbm a partir do que vc já fez..valeu pela heurística

Enviado do Email para Windows 10

De: Mauricio de Araujo
Enviado:quinta-feira, 28 de novembro de 2019 13:15
Para: obm-l@mat.puc-rio.br
Assunto: Re: [obm-l]

A impressão que eu tenho é a de que a quantidade de termos com i zeros passa a 
formar uma PA de terceira ordem.. 4 termos com 1 zero, 10 termos com 2 zeros e 
assim por diante... Não consegui provar, acho que teria de pensar mais...

S=(4,10,20,35,56,...)

Cheguei a esta ideia pensando assim:

Vamos concatenar os números x1, 1, x2, 3, x3, 9, x4 onde x1, x2, x3, x4 seriam 
a solução da equação x1 + x2 +x3 + x4 = Z (Z igual ao número de zeros que quero 
"distribuir")... outro jeito de ver seria assim: tenho de colocar zeros nos 
espaços a seguir _1_3_9_... chamei o primeiro espaço de x1, o segundo de x2, o 
terceiro de x3 e o quarto de x4.

A equação x1 + x2 + x3 + x4 = 1 tem 4 soluções => 4 maneiras de escrever com 1 
zero os termos da sequencia do Joaozinho (4!/3!)

A equação x1 + x2 + x3 + x4 = 2 tem 10 soluções => 10 maneiras de escrever com 
2 zeros os termos da sequencia do Joaozinho (5!/(3!.2!))

A equação x1 + x2 + x3 + x4 = 3 tem 20 soluções => 20 maneiras de escrever com 
3 zeros os termos da sequencia do Joaozinho. (6!/(3!.3!))

e assim por diante...

Aí é encontrar no braço o termo geral da sequencia S e a soma dos seus n 
primeiros termos (polinômio de grau 3)... Acho que dá trabalho mas dá para 
chegar no resultado... vou ficar devendo ele :)




Att,
__
Mauricio de Araujo
mailto:mauricio.de.ara...@gmail.com>>


Em qui., 28 de nov. de 2019 às 12:17, Jamil Silva 
mailto:jamildasi...@hotmail.com.br>> escreveu:
E se fosse:

20139, 21039, 21309, 21390, 200139, 201039, 201309, 201390, 210039, 210309, 
210390, 213090, 213900, ...
Qual o 2020º termo, mantendo as propriedades anteriores acrescidas do algarismo 
3(três) em qualquer posição entre um e nove ?


Enviado do Email para Windows 10


--
Esta mensagem foi verificada pelo sistema de antivírus e
acredita-se estar livre de perigo.

--
Esta mensagem foi verificada pelo sistema de antiv�rus e
acredita-se estar livre de perigo.


-- 
Esta mensagem foi verificada pelo sistema de antiv�rus e
 acredita-se estar livre de perigo.



[obm-l] RES: [obm-l] Nova competição de matemática parceira da OBM

2018-09-10 Por tôpico Thiago Santana
Muito interessante.

 

Pena que é apenas a partir da 8., tenho que aguardar para inscrever meu garoto.

 

TS

 

De: owner-ob...@mat.puc-rio.br  Em nome de samuel 
barbosa
Enviada em: terça-feira, 4 de setembro de 2018 18:27
Para: obm-l@mat.puc-rio.br
Assunto: [obm-l] Nova competição de matemática parceira da OBM

 

Caros,

 

Para garantir recursos para as olimpíadas regionais desse ano, a OBM criou uma 
parceria com a empresa Multilaser. Além dos recursos destinados às regionais, a 
equipe de professores que elabora a OBM está colaborando com uma nova 
competição, que é totalmente online, chamada Copa Multilaser:

 

http://www.copamultilaser.com.br/

 

A inscrição é gratuita e o sucesso na parceria pode ser útil para a OBM nos 
próximos anos. 

 

Abraços

Samuel

 


-- 
Esta mensagem foi verificada pelo sistema de antiv�s e 
acredita-se estar livre de perigo. 


-- 
Esta mensagem foi verificada pelo sistema de antiv�rus e
 acredita-se estar livre de perigo.



[obm-l] RES: [obm-l] ensino de matemática

2018-07-20 Por tôpico Luiz Antonio Rodrigues
Olá pessoal!
Bom dia!
Eu havia pensado em produzir um pequeno texto sobre a minha graduação, mas
acho interessante começar a discussão com algo que aconteceu no primeiro
dia de aula da meu curso de Licenciatura...
Numa sala de 50 pessoas, uma boa parte dos alunos respondeu 'não' a uma
pergunta da professora de Laboratório de Matemática: 'Quem pretende ser
professor de matemática?'. Eu fiquei espantado, mas pior foi no dia da
Colação de Grau: o número de formandos não chegava a 10...
O que vocês pensam sobre isso?
Um abraço!
Luiz Antonio

On Thu, Jul 19, 2018, 11:39 PM Anderson Torres 
wrote:

> Opa! Mantenham-me informado!
> Em seg, 16 de jul de 2018 às 12:39, Manoel Cesar Valente Lopes
>  escreveu:
> >
> > Me inclua nesta discussão!
> >
> >
> >
> > De: owner-ob...@mat.puc-rio.br  Em nome de
> Claudio Buffara
> > Enviada em: Wednesday, July 11, 2018 12:30 PM
> > Para: obm-l@mat.puc-rio.br
> > Assunto: [obm-l] ensino de matemática
> >
> >
> >
> > Prezados colegas da lista:
> >
> >
> >
> > Entendo que o tema pode ser off-topic pois não trata especificamente de
> problemas olímpicos, mas aqui vai de qualquer forma...
> >
> >
> >
> > Algum de vocês se interessa pelo ensino de matemática (escolar ou
> universitário)?
> >
> >
> >
> > Pergunto porque há anos tenho pensado na melhor forma de ensinar
> matemática (principalmente em termos de composição do currículo e de
> apresentação dos tópicos nos livros didáticos), estou convencido de que não
> estamos fazendo certo, nem na escola e nem na universidade, e gostaria de
> ter gente interessada pra debater idéias e, quem sabe, elaborar algum
> projeto mais concreto.
> >
> >
> >
> > Em linhas gerais, discordo da ordem em que os assuntos são abordados, na
> maioria dos livros.
> >
> > O foco é muito mais na ordem lógica (seguindo o rigor do método
> axiomático, mesmo em livros pra ensino médio) sem nenhuma preocupação:
> >
> > - com a motivação para os resultados que são apresentados (e, nos
> ensinos fundamental e médio, quase nunca demonstrados);
> >
> > - com tornar estes resultados intuitivos para o estudante.
> >
> >
> >
> > Também acho que certos assuntos deveriam ser incluídos e outros
> excluídos do currículo, mas este, pra mim, é um problema menor. Pois,
> qualquer que seja o tópico, se for bem ensinado e incentivar o aluno a
> pensar, já tá valendo.
> >
> >
> >
> > A meu ver, seria ideal se cada tópico do currículo de matemática fosse
> apresentado seguindo a sequência:
> >
> > identificação de padrões ("patterns") ==> formulação de conjecturas ==>
> demonstração destas conjecturas.
> >
> > Pois esta é a maneira como a matemática é criada.
> >
> > Mas acho que muito poucos professores estão capacitados pra ensinar
> matemática deste jeito.
> >
> >
> >
> > Em particular, no Ensino Médio, a ênfase nos últimos anos tem sido na
> tal contextualização, que pode ser vista em todo o seu esplendor nas provas
> do Enem.
> >
> > O resultado disso me parece ser um retrocesso na formação matemática dos
> alunos e também a disseminação da mentalidade de que a única matemática que
> deve ser estudada é aquela que é usada no dia-a-dia dos cidadãos comuns.
> >
> >
> >
> > E, na universidade, a coisa não é muito melhor, mesmo num assunto que só
> é visto na graduação em matemática. a análise real.
> >
> > Vejam só:
> >
> > Os livros tratam da topologia da reta antes de conceitos tais como
> compacidade e conexidade se mostrarem realmente necessários (o que, de
> fato, só ocorre em dimensão > 1; na reta, quase tudo pode ser demonstrado
> com base em sequências e no método da bisseção, que são coisas bastante
> intuitivas, mas que quase nunca são usadas).
> >
> >
> >
> > Limites e continuidade podem ser introduzidos também com base em
> sequências, interpretando-se os epsilons como margens de erro em
> aproximação.
> >
> >
> >
> > Aliás, a noção de que análise nada mais é do que uma teoria de
> aproximações quase nunca é mencionada.
> >
> > Por exemplo, foi só estudando a análise do R^n é que eu me dei conta de
> que a derivada é uma aproximação de uma função arbitrária por uma função
> afim.
> >
> > Antes disso, eu só sabia que "derivada = inclinação da reta tangente".
> >
> >
> >
> > Os livros também mencionam critérios de convergência de séries
> (Dirichlet, Abel, etc.) que vêm do nada (pois foram inventados para o
> estudo de séries de Fourier, que estes liros não abordam).
> >
> >
> >
> > E o principal resultado sobre convergência de séries de potências
> decorre quase trivialmente do estudo das PGs infinitas (assunto de Ensino
> Médio). Mas qual livro deixa isso explícito?
> >
> >
> >
> > E, pra terminar, poucos têm uma figura para ilustrar o teorema
> fundamental do cálculo que, com uma figura bem feita, fica bem intuitivo.
> No entanto, a análise na reta em geral é apresentada com um caráter
> aritmético/algébrico, mas quase nunca geométrico.
> >
> >
> >
> > Obrigado pela atenção.
> >
> >
> >
> > []s,
> >
> > Claudio.
> >
> >
> > --
> > Esta mensagem foi verificada 

[obm-l] Re: [obm-l] RES: [obm-l] ensino de matemática

2018-07-19 Por tôpico Anderson Torres
Opa! Mantenham-me informado!
Em seg, 16 de jul de 2018 às 12:39, Manoel Cesar Valente Lopes
 escreveu:
>
> Me inclua nesta discussão!
>
>
>
> De: owner-ob...@mat.puc-rio.br  Em nome de 
> Claudio Buffara
> Enviada em: Wednesday, July 11, 2018 12:30 PM
> Para: obm-l@mat.puc-rio.br
> Assunto: [obm-l] ensino de matemática
>
>
>
> Prezados colegas da lista:
>
>
>
> Entendo que o tema pode ser off-topic pois não trata especificamente de 
> problemas olímpicos, mas aqui vai de qualquer forma...
>
>
>
> Algum de vocês se interessa pelo ensino de matemática (escolar ou 
> universitário)?
>
>
>
> Pergunto porque há anos tenho pensado na melhor forma de ensinar matemática 
> (principalmente em termos de composição do currículo e de apresentação dos 
> tópicos nos livros didáticos), estou convencido de que não estamos fazendo 
> certo, nem na escola e nem na universidade, e gostaria de ter gente 
> interessada pra debater idéias e, quem sabe, elaborar algum projeto mais 
> concreto.
>
>
>
> Em linhas gerais, discordo da ordem em que os assuntos são abordados, na 
> maioria dos livros.
>
> O foco é muito mais na ordem lógica (seguindo o rigor do método axiomático, 
> mesmo em livros pra ensino médio) sem nenhuma preocupação:
>
> - com a motivação para os resultados que são apresentados (e, nos ensinos 
> fundamental e médio, quase nunca demonstrados);
>
> - com tornar estes resultados intuitivos para o estudante.
>
>
>
> Também acho que certos assuntos deveriam ser incluídos e outros excluídos do 
> currículo, mas este, pra mim, é um problema menor. Pois, qualquer que seja o 
> tópico, se for bem ensinado e incentivar o aluno a pensar, já tá valendo.
>
>
>
> A meu ver, seria ideal se cada tópico do currículo de matemática fosse 
> apresentado seguindo a sequência:
>
> identificação de padrões ("patterns") ==> formulação de conjecturas ==> 
> demonstração destas conjecturas.
>
> Pois esta é a maneira como a matemática é criada.
>
> Mas acho que muito poucos professores estão capacitados pra ensinar 
> matemática deste jeito.
>
>
>
> Em particular, no Ensino Médio, a ênfase nos últimos anos tem sido na tal 
> contextualização, que pode ser vista em todo o seu esplendor nas provas do 
> Enem.
>
> O resultado disso me parece ser um retrocesso na formação matemática dos 
> alunos e também a disseminação da mentalidade de que a única matemática que 
> deve ser estudada é aquela que é usada no dia-a-dia dos cidadãos comuns.
>
>
>
> E, na universidade, a coisa não é muito melhor, mesmo num assunto que só é 
> visto na graduação em matemática. a análise real.
>
> Vejam só:
>
> Os livros tratam da topologia da reta antes de conceitos tais como 
> compacidade e conexidade se mostrarem realmente necessários (o que, de fato, 
> só ocorre em dimensão > 1; na reta, quase tudo pode ser demonstrado com base 
> em sequências e no método da bisseção, que são coisas bastante intuitivas, 
> mas que quase nunca são usadas).
>
>
>
> Limites e continuidade podem ser introduzidos também com base em sequências, 
> interpretando-se os epsilons como margens de erro em aproximação.
>
>
>
> Aliás, a noção de que análise nada mais é do que uma teoria de aproximações 
> quase nunca é mencionada.
>
> Por exemplo, foi só estudando a análise do R^n é que eu me dei conta de que a 
> derivada é uma aproximação de uma função arbitrária por uma função afim.
>
> Antes disso, eu só sabia que "derivada = inclinação da reta tangente".
>
>
>
> Os livros também mencionam critérios de convergência de séries (Dirichlet, 
> Abel, etc.) que vêm do nada (pois foram inventados para o estudo de séries de 
> Fourier, que estes liros não abordam).
>
>
>
> E o principal resultado sobre convergência de séries de potências decorre 
> quase trivialmente do estudo das PGs infinitas (assunto de Ensino Médio). Mas 
> qual livro deixa isso explícito?
>
>
>
> E, pra terminar, poucos têm uma figura para ilustrar o teorema fundamental do 
> cálculo que, com uma figura bem feita, fica bem intuitivo. No entanto, a 
> análise na reta em geral é apresentada com um caráter aritmético/algébrico, 
> mas quase nunca geométrico.
>
>
>
> Obrigado pela atenção.
>
>
>
> []s,
>
> Claudio.
>
>
> --
> Esta mensagem foi verificada pelo sistema de antiv�rus e
> acredita-se estar livre de perigo.
>
>
> --
> Esta mensagem foi verificada pelo sistema de antivírus e
> acredita-se estar livre de perigo.

-- 
Esta mensagem foi verificada pelo sistema de antiv�rus e
 acredita-se estar livre de perigo.


=
Instru��es para entrar na lista, sair da lista e usar a lista em
http://www.mat.puc-rio.br/~obmlistas/obm-l.html
=


[obm-l] RES: [obm-l] ensino de matemática

2018-07-16 Por tôpico Manoel Cesar Valente Lopes
Me inclua nesta discussão!

De: owner-ob...@mat.puc-rio.br  Em nome de Claudio 
Buffara
Enviada em: Wednesday, July 11, 2018 12:30 PM
Para: obm-l@mat.puc-rio.br
Assunto: [obm-l] ensino de matemática

Prezados colegas da lista:

Entendo que o tema pode ser off-topic pois não trata especificamente de 
problemas olímpicos, mas aqui vai de qualquer forma...

Algum de vocês se interessa pelo ensino de matemática (escolar ou 
universitário)?

Pergunto porque há anos tenho pensado na melhor forma de ensinar matemática 
(principalmente em termos de composição do currículo e de apresentação dos 
tópicos nos livros didáticos), estou convencido de que não estamos fazendo 
certo, nem na escola e nem na universidade, e gostaria de ter gente interessada 
pra debater idéias e, quem sabe, elaborar algum projeto mais concreto.

Em linhas gerais, discordo da ordem em que os assuntos são abordados, na 
maioria dos livros.
O foco é muito mais na ordem lógica (seguindo o rigor do método axiomático, 
mesmo em livros pra ensino médio) sem nenhuma preocupação:
- com a motivação para os resultados que são apresentados (e, nos ensinos 
fundamental e médio, quase nunca demonstrados);
- com tornar estes resultados intuitivos para o estudante.

Também acho que certos assuntos deveriam ser incluídos e outros excluídos do 
currículo, mas este, pra mim, é um problema menor. Pois, qualquer que seja o 
tópico, se for bem ensinado e incentivar o aluno a pensar, já tá valendo.

A meu ver, seria ideal se cada tópico do currículo de matemática fosse 
apresentado seguindo a sequência:
identificação de padrões ("patterns") ==> formulação de conjecturas ==> 
demonstração destas conjecturas.
Pois esta é a maneira como a matemática é criada.
Mas acho que muito poucos professores estão capacitados pra ensinar matemática 
deste jeito.

Em particular, no Ensino Médio, a ênfase nos últimos anos tem sido na tal 
contextualização, que pode ser vista em todo o seu esplendor nas provas do Enem.
O resultado disso me parece ser um retrocesso na formação matemática dos alunos 
e também a disseminação da mentalidade de que a única matemática que deve ser 
estudada é aquela que é usada no dia-a-dia dos cidadãos comuns.

E, na universidade, a coisa não é muito melhor, mesmo num assunto que só é 
visto na graduação em matemática. a análise real.
Vejam só:
Os livros tratam da topologia da reta antes de conceitos tais como compacidade 
e conexidade se mostrarem realmente necessários (o que, de fato, só ocorre em 
dimensão > 1; na reta, quase tudo pode ser demonstrado com base em sequências e 
no método da bisseção, que são coisas bastante intuitivas, mas que quase nunca 
são usadas).

Limites e continuidade podem ser introduzidos também com base em sequências, 
interpretando-se os epsilons como margens de erro em aproximação.

Aliás, a noção de que análise nada mais é do que uma teoria de aproximações 
quase nunca é mencionada.
Por exemplo, foi só estudando a análise do R^n é que eu me dei conta de que a 
derivada é uma aproximação de uma função arbitrária por uma função afim.
Antes disso, eu só sabia que "derivada = inclinação da reta tangente".

Os livros também mencionam critérios de convergência de séries (Dirichlet, 
Abel, etc.) que vêm do nada (pois foram inventados para o estudo de séries de 
Fourier, que estes liros não abordam).

E o principal resultado sobre convergência de séries de potências decorre quase 
trivialmente do estudo das PGs infinitas (assunto de Ensino Médio). Mas qual 
livro deixa isso explícito?

E, pra terminar, poucos têm uma figura para ilustrar o teorema fundamental do 
cálculo que, com uma figura bem feita, fica bem intuitivo. No entanto, a 
análise na reta em geral é apresentada com um caráter aritmético/algébrico, mas 
quase nunca geométrico.

Obrigado pela atenção.

[]s,
Claudio.

--
Esta mensagem foi verificada pelo sistema de antiv�rus e
acredita-se estar livre de perigo.

-- 
Esta mensagem foi verificada pelo sistema de antiv�rus e
 acredita-se estar livre de perigo.



RES: [obm-l] Geometria

2018-04-12 Por tôpico Claudio Arconcher
Caros colegas, se bem entendi, o ponto D não pode ser marcado sobre a reta, ele 
deve ser construído.
A construção do ponto D é simples: tome-se o ponto Q`, simétrico do ponto Q, 
com relação à reta suporte dos pontos A,B e C, o quadrilátero PQRQ` é cíclico 
já que o ângulo BQ`C mede 60º e o ângulo PQR deve ser de 120º.
A intersecção dessa circunferência com a  reta por C paralela à reta BQ, 
fazendo 60º com a suporte mencionada, produz o ponto R, lado do terceiro 
triângulo equilátero CDR, aí sai o ponto D.
Uma construção utilizando o Geogebra mostra que as medidas deveriam ser 3,5 e 
x, e não 5,3 e x .
Por favor confiram.
Abraço.
Claudio

De: owner-ob...@mat.puc-rio.br [mailto:owner-ob...@mat.puc-rio.br] Em nome de 
Douglas Oliveira de Lima
Enviada em: quinta-feira, 12 de abril de 2018 08:56
Para: obm-l@mat.puc-rio.br
Assunto: [obm-l] Geometria

Caros amigos , tenho um problema bem legal e estou compartilhando. Ai vai:

Numa reta marcam-se os pontos A,B,C,D nesta ordem , e no mesmo semiplano 
constroem-se os triângulos equiláteros ABP, BCQ e CDR de lados 5, 3 e x 
respectivamente, sendo o angulo PQR igual a 120 graus, determine x.



Será que teria alguma construção bonita para solucionå-lo?

Abraco
Douglas Oliveira.

--
Esta mensagem foi verificada pelo sistema de antiv�rus e
acredita-se estar livre de perigo.

[https://ipmcdn.avast.com/images/icons/icon-envelope-tick-round-orange-animated-no-repeat-v1.gif]
  Livre de vírus. 
www.avast.com.

-- 
Esta mensagem foi verificada pelo sistema de antiv�rus e
 acredita-se estar livre de perigo.



RES: [obm-l] Perguntas pro Claudio Buffara

2018-04-10 Por tôpico bouskela
Olá a todos!

Envio esta mensagem para a Lista da OBM a fim de manifestar minha concordância 
com a mensagem postada pelo ARTUR STEINER e acrescentar (ratificar) dois pontos:

Primeiro ponto: — A utilidade das descobertas esdrúxulas e a sua aplicabilidade 
prática:

i=sqrt(-1)
Por volta de 1777, Euler resolveu criar uma raiz quadrada para -1 (uma tremenda 
maluquice!). Bem, esta “maluquice” — a criação dos Números Complexos — 
mostrou-se, posteriormente, como a base para o projeto de circuitos 
eletroeletrônicos e, sem ela, eu não poderia estar na frente desse computador 
(que não existiria). É fundamental, também, para o dimensionamento de 
estruturas (prédios, pontes etc.) submetidas a eventos dinâmicos (furacões, 
terremotos etc.).

Geometrias não-euclidianas
Em meados do século IXX, Riemann resolveu criar geometrias não-euclidianas 
(e.g., as geometrias elíptica e hiperbólica) — uma verdadeira blasfêmia, porque 
se contrapôs ao 5º Postulado (o das Paralelas) de Euclides. Mais tarde, no 
início do século XX, essas geometrias serviram para a formulação matemática da 
Teoria da Relatividade de Einstein.

Dito isto, fica o teorema (a ser ainda provado): — Toda descoberta matemática 
tem (ou terá) alguma aplicabilidade prática!

Segundo ponto: — O objetivo desta Lista:

O objetivo desta Lista é discutir a solução dos chamados problemas “olímpicos” 
(que, naturalmente, são difíceis) e, por conseguinte, servir de apoio para os 
estudantes que participam de olimpíadas matemáticas. Tenho a certeza de que é 
um objetivo plenamente válido.

Não obstante, esta Lista serve, também, para o deleite de todos (eu e tantos 
outros), que sejam amantes (no sentido metafórico) da Matemática.

Saudações!
Albert Bouskelá
mailto:bousk...@gmail.com

De: owner-ob...@mat.puc-rio.br  Em nome de Artur 
Steiner
Enviada em: terça-feira, 10 de abril de 2018 15:42
Para: obm-l@mat.puc-rio.br
Assunto: Re: [obm-l] Perguntas pro Claudio Buffara

Eu gostaria dr citar alguns pontos de caráter geral sobre sua perguntas 

Julgo conveniente lembrar que esta é uma lista para amantes da matemática, 
assim como há para amantes de música, jardinagem, literatura, etc. Participa 
que quiser. Ninguém é obrigado a resolver os problemas aqui disticutidos. Isto 
não aliena ninguém. Para os que têm interesse em algum ramo do conhecimento, é 
sempre bom interagir com quem tem talento no assunto. Não se trata de 
exibicionismo.  Muitos adeptos da Física gostariam de ter interagido com 
Stephen Hawking. 

Problemas olímpicos, assim como tópicos avançados na matemática, são estudados 
por quem nisso tem interesse. Concordo que nem sempre apresentam uma aplicação 
prática, pelo menos uma claramente identificável. Seria algo como perguntar " 
qual a utilidade da música clássica? Qual a de se aprender latim?" Mas para os 
aficcionados, há uma utilidade real e palpável: torna-os mais felizes, 
causa-lhes satisfação.

No caso da matemática, muito do que temos hoje só existe porque um dia, por 
exemplo, alguém resolveu a "inutilidade" de uma equação diferencial. 

Dois exemplos: estes complicados problemas sobre números primos aplicam-se em 
criptografia, para que transações bancárias possam ser feitas com uma boa dose 
de segurança. E há algum tempo vim a saber que, na elaboração das fontes de 
computador , cujos tamanhos variam conforme desejamos, sempre mantendo intacta 
a proporção, utiliza-se teoria dos fractais, algo ligado a análise complexa 
avançada, algo que, para muitos, é uma total perda de tempo.

Apenas algumas reflexões. 

Abraços

Artur Costa Steiner

Em Ter, 10 de abr de 2018 13:17, Marcela Costa 
 escreveu:
Caros participantes da lista obm-l.

Tenho seguido esta lista lendo as mensagens de fora há algum tempo e fiquei 
cismada com duas mensagens que o participante Claudio Buffara enviou em 23 de 
março ( https://www.mail-archive.com/obm-l@mat.puc-rio.br/msg55232.html ) e 25 
de março ( https://www.mail-archive.com/obm-l@mat.puc-rio.br/msg55196.html), a 
respeito do ensino de matemática e decidi participar.

Dessa forma, tenho as seguintes perguntas pra ele:

1) O Sr. diz que produtos notáveis e fatorações são "notoriamente mal 
ensinados". O Sr. tem alguma sugestão de como ensinar melhor estes tópicos?

2) O Sr. não acha um pouco arrogante fazer uma afirmação como esta, já que o 
Sr. tem um talento claramente acima da média em matemática e pertence à elite 
dos "olímpicos"?

3) O Sr. não acha que o exibicionismo com estes problemas dificílimos acaba por 
alienar os alunos normais?

4) Qual a aplicabilidade na vida real de problemas de olimpíadas de matemática?

Sds

-- 
Esta mensagem foi verificada pelo sistema de antivírus e 
acredita-se estar livre de perigo. 

-- 
Esta mensagem foi verificada pelo sistema de antiv�s e 
acredita-se estar livre de perigo. 


-- 
Esta mensagem foi verificada pelo sistema de antiv�rus e
 acredita-se estar livre de perigo.



RES: [obm-l] Geometria plana

2018-04-02 Por tôpico Claudio Arconcher
Bom dia caros colegas.
Ponhamos ABCD o quadrado (o ponto A está no lado de baixo e à esquerda, 
segue-se o ponto B à direita, C e D estão no lado de acima fechando o circuito 
ABCD ).
Ponhamos: AP=x e AQ=y, segue-se, QD=1-y e PB=1-x.
Tracemos a circunferência de centro C e raio 1, ela tangencia AD em D e AB em 
B, agora seja M um ponto no quarto dessa circunferência interno ao quadrado 
ABCD e tracemos a tangente a ela por M, cortando AD em Q e AB em P ( serão, de 
fato os pontos esperados ), tem-se: QD=1-y = QM e PB=1 – x = PM, o perímetro do 
triângulo retângulo QAP é igual a 2. Reciprocamente se consideramos o triângulo 
AQP de perímetro 2 fixado antes o ponto M será o mesmo, todos esses triângulos 
são assim obtidos, com PQ tangente à circunferência em um ponto M com a 
propriedade descrita.
Agora basta examinar as congruências dos triângulos retângulos CDQ e CMQ e, 
também, CBP e CMP, isso nos leva a concluir que o ângulo PCQ mede 45 º.
Espero que o “coelhinho da Páscoa” concorde comigo.
Abraço.
Cláudio.

De: owner-ob...@mat.puc-rio.br [mailto:owner-ob...@mat.puc-rio.br] Em nome de 
Douglas Oliveira de Lima
Enviada em: domingo, 1 de abril de 2018 17:25
Para: obm-l@mat.puc-rio.br
Assunto: [obm-l] Geometria plana

Olá amigos, pra quem gosta de geometria plana, compartilhando aqui uma questão 
do coelhinho da páscoa que achei legal.

1) Em um quadrado ABCD de lado unitário tomam-se os pontos P e Q sobre os lados 
AB e AD respectivamente, de modo que o perímetro do triângulo APQ seja igual a 
2. Calcule a medida do ângulo  PCQ.

Um abraço

Douglas Oliveira.

--
Esta mensagem foi verificada pelo sistema de antiv�rus e
acredita-se estar livre de perigo.

[https://ipmcdn.avast.com/images/icons/icon-envelope-tick-round-orange-animated-no-repeat-v1.gif]
  Livre de vírus. 
www.avast.com.

-- 
Esta mensagem foi verificada pelo sistema de antiv�rus e
 acredita-se estar livre de perigo.



[obm-l] RES: [obm-l] Re: [obm-l] Raízes transcendentes

2018-03-21 Por tôpico bouskela
Olá!

Pois é, a equação a^x=x^a; sendo “a” real, positivo e maior do que zero é muito 
interessante.

1) Quando a=e, esta equação tem uma única raiz: x=e;
2) Quando a=1, esta equação tem uma única raiz: x=1;
3) Quando a=2, esta equação tem três raízes: x=2, 4 e mais uma transcendente 
(-0.746958…)
4) Quando a=4, esta equação tem três raízes: x=2, 4 e mais uma transcendente 
(-0.746958…)
5) Quando ae)
6) Quando a>e, esta equação tem duas raízes: x=a e mais uma transcendente (x Em nome de Claudio 
Buffara
Enviada em: quarta-feira, 21 de março de 2018 17:51
Para: obm-l@mat.puc-rio.br
Assunto: [obm-l] Re: [obm-l] Raízes transcendentes

Tá certo isso? Pois, para todo n natural, n sempre é raiz de x^n = n^x.

2018-03-21 16:45 GMT-03:00 Artur Steiner :
Mostre que, para todo inteiro n >= 3, n diferente de 4, as raÃzes reais da 
equação x^n = n^x são transcendentes.

Artur

Enviado do meu iPad
--
Esta mensagem foi verificada pelo sistema de antivírus e
 acredita-se estar livre de perigo.


=
Instruções para entrar na lista, sair da lista e usar a lista em
http://www.mat.puc-rio.br/~obmlistas/obm-l.html
=


-- 
Esta mensagem foi verificada pelo sistema de antiv�s e 
acredita-se estar livre de perigo. 


-- 
Esta mensagem foi verificada pelo sistema de antiv�rus e
 acredita-se estar livre de perigo.


=
Instru��es para entrar na lista, sair da lista e usar a lista em
http://www.mat.puc-rio.br/~obmlistas/obm-l.html
=


[obm-l] RES: [obm-l] Re: [obm-l] Raízes transcendentes

2018-03-21 Por tôpico bouskela
Olá!

 

Pois é, a equação a^x=x^a; sendo “a” real, positivo e maior do que zero é muito 
interessante.

 

1.  Quando a=e, esta equação tem uma única raiz: x=e;
2.  Quando a=1, esta equação tem uma única raiz: x=1;
3.  Quando a=2, esta equação tem três raízes: x=2, 4 e mais uma 
transcendente (-0.746958…)
4.  Quando a=4, esta equação tem três raízes: x=2, 4 e mais uma 
transcendente (-0.746958…)
5.  Quando ae)
6.  Quando a>e, esta equação tem duas raízes: x=a e mais uma transcendente 
(x bousk...@gmail.com

 

De: owner-ob...@mat.puc-rio.br  Em nome de Claudio 
Buffara
Enviada em: quarta-feira, 21 de março de 2018 17:51
Para: obm-l@mat.puc-rio.br
Assunto: [obm-l] Re: [obm-l] Raízes transcendentes

 

Tá certo isso? Pois, para todo n natural, n sempre é raiz de x^n = n^x.

 

2018-03-21 16:45 GMT-03:00 Artur Steiner :

Mostre que, para todo inteiro n >= 3, n diferente de 4, as raízes reais da 
equação x^n = n^x são transcendentes.

Artur

Enviado do meu iPad
--
Esta mensagem foi verificada pelo sistema de antivírus e
 acredita-se estar livre de perigo.


=
Instruções para entrar na lista, sair da lista e usar a lista em
http://www.mat.puc-rio.br/~obmlistas/obm-l.html
=

 


-- 
Esta mensagem foi verificada pelo sistema de antiv�s e 
acredita-se estar livre de perigo. 


-- 
Esta mensagem foi verificada pelo sistema de antiv�rus e
 acredita-se estar livre de perigo.



[obm-l] RES: [obm-l] Re: [obm-l] Re: Indução dúvida

2016-01-20 Por tôpico Albert Bouskela - YMAIL
Prezados,

A estrutura pode até estar correta, mas, tal como colocada, ela só complica as 
coisas. Por exemplo, passa a ser necessário conhecer (i.e., determinar) 
NÃO[P(n+1)] e isto pode não ser trivial! Mesmo que seja, não acho um bom 
caminho. Vou dar um exemplo: pegar o Último Teorema de Fermat e tentar prová-lo 
por Indução Finita:

Último Teorema de Fermat: x^n + y^n =/ z^n ; "x", "y" e "z" são naturais E "n" 
também é natural E n>=3.
=/ é "diferente"
>= é "maior OU igual"

* Método Convencional:
1) O caso-base refere-se a: n=3;
2) Provamos que: x^3 + y^3 =/ z^3 — não é muito difícil...
3) Supomos que: x^n + y^n =/ z^n (n>=3);
4) Daí vamos tentar provar que: x^(n+1) + y^(n+1) =/ z^(n+1) — bem, é muito 
difícil e, até agora, ninguém conseguiu (a prova apresentada pelo Andrew Wiles 
passa longe disto...).
5) Apesar desta "pequena" dificuldade, pode ser um caminho...

* Método Proposto:
1) Idem;
2) Idem;
3) Idem;
E aí começa a complicação:
4) NÃO[P(n+1)]: x^(n+1) + y^(n+1) = z^(n+1) — esta até que é trivial...
5) Aí temos que buscar uma contradição. Por exemplo, provar que x^(n+1) + 
y^(n+1) =/ z^(n+1). Mas, aí voltamos ao Método Convencional. Não valeu!
6) Podemos, então, partir para a equivalência: P->Q é equivalente a: ~PvQ. 
Vamos lá:
7) P(n)->P(n+1) é equivalente a: ~P(n)vP(n+1)
8)~P(n): x^n + y^n = z^n (n>=3); P(n+1): x^(n+1) + y^(n+1) =/ z^(n+1)
9) Finalmente: [ x^n + y^n = z^n (n>=3) ] OU [ x^(n+1) + y^(n+1) =/ z^(n+1) ]
10) Parece que não progredimos muito... 

Sds.,
Albert Bouskelá
mailto:bousk...@ymail.com

De: owner-ob...@mat.puc-rio.br [mailto:owner-ob...@mat.puc-rio.br] Em nome de 
Ralph Teixeira
Enviada em: terça-feira, 19 de janeiro de 2016 19:14
Para: obm-l@mat.puc-rio.br
Assunto: [obm-l] Re: [obm-l] Re: Indução dúvida

Sim, a estrutura me parece correta.

2016-01-18 15:47 GMT-02:00 Israel Meireles Chrisostomo 
:
Por exemplo, eu quero provar que f(n)>c para todo n inteiro.Então, eu provei o 
caso base,e considerei a hipótese de indução, suponha que é válido para um k 
que f(k)>c e supus que f(k+1)c e  f(k+1)c, o que é uma contradição, 
pois f(k+1) não pode ser maior e menor do que c ao mesmo tempo. Então, isto 
pode ser considerada uma prova correta?
Eu dei uma olhada em lógica e vi que a negação do condicional P->Q é P^~Q, ou 
seja ~(P->Q )P^~Q

Em 18 de janeiro de 2016 15:30, Israel Meireles Chrisostomo 
 escreveu:
Em uma prova por indução, eu devo provar que P(n) implica P(n+1).Eu posso fazer 
isso da seguinte forma: suponha que P(n) é verdadeira, e suponha que P(n+1) é 
falsa, mas ao supor que P(n) é verdadeira e P(n+1) é falsa isto implica que 
P(n+1) é verdadeira(contradição, pois supomos que P(n+1) é falsa e no entanto é 
verdadeira, uma proposição não pode ser falsa e verdadeira ao mesmo 
tempo)-tendo em vista que já provei o caso base, isto pode ser considerado uma 
prova?Isto me pareceu correto, mas não sei se está correto.Eu bem sei que posso 
provar a contra positiva, que é o caso "inverso" ao que eu estou falando.Mas 
esse caso também é uma prova?




=
Instru��es para entrar na lista, sair da lista e usar a lista em
http://www.mat.puc-rio.br/~obmlistas/obm-l.html
=


[obm-l] RES: [obm-l] Re: Indução dúvida

2016-01-18 Por tôpico Albert Bouskela - YMAIL
Olá!

 

Indução Finita:

 

1) Considere a proposição “P”, aplicada sobre um DETERMINADO número INTEIRO 
“m”.

2) Deve-se provar que P(m) é verdadeira.

3) Obs.: em geral, m=1.

4) Considere QUALQUER inteiro “n”, sendo n>m.

5) Hipótese de indução: P(n) é verdadeira. I.e., P(n) é verdadeira POR 
HIPÓTESE.

6) Deve-se provar que P(n+1) é verdadeira, considerando que P(m) é 
verdadeira (foi provado em “2”) E que P(n) é verdadeira (é a hipótese de 
indução).

7) I.e., deve-se provar que: SE [ P(m) E P(n) são verdadeiras ] ENTÃO [ 
P(n+1) é verdadeira ].

8) [ P(m) E P(n) são verdadeiras ] => [ P(n+1) é verdadeira ] (*).

9) Feita a prova supracitada, fica provado que: P(m) é verdadeira E P(n) é 
verdadeira para QUALQUER inteiro “n”, sendo n>m. 

 

(*) O item “8” é equivalente a provar que a proposição { NÃO [ P(m) e P(n) ] OU 
[ P(n+1) ] } é verdadeira.

 

Sds.,

  _  

Albert Bouskelá

  bousk...@ymail.com

 

De: owner-ob...@mat.puc-rio.br [mailto:owner-ob...@mat.puc-rio.br] Em nome de 
Israel Meireles Chrisostomo
Enviada em: segunda-feira, 18 de janeiro de 2016 15:47
Para: obm-l@mat.puc-rio.br
Assunto: [obm-l] Re: Indução dúvida

 

Por exemplo, eu quero provar que f(n)>c para todo n inteiro.Então, eu provei o 
caso base,e considerei a hipótese de indução, suponha que é válido para um k 
que f(k)>c e supus que f(k+1)c e  f(k+1)c, o que é uma contradição, 
pois f(k+1) não pode ser maior e menor do que c ao mesmo tempo. Então, isto 
pode ser considerada uma prova correta?
Eu dei uma olhada em lógica e vi que a negação do condicional P->Q é P^~Q, ou 
seja ~(P->Q )P^~Q

 

Em 18 de janeiro de 2016 15:30, Israel Meireles Chrisostomo 
 > escreveu:

Em uma prova por indução, eu devo provar que P(n) implica P(n+1).Eu posso fazer 
isso da seguinte forma: suponha que P(n) é verdadeira, e suponha que P(n+1) é 
falsa, mas ao supor que P(n) é verdadeira e P(n+1) é falsa isto implica que 
P(n+1) é verdadeira(contradição, pois supomos que P(n+1) é falsa e no entanto é 
verdadeira, uma proposição não pode ser falsa e verdadeira ao mesmo 
tempo)-tendo em vista que já provei o caso base, isto pode ser considerado uma 
prova?Isto me pareceu correto, mas não sei se está correto.Eu bem sei que posso 
provar a contra positiva, que é o caso "inverso" ao que eu estou falando.Mas 
esse caso também é uma prova?

 



[obm-l] RES: [obm-l] RE: [obm-l] Soma de números compostos

2015-12-12 Por tôpico Albert Bouskela - YMAIL
Olá!

Todos os naturais (n) obedecem à seguinte lei de formação:

n = soma [i=0, p] [k(i)x2^i]; k(i)={0, 1}

I.e., todos os naturais podem ser escritos como a soma de potências de 2.
Nesta soma, cada potência de 2 aparece uma, e somente uma, vez. Esta é uma
correspondência biunívoca entre o natural "n" e a respectiva soma das
potências de 2. 

E.g., 23 = (1x2^0) + (1x2^1) + (1x2^2 ) + (0x2^3 ) + (1x2^4)

Sds.,
Albert. 


De: owner-ob...@mat.puc-rio.br [mailto:owner-ob...@mat.puc-rio.br] Em nome
de Eduardo Henrique
Enviada em: sábado, 12 de dezembro de 2015 15:13
Para: obm-l@mat.puc-rio.br
Assunto: [obm-l] RE: [obm-l] Soma de números compostos

Cara, acho que todo natural ímpar maior que 11 se escreve como 9+2*n, n
natural.

Att., 
Eduardo

From: mailto:marconeborge...@hotmail.com
To: mailto:obm-l@mat.puc-rio.br
Subject: [obm-l] Soma de números compostos
Date: Sat, 12 Dec 2015 01:36:39 +
Mostre que todo inteiro n > 11 pode ser escrito como soma de números
compostos positivos

para n par : n = 11 + 2t-1 = 4 + [2(t + 3)]
mas...

-- 
Esta mensagem foi verificada pelo sistema de antivírus e 
acredita-se estar livre de perigo. 

-- 
Esta mensagem foi verificada pelo sistema de antivírus e 
acredita-se estar livre de perigo. 


-- 
Esta mensagem foi verificada pelo sistema de antivírus e
 acredita-se estar livre de perigo.


=
Instruções para entrar na lista, sair da lista e usar a lista em
http://www.mat.puc-rio.br/~obmlistas/obm-l.html
=


RES: [obm-l] relação trigonométrica

2015-10-22 Por tôpico Vitório Batista Lima da Silva
Boa tarde Luis

Essa relação de Napier pode ser vista no site 
http://math.tutorvista.com/trigonometry/napiers-analogy.html:

Abs,
Vitório

De: owner-ob...@mat.puc-rio.br [mailto:owner-ob...@mat.puc-rio.br] Em nome de 
Luís
Enviada em: quinta-feira, 22 de outubro de 2015 08:22
Para: obm-l@mat.puc-rio.br
Assunto: [obm-l] relação trigonométrica

Sauda,c~oes,

Considere o triângulo ABC com b>c e o ângulo
D = (B-C)/2.

Como provar que tan D = ((b-c)/(b+c)) cot(A/2) ?

Abs,
Luís

--
Esta mensagem foi verificada pelo sistema de antivírus e
acredita-se estar livre de perigo.

-- 
Esta mensagem foi verificada pelo sistema de antivírus e
 acredita-se estar livre de perigo.



[obm-l] RE: RES: [obm-l] relação trigonométrica

2015-10-22 Por tôpico Luís
Sauda,c~oes, oi Vitório, 

Ia mandar a mensagem abaixo quando vi a sua resposta. 
Vou ver o seu link em seguida. 

Minha mensagem param outro site:

===
Solved. 

It is known that in triangle ABC
tan ((B-C)/2) = (b-c)/(r+r_a) and tan(A/2)=r/(p-a) = r_a/p. 

Then r+r_a=(b+c) tan(A/2) and the result follows. 
===

Abs, 

Luís 



From: vitorio.si...@trf1.jus.br
To: obm-l@mat.puc-rio.br
Date: Thu, 22 Oct 2015 15:42:45 -0200
Subject: RES: [obm-l] relação trigonométrica
















Boa tarde Luis

 

Essa relação de Napier pode ser vista no
site http://math.tutorvista.com/trigonometry/napiers-analogy.html:

 

Abs,

Vitório









De: owner-ob...@mat.puc-rio.br
[mailto:owner-ob...@mat.puc-rio.br] Em nome de Luís

Enviada em: quinta-feira, 22 de
outubro de 2015 08:22

Para: obm-l@mat.puc-rio.br

Assunto: [obm-l] relação
trigonométrica



 



Sauda,c~oes, 



Considere o triângulo ABC com b>c e o ângulo 

D = (B-C)/2. 



Como provar que tan D = ((b-c)/(b+c)) cot(A/2) ? 



Abs, 

Luís 





-- 

Esta mensagem foi verificada pelo sistema de antivírus e 

acredita-se estar livre de perigo. 




--

Esta mensagem foi verificada pelo sistema de antivírus e 

 acredita-se estar livre de perigo.   
-- 
Esta mensagem foi verificada pelo sistema de antivírus e
 acredita-se estar livre de perigo.



Re: [obm-l] RES: [obm-l] Düvida sobre o infinito de Cantor

2015-10-14 Por tôpico antoniogo
Obrigado pela explicação Artur Steiner. Pelo que você falou, a 
explicação dele não tem nem sentido, correto?


sds


Antonio G Oliveira

On 2015-10-14 14:04, Vitório Batista Lima da Silva wrote:

Eita ...maldade Steiner rsrsrsrs

-Mensagem original-
De: owner-ob...@mat.puc-rio.br [mailto:owner-ob...@mat.puc-rio.br] Em
nome de Artur Costa Steiner
Enviada em: quarta-feira, 14 de outubro de 2015 13:37
Para: obm-l@mat.puc-rio.br
Assunto: Re: [obm-l] Düvida sobre o infinito de Cantor

O  filósofo, assim ele se proclama, Olavo de Carvalho devia se abster
de falar sobre o que não conhece. O conjunto dos inteiros e o dos
pares  são conceitos matematicamente distintos. Não são iguais
simplesmente porque nem todo inteiro é par.

Eles tem a mesma cardinalidade, há uma bijeção entre eles.  Uma das
características de conjuntos infinitos é terem a mesma cardinalidade
que um de seus subconjuntos próprios.

Não tem nada a ver com a parte ser igual ao todo. Aliás, para dizer
isto de forma matematicamente correta, é preciso definir o que
significa ser maior. No caso de conjuntos, costuma-se às vezes dizer
que A é menor que B se A for subconjunto próprio de B. Sendo assim, o
o conjunto dos pares é menor do que o dos inteiros.

Para deixar Olavo ainda mais louco, diga a ele que no intervalo (0, 1)
há tantos elementos quanto em toda a reta real. E também tantos quanto
em todo o espaço R^3... Segundo ele, isto implica que o intervalo (0,
1) e o espaço R^3 são a mesma coisa.

Ele talvez tenha a resposta para a hipótese do contínuo.

Artur Costa Steiner


Em 14 de out de 2015, às 12:37, antoni...@openmailbox.org escreveu:

Boa tarde grupo

Um amigo meu apresentou um texto de um professor que teria refutado 
Cantor.
O texto está entre as páginas 104 e 106 
(http://forum.antinovaordemmundial.com/attachment.php?aid=2523)

No texto ele diz o seguinte:

Só para dar um exemplo: O célebre Georg Cantor acreditou poder 
refutar o
5º princípio de Euclides ( de que o todo é maior que a parte ) pelo 
argumento de
que o conjunto dos números pares, embora sendo parte do conjunto dos 
números
inteiros, pode ser posto em correspondência biunívoca com ele, de 
modo
que os dois conjuntos teriam o mesmo número de elementos e, assim, a 
parte

seria igual ao todo.

Ele termina dizendo isto:

No seu “argumento”, não se trata de uma verdadeira distinção 
entre todo e
parte, mas sim de uma comparação meramente verbal entre um todo e o 
mesmo
todo, diversamente denominado. Não se tratando de um verdadeiro todo 
e de
uma verdadeira parte, não se pode falar então de uma igualdade de 
elementos
entre todo e parte, nem, portanto, de uma refutação do 5º 
princípio de Euclides.

Cantor erra o alvo por muitos metros

Existe alguma demonstração neste texto que Cantor estaria errado?

sds


Antonio G Oliveira


--
Esta mensagem foi verificada pelo sistema de antivírus e
acredita-se estar livre de perigo.

=
Instruções para entrar na lista, sair da lista e usar a lista em
http://www.mat.puc-rio.br/~obmlistas/obm-l.html
=


--
Esta mensagem foi verificada pelo sistema de antiv�rus e
 acredita-se estar livre de perigo.


=
Instru��es para entrar na lista, sair da lista e usar a lista em
http://www.mat.puc-rio.br/~obmlistas/obm-l.html
=



--
Esta mensagem foi verificada pelo sistema de antiv�rus e
acredita-se estar livre de perigo.

=
Instru��es para entrar na lista, sair da lista e usar a lista em
http://www.mat.puc-rio.br/~obmlistas/obm-l.html
=


[obm-l] RES: [obm-l] Düvida sobre o infinito de Cantor

2015-10-14 Por tôpico Vitório Batista Lima da Silva
Eita ...maldade Steiner rsrsrsrs

-Mensagem original-
De: owner-ob...@mat.puc-rio.br [mailto:owner-ob...@mat.puc-rio.br] Em nome de 
Artur Costa Steiner
Enviada em: quarta-feira, 14 de outubro de 2015 13:37
Para: obm-l@mat.puc-rio.br
Assunto: Re: [obm-l] Düvida sobre o infinito de Cantor

O  filósofo, assim ele se proclama, Olavo de Carvalho devia se abster de falar 
sobre o que não conhece. O conjunto dos inteiros e o dos pares  são conceitos 
matematicamente distintos. Não são iguais simplesmente porque nem todo inteiro 
é par.

Eles tem a mesma cardinalidade, há uma bijeção entre eles.  Uma das 
características de conjuntos infinitos é terem a mesma cardinalidade que um de 
seus subconjuntos próprios. 

Não tem nada a ver com a parte ser igual ao todo. Aliás, para dizer  isto de 
forma matematicamente correta, é preciso definir o que significa ser maior. No 
caso de conjuntos, costuma-se às vezes dizer que A é menor que B se A for 
subconjunto próprio de B. Sendo assim, o o conjunto dos pares é menor do que o 
dos inteiros.

Para deixar Olavo ainda mais louco, diga a ele que no intervalo (0, 1) há 
tantos elementos quanto em toda a reta real. E também tantos quanto em todo o 
espaço R^3... Segundo ele, isto implica que o intervalo (0, 1) e o espaço R^3 
são a mesma coisa.

Ele talvez tenha a resposta para a hipótese do contínuo. 

Artur Costa Steiner

> Em 14 de out de 2015, às 12:37, antoni...@openmailbox.org escreveu:
> 
> Boa tarde grupo
> 
> Um amigo meu apresentou um texto de um professor que teria refutado Cantor.
> O texto está entre as páginas 104 e 106 
> (http://forum.antinovaordemmundial.com/attachment.php?aid=2523)
> No texto ele diz o seguinte:
> 
> Só para dar um exemplo: O célebre Georg Cantor acreditou poder refutar o
> 5º princípio de Euclides ( de que o todo é maior que a parte ) pelo 
> argumento de
> que o conjunto dos números pares, embora sendo parte do conjunto dos números
> inteiros, pode ser posto em correspondência biunívoca com ele, de modo
> que os dois conjuntos teriam o mesmo número de elementos e, assim, a parte
> seria igual ao todo.
> 
> Ele termina dizendo isto:
> 
> No seu “argumento”, não se trata de uma verdadeira distinção entre 
> todo e
> parte, mas sim de uma comparação meramente verbal entre um todo e o mesmo
> todo, diversamente denominado. Não se tratando de um verdadeiro todo e de
> uma verdadeira parte, não se pode falar então de uma igualdade de elementos
> entre todo e parte, nem, portanto, de uma refutação do 5º princípio de 
> Euclides.
> Cantor erra o alvo por muitos metros
> 
> Existe alguma demonstração neste texto que Cantor estaria errado?
> 
> sds
> 
> 
> Antonio G Oliveira
> 
> 
> -- 
> Esta mensagem foi verificada pelo sistema de antivírus e
> acredita-se estar livre de perigo.
> 
> =
> Instruções para entrar na lista, sair da lista e usar a lista em
> http://www.mat.puc-rio.br/~obmlistas/obm-l.html
> =

-- 
Esta mensagem foi verificada pelo sistema de antiv�rus e
 acredita-se estar livre de perigo.


=
Instru��es para entrar na lista, sair da lista e usar a lista em
http://www.mat.puc-rio.br/~obmlistas/obm-l.html
=



-- 
Esta mensagem foi verificada pelo sistema de antiv�rus e
 acredita-se estar livre de perigo.


=
Instru��es para entrar na lista, sair da lista e usar a lista em
http://www.mat.puc-rio.br/~obmlistas/obm-l.html
=


Re: [obm-l] RES: [obm-l] Düvida sobre o infinito de Cantor

2015-10-14 Por tôpico Artur Costa Steiner
Correto. Sob o ponto de vista matemático, o que ele afirma, de forma arrogante, 
não faz sentido.  E nem sob o ponto de vista filosófico. Ele deveria ler um 
livro sobre teoria dos conjuntos, assunto que, antes de estat na matemática, 
está na lógica. Como o Naive Set Theory, de Halmos.

Quando se tratam de conjuntos infinitos, o conceito de mesmo número de 
elementos fica sem sentido. Mas o de mesma cardnalidade, dizendo que há uma 
bijeção entre os dois conjuntos, é perfeitamente clara. 

Por exemplo, (-pi/2 , pi/2) e a reta real tem a mesma cardinalidade. Uma 
bijeção do primeiro sobre a segunda é f(x) = tan(x). E como podemos mostrar que 
todos intervalos nāo vazios têm a mesma cardinalidade, segue-se que todos têm a 
cardinalidade de R.

Artur Costa Steiner

> Em 14 de out de 2015, às 15:08, antoni...@openmailbox.org escreveu:
> 
> Obrigado pela explicação Artur Steiner. Pelo que você falou, a 
> explicação dele não tem nem sentido, correto?
> 
> sds
> 
> 
> Antonio G Oliveira
> 
>> On 2015-10-14 14:04, Vitório Batista Lima da Silva wrote:
>> Eita ...maldade Steiner rsrsrsrs
>> -Mensagem original-
>> De: owner-ob...@mat.puc-rio.br [mailto:owner-ob...@mat.puc-rio.br] Em
>> nome de Artur Costa Steiner
>> Enviada em: quarta-feira, 14 de outubro de 2015 13:37
>> Para: obm-l@mat.puc-rio.br
>> Assunto: Re: [obm-l] Düvida sobre o infinito de Cantor
>> O  filósofo, assim ele se proclama, Olavo de Carvalho devia se abster
>> de falar sobre o que não conhece. O conjunto dos inteiros e o dos
>> pares  são conceitos matematicamente distintos. Não são iguais
>> simplesmente porque nem todo inteiro é par.
>> Eles tem a mesma cardinalidade, há uma bijeção entre eles.  Uma das
>> características de conjuntos infinitos é terem a mesma cardinalidade
>> que um de seus subconjuntos próprios.
>> Não tem nada a ver com a parte ser igual ao todo. Aliás, para dizer
>> isto de forma matematicamente correta, é preciso definir o que
>> significa ser maior. No caso de conjuntos, costuma-se às vezes dizer
>> que A é menor que B se A for subconjunto próprio de B. Sendo assim, o
>> o conjunto dos pares é menor do que o dos inteiros.
>> Para deixar Olavo ainda mais louco, diga a ele que no intervalo (0, 1)
>> há tantos elementos quanto em toda a reta real. E também tantos quanto
>> em todo o espaço R^3... Segundo ele, isto implica que o intervalo (0,
>> 1) e o espaço R^3 são a mesma coisa.
>> Ele talvez tenha a resposta para a hipótese do contínuo.
>> Artur Costa Steiner
>>> Em 14 de out de 2015, Ã s 12:37, antoni...@openmailbox.org escreveu:
>>> Boa tarde grupo
>>> Um amigo meu apresentou um texto de um professor que teria refutado Cantor.
>>> O texto está entre as páginas 104 e 106 
>>> (http://forum.antinovaordemmundial.com/attachment.php?aid=2523)
>>> No texto ele diz o seguinte:
>>> Só para dar um exemplo: O célebre Georg Cantor acreditou poder 
>>> refutar o
>>> 5º princípio de Euclides ( de que o todo é maior que a parte ) 
>>> pelo argumento de
>>> que o conjunto dos números pares, embora sendo parte do conjunto dos 
>>> números
>>> inteiros, pode ser posto em correspondência biunívoca com ele, de modo
>>> que os dois conjuntos teriam o mesmo número de elementos e, assim, a 
>>> parte
>>> seria igual ao todo.
>>> Ele termina dizendo isto:
>>> No seu “argumento”, não se trata de uma verdadeira 
>>> distinção entre todo e
>>> parte, mas sim de uma comparação meramente verbal entre um todo e o 
>>> mesmo
>>> todo, diversamente denominado. Não se tratando de um verdadeiro todo e de
>>> uma verdadeira parte, não se pode falar então de uma igualdade de 
>>> elementos
>>> entre todo e parte, nem, portanto, de uma refutação do 5º 
>>> princípio de Euclides.
>>> Cantor erra o alvo por muitos metros
>>> Existe alguma demonstração neste texto que Cantor estaria errado?
>>> sds
>>> 
>>> Antonio G Oliveira
>>> --
>>> Esta mensagem foi verificada pelo sistema de antivírus e
>>> acredita-se estar livre de perigo.
>>> =
>>> Instruções para entrar na lista, sair da lista e usar a lista em
>>> http://www.mat.puc-rio.br/~obmlistas/obm-l.html
>>> =
>> --
>> Esta mensagem foi verificada pelo sistema de antiv�rus e
>> acredita-se estar livre de perigo.
>> =
>> Instru��es para entrar na lista, sair da lista e usar a lista em
>> http://www.mat.puc-rio.br/~obmlistas/obm-l.html
>> =
> 
> 
> -- 
> Esta mensagem foi verificada pelo sistema de antivírus e
> acredita-se estar livre de perigo.
> 
> =
> Instruções para entrar na lista, sair da 

Re: [obm-l] RES: [obm-l] Düvida sobre o infinito de Cantor

2015-10-14 Por tôpico Celso Figueiredo
Isso mesmo, aliás como quase tudo que ele afirma .

ce...@uerj.br
Tels: 999712520, 986361413 e 23342176.
sala 10.043 F

> Em 14 de out de 2015, às 15:08, antoni...@openmailbox.org escreveu:
> 
> Obrigado pela explicação Artur Steiner. Pelo que você falou, a 
> explicação dele não tem nem sentido, correto?
> 
> sds
> 
> 
> Antonio G Oliveira
> 
>> On 2015-10-14 14:04, Vitório Batista Lima da Silva wrote:
>> Eita ...maldade Steiner rsrsrsrs
>> -Mensagem original-
>> De: owner-ob...@mat.puc-rio.br [mailto:owner-ob...@mat.puc-rio.br] Em
>> nome de Artur Costa Steiner
>> Enviada em: quarta-feira, 14 de outubro de 2015 13:37
>> Para: obm-l@mat.puc-rio.br
>> Assunto: Re: [obm-l] Düvida sobre o infinito de Cantor
>> O  filósofo, assim ele se proclama, Olavo de Carvalho devia se abster
>> de falar sobre o que não conhece. O conjunto dos inteiros e o dos
>> pares  são conceitos matematicamente distintos. Não são iguais
>> simplesmente porque nem todo inteiro é par.
>> Eles tem a mesma cardinalidade, há uma bijeção entre eles.  Uma das
>> características de conjuntos infinitos é terem a mesma cardinalidade
>> que um de seus subconjuntos próprios.
>> Não tem nada a ver com a parte ser igual ao todo. Aliás, para dizer
>> isto de forma matematicamente correta, é preciso definir o que
>> significa ser maior. No caso de conjuntos, costuma-se às vezes dizer
>> que A é menor que B se A for subconjunto próprio de B. Sendo assim, o
>> o conjunto dos pares é menor do que o dos inteiros.
>> Para deixar Olavo ainda mais louco, diga a ele que no intervalo (0, 1)
>> há tantos elementos quanto em toda a reta real. E também tantos quanto
>> em todo o espaço R^3... Segundo ele, isto implica que o intervalo (0,
>> 1) e o espaço R^3 são a mesma coisa.
>> Ele talvez tenha a resposta para a hipótese do contínuo.
>> Artur Costa Steiner
>>> Em 14 de out de 2015, Ã s 12:37, antoni...@openmailbox.org escreveu:
>>> Boa tarde grupo
>>> Um amigo meu apresentou um texto de um professor que teria refutado Cantor.
>>> O texto está entre as páginas 104 e 106 
>>> (http://forum.antinovaordemmundial.com/attachment.php?aid=2523)
>>> No texto ele diz o seguinte:
>>> Só para dar um exemplo: O célebre Georg Cantor acreditou poder 
>>> refutar o
>>> 5º princípio de Euclides ( de que o todo é maior que a parte ) 
>>> pelo argumento de
>>> que o conjunto dos números pares, embora sendo parte do conjunto dos 
>>> números
>>> inteiros, pode ser posto em correspondência biunívoca com ele, de modo
>>> que os dois conjuntos teriam o mesmo número de elementos e, assim, a 
>>> parte
>>> seria igual ao todo.
>>> Ele termina dizendo isto:
>>> No seu “argumento”, não se trata de uma verdadeira 
>>> distinção entre todo e
>>> parte, mas sim de uma comparação meramente verbal entre um todo e o 
>>> mesmo
>>> todo, diversamente denominado. Não se tratando de um verdadeiro todo e de
>>> uma verdadeira parte, não se pode falar então de uma igualdade de 
>>> elementos
>>> entre todo e parte, nem, portanto, de uma refutação do 5º 
>>> princípio de Euclides.
>>> Cantor erra o alvo por muitos metros
>>> Existe alguma demonstração neste texto que Cantor estaria errado?
>>> sds
>>> 
>>> Antonio G Oliveira
>>> --
>>> Esta mensagem foi verificada pelo sistema de antivírus e
>>> acredita-se estar livre de perigo.
>>> =
>>> Instruções para entrar na lista, sair da lista e usar a lista em
>>> http://www.mat.puc-rio.br/~obmlistas/obm-l.html
>>> =
>> --
>> Esta mensagem foi verificada pelo sistema de antiv�rus e
>> acredita-se estar livre de perigo.
>> =
>> Instru��es para entrar na lista, sair da lista e usar a lista em
>> http://www.mat.puc-rio.br/~obmlistas/obm-l.html
>> =
> 
> 
> -- 
> Esta mensagem foi verificada pelo sistema de antivírus e
> acredita-se estar livre de perigo.
> 
> =
> Instruções para entrar na lista, sair da lista e usar a lista em
> http://www.mat.puc-rio.br/~obmlistas/obm-l.html
> =

-- 
Esta mensagem foi verificada pelo sistema de antiv�rus e
 acredita-se estar livre de perigo.


=
Instru��es para entrar na lista, sair da lista e usar a lista em
http://www.mat.puc-rio.br/~obmlistas/obm-l.html
=


RES: [obm-l] Quadrados e cubos vizinhos?

2015-10-06 Por tôpico Albert Bouskela
Olá!

 

Podemos escrever assim: m^2 - n^3 = 1

 

Esta equação é um caso particular da Conjectura de Catalan. Esta conjectura 
afirma que a equação m^p - n^q = 1 tem uma única solução (entre os inteiros): 
3^2 - 2^3 = 1

 

A Conjectura de Catalan foi formulada em 1844 e provada, em 2002, por Preda 
Mihăilescu. Porém, parte da demonstração de Mihăilescu requer o uso de recursos 
computacionais (cálculos extensivos). Posteriormente, em 2012, Jamel Ghanouchi 
conseguiu uma demonstração puramente algébrica (analítica), através da 
generalização da Equação de Pillai: Y^p = X^q + a

 

Sds.,

Albert.

 

De: owner-ob...@mat.puc-rio.br [mailto:owner-ob...@mat.puc-rio.br] Em nome de 
marcone augusto araújo borges
Enviada em: terça-feira, 6 de outubro de 2015 17:13
Para: obm-l@mat.puc-rio.br
Assunto: [obm-l] Quadrados e cubos vizinhos?

 

0^3 + 1 = 1^2

2^3 + 1 = 3^2

Minha pergunta é:

Quais são os inteiros não negativos n e m tais que n^3 + 1 = m^2?


-- 
Esta mensagem foi verificada pelo sistema de antivírus e 
acredita-se estar livre de perigo. 


-- 
Esta mensagem foi verificada pelo sistema de antiv�rus e
 acredita-se estar livre de perigo.



[obm-l] Re: [obm-l] RES: [obm-l] Re: [obm-l] Dízima

2015-06-19 Por tôpico Luiz Claudio Valverde
De um baralho de poquer (7, 8, 9, 10, valete, dama, rei e as, cada um
desses grupos aparecendo em 4 naipes: copas, ouros, paus, espadas),
sacam-se simultaneamente 5 cartas.
a) Quantas sao as extracoes possíveis?
Quantas s~ao as extracoes nas quais se forma:
b) um par (duas cartas em um mesmo grupo e as outras tr^es em tr^es
outros grupos diferentes)?
c) dois pares (duas cartas em um grupo, duas em outro grupo e uma
em um terceiro grupo)?
d) uma trinca (tr^es cartas em um grupo e as outras duas em dois
outros grupos diferentes)?
e) um \four (quatro cartas em um grupo e uma em outro grupo)?
f) um \full hand (tr^es cartas em um grupo e duas em outro grupo)?
g) uma sequ^encia (5 cartas de grupos consecutivos, n~ao sendo todas
do mesmo naipe)?
h) um \flush (5 cartas do mesmo naipe, n~ao sendo elas de 5 grupos
consecutivos)?
i) um \straight flush (5 cartas de grupos consecutivos, todas do
mesmo naipe)?
j ) um \royal straight  flush (10, valete, dama, rei e as de um mesmo
naipe)?

Em 19 de junho de 2015 20:29, Pedro Costa npc1...@gmail.com escreveu:

 A resposta é 956, Na explicação de candre t=957 ou não entendi  a sua
 solução? ou a resposta do livro está errada?



 *De:* owner-ob...@mat.puc-rio.br [mailto:owner-ob...@mat.puc-rio.br] *Em
 nome de *Mauricio de Araujo
 *Enviada em:* sexta-feira, 19 de junho de 2015 16:14
 *Para:* obm-l@mat.puc-rio.br
 *Assunto:* [obm-l] Re: [obm-l] Dízima




 http://www.tutorbrasil.com.br/forum/matematica-olimpiadas/dizimas-periodicas-t36966.html



 Em 19 de junho de 2015 11:05, Pedro Costa npc1...@gmail.com escreveu:

 Questão do livro( problemas selecionados de matemática - Gandbi- Pág.: 20
 questão : 63). Já faz dois anos que tento resolver
 este problema e não tem sucesso. Alguém de vocês poderia me ajudar.
 (questão: 63) Seja N o número de algarismos do período da dízima [image:
 \frac{1}{3^{2005}]. O número de algarismos de
 N é igual a:

 a) 952
 b) 953
 c) 954
 d) 955
 e) 956




 --

 [image: Avast logo] http://www.avast.com/

 Este email foi escaneado pelo Avast antivírus.
 www.avast.com



 --
 Esta mensagem foi verificada pelo sistema de antivírus e
 acredita-se estar livre de perigo.





 --

 Abraços


 oɾnɐɹɐ ǝp oıɔıɹnɐɯ




 --
 Esta mensagem foi verificada pelo sistema de antiv?s e
 acredita-se estar livre de perigo.


 --
   [image: Avast logo] http://www.avast.com/

 Este email foi escaneado pelo Avast antivírus.
 www.avast.com


 --
 Esta mensagem foi verificada pelo sistema de antivírus e
 acredita-se estar livre de perigo.




-- 
Luiz Claudio Valverde

luizvalve...@globo.com
(11) 98578-6562

-- 
Esta mensagem foi verificada pelo sistema de antiv�rus e
 acredita-se estar livre de perigo.



[obm-l] RES: [obm-l] Re: [obm-l] Dízima

2015-06-19 Por tôpico Pedro Costa
A resposta é 956, Na explicação de candre t=957 ou não entendi  a sua solução? 
ou a resposta do livro está errada?



De: owner-ob...@mat.puc-rio.br [mailto:owner-ob...@mat.puc-rio.br] Em nome de 
Mauricio de Araujo
Enviada em: sexta-feira, 19 de junho de 2015 16:14
Para: obm-l@mat.puc-rio.br
Assunto: [obm-l] Re: [obm-l] Dízima



http://www.tutorbrasil.com.br/forum/matematica-olimpiadas/dizimas-periodicas-t36966.html



Em 19 de junho de 2015 11:05, Pedro Costa npc1...@gmail.com 
mailto:npc1...@gmail.com  escreveu:

Questão do livro( problemas selecionados de matemática - Gandbi- Pág.: 20 
questão : 63). Já faz dois anos que tento resolver
este problema e não tem sucesso. Alguém de vocês poderia me ajudar.
(questão: 63) Seja N o número de algarismos do período da dízima . O número de 
algarismos de
N é igual a:

a) 952
b) 953
c) 954
d) 955
e) 956





  _


 http://www.avast.com/

Este email foi escaneado pelo Avast antivírus.
www.avast.com http://www.avast.com/



--
Esta mensagem foi verificada pelo sistema de antivírus e
acredita-se estar livre de perigo.







--

Abraços


oɾnɐɹɐ ǝp oıɔıɹnɐɯ




--
Esta mensagem foi verificada pelo sistema de antiv�s e
acredita-se estar livre de perigo.



---
Este email foi escaneado pelo Avast antivírus.
http://www.avast.com

-- 
Esta mensagem foi verificada pelo sistema de antiv�rus e
 acredita-se estar livre de perigo.



RES: [obm-l] Problema Interessante de Geometria

2015-06-09 Por tôpico Albert Bouskela
Olá, Ralph,

 

O arquivo GeoGebra (“Hexagons.ggb”) foi bloqueado pelo sistema que administra 
esta Lista, em face da possibilidade de vírus (por tratar-se de um arquivo 
executável).

 

Peço, então, que envie o respectivo arquivo diretamente para o meu e-mail.

 

Prometo (como sempre…) tentar encontrar uma solução ainda mais complicada do 
que as já disponíveis na literatura e (para compensar!) válida somente para uns 
poucos casos particulares.

 

Sds.,

  _  

Albert Bouskelá

 mailto:bousk...@gmail.com bousk...@gmail.com

 

De: owner-ob...@mat.puc-rio.br [mailto:owner-ob...@mat.puc-rio.br] Em nome de 
Ralph Teixeira
Enviada em: segunda-feira, 8 de junho de 2015 21:03
Para: obm-l@mat.puc-rio.br
Assunto: [obm-l] {Filename?} Problema Interessante de Geometria

 

Ola a todos.

 

Eu e minha aluna de Mestrado Fabiola encontramos um problema bem facil de 
enunciar que esclareceria um ponto da dissertacao de mestrado dela... No 
entanto, a gente soh encontrou umas solucoes bem complicadas na literatura, e 
mesmo assim parecem ser apenas para alguns casos particulares simetricos... 
Entao coloco aqui -- quem tiver uma solucao elegante ganha um agradecimento na 
dissertacao! :) :)

 

(Eu pensei ateh em sugerir esse problema para alguma OBM, mas como ainda nao 
sei resolver e acabei mostrando a alguns alunos, vou soltar logo ele aqui.)

 

Sao dados dois poligonos convexos P1P2...Pn e Q1Q2...Qn (onde n4) contendo a 
origem O em seu interior. Sabe-se que:

-- Eles tem lados respectivamente paralelos (isto eh, PiP_{i+1} // QiQ_{i+1} 
para i=1,2,...,n, indices modulo n);

-- Triangulos com vertice em O e um lado do poligono tem areas respectivamente 
iguais (isto eh, Area(OPiP_{i+1}) = Area(OQiQ_{i+1}) para i=1,2,...n, indices 
modulo n).

Pergunta-se: os poligonos tem que ser congruentes?

 

Quem quiser brincar, vide o Geogebra anexo que ilustra o caso n=6 (fiz uma 
copia de Q longe da origem para facilitar a visualizacao -- a origem para Q 
eh O_1). Pode brincar como quiser com os Q's, e com P_1 -- os outros pontos sao 
calculados para satisfazer as condicoes acima... Mas alguem consegue fazer o 
poligono P fechar (isto eh, P1=P7) sem que ele seja congruente ao Q (mas 
mantendo ambos convexos e mantendo a origem O dentro de P?)

 

Nota: se n=4, dois paralelogramos distintos de mesma area centrados na origem 
sao contra-exemplo!

 

Abraco, Ralph.


-- 
Esta mensagem foi verificada pelo sistema de antiv�rus e
 acredita-se estar livre de perigo.



RES: [obm-l] exatamente

2015-06-05 Por tôpico Vitório Batista Lima da Silva
Isso mesmo Ralph,

Pior ...trocaram para o gabarito errado, 3/38. Concurso de Oficial – Exército – 
Magistério Matemática 2014/2015


De: owner-ob...@mat.puc-rio.br [mailto:owner-ob...@mat.puc-rio.br] Em nome de 
Ralph Teixeira
Enviada em: quinta-feira, 4 de junho de 2015 16:55
Para: obm-l@mat.puc-rio.br
Assunto: Re: [obm-l] exatamente

Para mim, voce acertou, e essa resposta estah errada.

Voce escolheu fazer a contagem como se a ordem importasse, o que eh 
perfeitamente valido. Estamos supondo que o sorteio eh justo, no sentido de que 
todas as PESSOAS tem a mesma chance de serem sorteadas (e nao todas as PATENTES,

Entao, sim, ha 21*20*19 escolhas possiveis das 3 pessoas (onde a ordem importa).

Agora, para ter exatamente um sargento, temos 3 maneiras: escolhendo SXX, XSX 
ou XXS (onde S eh um sargento, X eh um nao-sargento). Cada uma dessas 3 
hipoteses tem 6*15*14 opcoes, para um total de 6*15*14*3, exatamente como voce 
fez.

Abraco, Ralph.

Obs.: Se fizessemos tudo como se a ordem NAO importasse, seriam C(21,3) opcoes 
no total, das quais servem C(6,1)*C(15,2) (escolha um sargento, e dois nao 
sargentos). Mesma resposta.

2015-06-04 12:28 GMT-03:00 Prof. Vitório Gauss 
vitorioga...@uol.com.brmailto:vitorioga...@uol.com.br:

Colegas,

Uma turma formada por 10 tenentes, 5 cabos e 6 sargentos concorre a 3 bilhetes 
de passagens mediante sorteio, sem reposicao de seus nomes. Qual a 
probabilidade de dentre as 3 passagens sorteadas exatamente uma ser ganha por 
um sargento?

fiz assim: espaco amostral = 21*20*19 = 7980;

evento favoravel = 3*15*14*6 = 3780

P=3780/7980 = 9/19

contudo a resposta foi 3/38, o que equivale a 630/7980, ou seja fizeram 
3*15*14*1.

pensei que tinha 6 escolhas para sargento, mas como foi pedido EXATAMENTE 
deveria ter usado 1 apenas.

Certo???

Abs


Vitorio


--
Esta mensagem foi verificada pelo sistema de antivírus e
acredita-se estar livre de perigo.


--
Esta mensagem foi verificada pelo sistema de antiv��s e
acredita-se estar livre de perigo.

-- 
Esta mensagem foi verificada pelo sistema de antiv�rus e
 acredita-se estar livre de perigo.



RES: [obm-l] irracionalidade

2015-04-29 Por tôpico Albert Bouskela
Não deve ser essa a proposição, veja:

 

(sqrt(3))^3 = 3*sqrt(3) (irracional)

(sqrt(3+1))^3 = 8 (racional)

 

  _  

Albert Bouskelá

 mailto:bousk...@gmail.com bousk...@gmail.com

 

De: owner-ob...@mat.puc-rio.br [mailto:owner-ob...@mat.puc-rio.br] Em nome de 
Israel Meireles Chrisostomo
Enviada em: quarta-feira, 29 de abril de 2015 13:50
Para: obm-l@mat.puc-rio.br
Assunto: [obm-l] irracionalidade

 

Alguém sabe se é possível provar que:seja k um natural,então se r^k é 
irracional então (r+1)^k também é irracional?


-- 
Esta mensagem foi verificada pelo sistema de antiv�s e 
acredita-se estar livre de perigo. 


-- 
Esta mensagem foi verificada pelo sistema de antiv�rus e
 acredita-se estar livre de perigo.



RE: RES: [obm-l] Sumidos

2015-01-20 Por tôpico Paulo Santa Rita
Caríssimo Bouskela,
Olha eu aqui  ! Espera um pouquinho que vou voltar a participar. Aproveito a 
oportunidade para sugerir a leitura de um artigo de divulgação científica que 
aIolanda me convenceu  publicar. Prometi a ela que tentaria falar de 
MecânicaQuântica SEM USAR MATEMÁTICA.
O propósito da revista é unir o subjetivo (arte) ao objetivo (ciência) Nada 
melhorque falar da interpretação de Copenhagem, que destroi a objetividade 
clássica tacitamente admitida e introdução o (a consciência do) observador  (no 
momentoonde há o colapso da função de onda )
Será que consegui ? Só os colegas podem dizer. O link do artigo é (pag 5 ) : 
http://issuu.com/iolandabrazao/docs/revista_encenao_-_janeiro_2015
Um abraço a todos !PSR, 32001140A39


From: bousk...@gmail.com
To: obm-l@mat.puc-rio.br
Subject: RES: [obm-l] Sumidos
Date: Thu, 18 Dec 2014 10:45:45 -0200

Olá! É bom ter todos de volta! Saudações! Mas… cadê o Nehab? Cadê o Santa Rita? 
Cadê o Rogerio Ponce? Cadê tantos outros? Será que viraram Papai Noel (não sei 
qual é o plural de “Papai Noel”)? Feliz Natal! Feliz 2015! (Peço que não 
entendam 2015! como o fatorial de 2015)Albert Bouskelábousk...@gmail.com De: 
owner-ob...@mat.puc-rio.br [mailto:owner-ob...@mat.puc-rio.br] Em nome de João 
Maldonado
Enviada em: quinta-feira, 18 de dezembro de 2014 02:19
Para: obm-l@mat.puc-rio.br
Assunto: RE: [obm-l] Sumidos Haha, tbm tava com saudade das suas questões 
marcone :)

O ano tava muito corrido, não deu pra acompanhar muito aqui...
Agora que peguei férias provavelmente vou ter mais tempo

[]'s
JoãoFrom: marconeborge...@hotmail.com
To: obm-l@mat.puc-rio.br
Subject: [obm-l] Sumidos
Date: Wed, 17 Dec 2014 23:36:32 +João Maldonado e outros sumidos fizeram 
falta, 
-- 
Esta mensagem foi verificada pelo sistema de antivírus e 
acredita-se estar livre de perigo.
-- 
Esta mensagem foi verificada pelo sistema de antivírus e 
acredita-se estar livre de perigo. 
--

Esta mensagem foi verificada pelo sistema de antivírus e 

 acredita-se estar livre de perigo.   
-- 
Esta mensagem foi verificada pelo sistema de antivírus e
 acredita-se estar livre de perigo.



RES: [obm-l] Sumidos

2014-12-18 Por tôpico Albert Bouskela
Olá!

 

É bom ter todos de volta! Saudações! Mas… cadê o Nehab? Cadê o Santa Rita?
Cadê o Rogerio Ponce? Cadê tantos outros? Será que viraram Papai Noel (não
sei qual é o plural de “Papai Noel”)?

 

Feliz Natal! Feliz 2015! (Peço que não entendam 2015! como o fatorial de
2015)

  _  

Albert Bouskelá

 mailto:bousk...@gmail.com bousk...@gmail.com

 

De: owner-ob...@mat.puc-rio.br [mailto:owner-ob...@mat.puc-rio.br] Em nome
de João Maldonado
Enviada em: quinta-feira, 18 de dezembro de 2014 02:19
Para: obm-l@mat.puc-rio.br
Assunto: RE: [obm-l] Sumidos

 

Haha, tbm tava com saudade das suas questões marcone :)

O ano tava muito corrido, não deu pra acompanhar muito aqui...
Agora que peguei férias provavelmente vou ter mais tempo

[]'s
João

  _  

From: marconeborge...@hotmail.com mailto:marconeborge...@hotmail.com 
To: obm-l@mat.puc-rio.br mailto:obm-l@mat.puc-rio.br 
Subject: [obm-l] Sumidos
Date: Wed, 17 Dec 2014 23:36:32 +

João Maldonado e outros sumidos fizeram falta, 


-- 
Esta mensagem foi verificada pelo sistema de antivírus e 
acredita-se estar livre de perigo.


-- 
Esta mensagem foi verificada pelo sistema de antivírus e 
acredita-se estar livre de perigo. 


-- 
Esta mensagem foi verificada pelo sistema de antivírus e
 acredita-se estar livre de perigo.



[obm-l] Re: [obm-l] RES: [obm-l] Re: [obm-l] Re: [obm-l] RES: [obm-l] Problema da Olimpíada de Matemática de Moscou

2014-09-05 Por tôpico Mauricio de Araujo
é o antigo noves fora que minha mãe usava...


2014-09-04 21:25 GMT-03:00 Albert Bouskela bousk...@ymail.com:

 Olá!



 Pois é! Problemas (equações) que envolvem um determinado número (natural)
 e a soma dos algarismos que o compõem, geralmente, são resolvidos através
 da propriedade mencionada pelo Ralph:



 S(x) = x (mod. 9)



 Ou, o que dá no mesmo, mas as vezes pode ser mais útil:



 “x” e S(x) deixam o mesmo resto na divisão por 9.



 Exemplo:



 Mostre que a soma da soma da soma (3 vezes) dos algarismos de 50^50 e
 770^770 são iguais.



 

 Albert Bouskelá

 bousk...@ymail.com



 *De:* owner-ob...@mat.puc-rio.br [mailto:owner-ob...@mat.puc-rio.br] *Em
 nome de *Ralph Teixeira
 *Enviada em:* quarta-feira, 3 de setembro de 2014 21:27
 *Para:* obm-l@mat.puc-rio.br
 *Assunto:* [obm-l] Re: [obm-l] Re: [obm-l] RES: [obm-l] Problema da
 Olimpíada de Matemática de Moscou



 Ah, eh verdade, dah para acelerar MUITO notando que:



 S(x) = x (mod 9)



 Entao x+S(x)+S(S(x)) = 3x (mod 9)



 Isto eh, x+S(x)+S(S(x)) eh sempre divisivel por 3 -- e portanto nunca pode
 ser 1993.



 Abraco,

  Ralph



 2014-09-03 19:42 GMT-03:00 Mauricio de Araujo 
 mauricio.de.ara...@gmail.com:

 não tem solução!! hehehe



 2014-09-03 19:07 GMT-03:00 Albert Bouskela bousk...@ymail.com:

 Olá!



 A melhor solução é pelo “cheiro”



 1) x1899 | 1899+S(1899)+SS(1899)=19351993

 2) x1959 | 1959+S(1959)+SS(1959)=19891993

 3) S≥16 (x=1960) e SS≥2 (S=20)

 4) x=1993-16-2=1975

 5) 1960=x=1975

 6) Agora é no braço…

 7) Mas há uma surpresa no final!



 

 Albert Bouskelá

 bousk...@ymail.com

 *De:* owner-ob...@mat.puc-rio.br [mailto:owner-ob...@mat.puc-rio.br] *Em
 nome de *Mauricio de Araujo
 *Enviada em:* quarta-feira, 3 de setembro de 2014 11:36
 *Para:* obm-l@mat.puc-rio.br
 *Assunto:* [obm-l] Problema da Olimpíada de Matemática de Moscou



 Seja S(x) a soma dos dígitos de um inteiro positivo x.



 Resolver: x + S(x) + S(S(x)) = 1993.



 --

 Abraços


 oɾnɐɹɐ ǝp oıɔıɹnɐɯ

 --
 Esta mensagem foi verificada pelo sistema de antivírus e
 acredita-se estar livre de perigo.




-- 
Abraços

oɾnɐɹɐ ǝp oıɔıɹnɐɯ

-- 
Esta mensagem foi verificada pelo sistema de antiv�rus e
 acredita-se estar livre de perigo.



[obm-l] Re: [obm-l] RES: [obm-l] Re: [obm-l] Re: [obm-l] RES: [obm-l] Problema da Olimpíada de Matemática de Moscou

2014-09-05 Por tôpico Douglas Oliveira de Lima
Existe uma questão muito legal que acabei de fazer desse mesmo assunto,
caso seja do seu interesse praticar ai vai.
A soma dos algarismos de um numero n vale 100, e a soma dos digitos do
numero 44n vale 800, Calcular a soma dos digitos de 3n.
Douglas Oliveira


Em 4 de setembro de 2014 21:25, Albert Bouskela bousk...@ymail.com
escreveu:

 Olá!



 Pois é! Problemas (equações) que envolvem um determinado número (natural)
 e a soma dos algarismos que o compõem, geralmente, são resolvidos através
 da propriedade mencionada pelo Ralph:



 S(x) = x (mod. 9)



 Ou, o que dá no mesmo, mas as vezes pode ser mais útil:



 “x” e S(x) deixam o mesmo resto na divisão por 9.



 Exemplo:



 Mostre que a soma da soma da soma (3 vezes) dos algarismos de 50^50 e
 770^770 são iguais.



 

 Albert Bouskelá

 bousk...@ymail.com



 *De:* owner-ob...@mat.puc-rio.br [mailto:owner-ob...@mat.puc-rio.br] *Em
 nome de *Ralph Teixeira
 *Enviada em:* quarta-feira, 3 de setembro de 2014 21:27
 *Para:* obm-l@mat.puc-rio.br
 *Assunto:* [obm-l] Re: [obm-l] Re: [obm-l] RES: [obm-l] Problema da
 Olimpíada de Matemática de Moscou



 Ah, eh verdade, dah para acelerar MUITO notando que:



 S(x) = x (mod 9)



 Entao x+S(x)+S(S(x)) = 3x (mod 9)



 Isto eh, x+S(x)+S(S(x)) eh sempre divisivel por 3 -- e portanto nunca pode
 ser 1993.



 Abraco,

  Ralph



 2014-09-03 19:42 GMT-03:00 Mauricio de Araujo 
 mauricio.de.ara...@gmail.com:

 não tem solução!! hehehe



 2014-09-03 19:07 GMT-03:00 Albert Bouskela bousk...@ymail.com:

 Olá!



 A melhor solução é pelo “cheiro”



 1) x1899 | 1899+S(1899)+SS(1899)=19351993

 2) x1959 | 1959+S(1959)+SS(1959)=19891993

 3) S≥16 (x=1960) e SS≥2 (S=20)

 4) x=1993-16-2=1975

 5) 1960=x=1975

 6) Agora é no braço…

 7) Mas há uma surpresa no final!



 

 Albert Bouskelá

 bousk...@ymail.com

 *De:* owner-ob...@mat.puc-rio.br [mailto:owner-ob...@mat.puc-rio.br] *Em
 nome de *Mauricio de Araujo
 *Enviada em:* quarta-feira, 3 de setembro de 2014 11:36
 *Para:* obm-l@mat.puc-rio.br
 *Assunto:* [obm-l] Problema da Olimpíada de Matemática de Moscou



 Seja S(x) a soma dos dígitos de um inteiro positivo x.



 Resolver: x + S(x) + S(S(x)) = 1993.



 --

 Abraços


 oɾnɐɹɐ ǝp oıɔıɹnɐɯ

 --
 Esta mensagem foi verificada pelo sistema de antivírus e
 acredita-se estar livre de perigo.


-- 
Esta mensagem foi verificada pelo sistema de antiv�rus e
 acredita-se estar livre de perigo.



[obm-l] Re: [obm-l] RES: [obm-l] Re: [obm-l] Re: [obm-l] RES: [obm-l] Problema da Olimpíada de Matemática de Moscou

2014-09-05 Por tôpico Pedro José
Boa tarde!

O número de algarismos x não nulos de 50^50 é igual ao número de
algarismos de 5^50

seja t = [x], t Ɛ e  x - 1  t  = x

x = [50*log(5)]+1 = 35 == S (50^50) = 9*35 = 315 == S(S(50^50)) = 2+9
+9 = 13 == S(S(S50^50) 10 ==

== S(S(50^50)) só tem um algarismo

S(S(S(50^50))) ≡ 3* (50^50) mod 9

50^50 ≡ 5^50 * 10^50 ≡  5^50 mod 9 pois, 10 ≡ 1 mod9

5^50 ≡ (5^6)^8 * 5^2 ≡ 7 mod 9 (pois, 5^6 ≡  1 mod 9)  == S(S(S(50^50)))
≡  3* 7 ≡ 3 mod9. Como  0= S(S(S(50^50))) 10 ==

==  S(S(S(50^50))) = 3 (i)

O número de algarismos y  não nulos de 770^770 é gual ao número de
algarismos de 77^770

y =  [770* log (77)] + 1 == y = 1453 == S(770^770) = 13077 (9*1453)  ==
S(S(770^770)) = 30 (1 + 2 + 9 + 9 + 9) ==

S(S(S(770^770))) = 11 (2+9)

Se S(S(S(770^770))) possui dois algarismos 770^770 ≡ 1 mod 9 ou 770^770 ≡ 2
mod 9

770^770 ≡ 77^770 * 10^770 ≡  77^770 mod 9 pois 10 ≡ 1 mod9

77^770 ≡ (77^6)^128 * 7^2 ≡ 4 mod9  (pois, 5^6 ≡  1 mod 9) ==
S(S(S(770^770))) só possui um algarismo == 0= S(S(S(770^770))) 10

== S(S(S(770^770))) 3* 4 ≡ 3 mod9. Como  0= S(S(S(770^770)))10 ==
S(S(S(770^770))) = 3 (ii)

(i) e (ii) == S(S(S(50^50))) = S(S(S(770^770)))

Saudações,
PJMS.



Em 4 de setembro de 2014 21:25, Albert Bouskela bousk...@ymail.com
escreveu:

 Olá!



 Pois é! Problemas (equações) que envolvem um determinado número (natural)
 e a soma dos algarismos que o compõem, geralmente, são resolvidos através
 da propriedade mencionada pelo Ralph:



 S(x) = x (mod. 9)



 Ou, o que dá no mesmo, mas as vezes pode ser mais útil:



 “x” e S(x) deixam o mesmo resto na divisão por 9.



 Exemplo:



 Mostre que a soma da soma da soma (3 vezes) dos algarismos de 50^50 e
 770^770 são iguais.



 

 Albert Bouskelá

 bousk...@ymail.com



 *De:* owner-ob...@mat.puc-rio.br [mailto:owner-ob...@mat.puc-rio.br] *Em
 nome de *Ralph Teixeira
 *Enviada em:* quarta-feira, 3 de setembro de 2014 21:27
 *Para:* obm-l@mat.puc-rio.br
 *Assunto:* [obm-l] Re: [obm-l] Re: [obm-l] RES: [obm-l] Problema da
 Olimpíada de Matemática de Moscou



 Ah, eh verdade, dah para acelerar MUITO notando que:



 S(x) = x (mod 9)



 Entao x+S(x)+S(S(x)) = 3x (mod 9)



 Isto eh, x+S(x)+S(S(x)) eh sempre divisivel por 3 -- e portanto nunca pode
 ser 1993.



 Abraco,

  Ralph



 2014-09-03 19:42 GMT-03:00 Mauricio de Araujo 
 mauricio.de.ara...@gmail.com:

 não tem solução!! hehehe



 2014-09-03 19:07 GMT-03:00 Albert Bouskela bousk...@ymail.com:

 Olá!



 A melhor solução é pelo “cheiro”



 1) x1899 | 1899+S(1899)+SS(1899)=19351993

 2) x1959 | 1959+S(1959)+SS(1959)=19891993

 3) S≥16 (x=1960) e SS≥2 (S=20)

 4) x=1993-16-2=1975

 5) 1960=x=1975

 6) Agora é no braço…

 7) Mas há uma surpresa no final!



 

 Albert Bouskelá

 bousk...@ymail.com

 *De:* owner-ob...@mat.puc-rio.br [mailto:owner-ob...@mat.puc-rio.br] *Em
 nome de *Mauricio de Araujo
 *Enviada em:* quarta-feira, 3 de setembro de 2014 11:36
 *Para:* obm-l@mat.puc-rio.br
 *Assunto:* [obm-l] Problema da Olimpíada de Matemática de Moscou



 Seja S(x) a soma dos dígitos de um inteiro positivo x.



 Resolver: x + S(x) + S(S(x)) = 1993.



 --

 Abraços


 oɾnɐɹɐ ǝp oıɔıɹnɐɯ

 --
 Esta mensagem foi verificada pelo sistema de antivírus e
 acredita-se estar livre de perigo.


-- 
Esta mensagem foi verificada pelo sistema de antiv�rus e
 acredita-se estar livre de perigo.



[obm-l] RES: [obm-l] Problema da Olimpiada de Matemática de Moscou

2014-09-03 Por tôpico Albert Bouskela
Olá!

 

A melhor solução é pelo “cheiro”

 

1) x1899 | 1899+S(1899)+SS(1899)=19351993

2) x1959 | 1959+S(1959)+SS(1959)=19891993

3) S≥16 (x=1960) e SS≥2 (S=20)

4) x≤1993-16-2=1975

5) 1960≤x≤1975

6) Agora é no braço…

7) Mas há uma surpresa no final!

 

  _  

Albert Bouskelá

 mailto:bousk...@ymail.com bousk...@ymail.com

 

De: owner-ob...@mat.puc-rio.br [mailto:owner-ob...@mat.puc-rio.br] Em nome de 
Mauricio de Araujo
Enviada em: quarta-feira, 3 de setembro de 2014 11:36
Para: obm-l@mat.puc-rio.br
Assunto: [obm-l] Problema da Olimpiada de Matemática de Moscou

 

Seja S(x) a soma dos dígitos de um inteiro positivo x.

 

Resolver: x + S(x) + S(S(x)) = 1993.

 

-- 

Abraços


oɾnɐɹɐ ǝp oıɔıɹnɐɯ

 


-- 
Esta mensagem foi verificada pelo sistema de antiv�s e 
acredita-se estar livre de perigo. 


-- 
Esta mensagem foi verificada pelo sistema de antiv�rus e
 acredita-se estar livre de perigo.



[obm-l] Re: [obm-l] RES: [obm-l] Problema da Olimpiada de Matemática de Moscou

2014-09-03 Por tôpico Mauricio de Araujo
não tem solução!! hehehe


2014-09-03 19:07 GMT-03:00 Albert Bouskela bousk...@ymail.com:

 Olá!



 A melhor solução é pelo “cheiro”



 1) x1899 | 1899+S(1899)+SS(1899)=19351993

 2) x1959 | 1959+S(1959)+SS(1959)=19891993

 3) S≥16 (x=1960) e SS≥2 (S=20)

 4) x≤1993-16-2=1975

 5) 1960≤x≤1975

 6) Agora é no braço…

 7) Mas há uma surpresa no final!


 --

 *Albert Bouskelá*

 bousk...@ymail.com



 *De:* owner-ob...@mat.puc-rio.br [mailto:owner-ob...@mat.puc-rio.br] *Em
 nome de *Mauricio de Araujo
 *Enviada em:* quarta-feira, 3 de setembro de 2014 11:36
 *Para:* obm-l@mat.puc-rio.br
 *Assunto:* [obm-l] Problema da Olimpiada de Matemática de Moscou



 Seja S(x) a soma dos dígitos de um inteiro positivo x.



 Resolver: x + S(x) + S(S(x)) = 1993.



 --

 Abraços


 oɾnɐɹɐ ǝp oıɔıɹnɐɯ




 --
 Esta mensagem foi verificada pelo sistema de antiv?s e
 acredita-se estar livre de perigo.


 --
 Esta mensagem foi verificada pelo sistema de antivírus e
 acredita-se estar livre de perigo.




-- 
Abraços

oɾnɐɹɐ ǝp oıɔıɹnɐɯ

-- 
Esta mensagem foi verificada pelo sistema de antiv�rus e
 acredita-se estar livre de perigo.



[obm-l] Re: [obm-l] Re: [obm-l] RES: [obm-l] Problema da Olimpiada de Matemática de Moscou

2014-09-03 Por tôpico Ralph Teixeira
Ah, eh verdade, dah para acelerar MUITO notando que:

S(x) = x (mod 9)

Entao x+S(x)+S(S(x)) = 3x (mod 9)

Isto eh, x+S(x)+S(S(x)) eh sempre divisivel por 3 -- e portanto nunca pode
ser 1993.

Abraco,
 Ralph


2014-09-03 19:42 GMT-03:00 Mauricio de Araujo mauricio.de.ara...@gmail.com
:

 não tem solução!! hehehe


 2014-09-03 19:07 GMT-03:00 Albert Bouskela bousk...@ymail.com:

 Olá!



 A melhor solução é pelo “cheiro”



 1) x1899 | 1899+S(1899)+SS(1899)=19351993

 2) x1959 | 1959+S(1959)+SS(1959)=19891993

 3) S≥16 (x=1960) e SS≥2 (S=20)

 4) x≤1993-16-2=1975

 5) 1960≤x≤1975

 6) Agora é no braço…

 7) Mas há uma surpresa no final!


 --

 *Albert Bouskelá*

 bousk...@ymail.com



 *De:* owner-ob...@mat.puc-rio.br [mailto:owner-ob...@mat.puc-rio.br] *Em
 nome de *Mauricio de Araujo
 *Enviada em:* quarta-feira, 3 de setembro de 2014 11:36
 *Para:* obm-l@mat.puc-rio.br
 *Assunto:* [obm-l] Problema da Olimpiada de Matemática de Moscou



 Seja S(x) a soma dos dígitos de um inteiro positivo x.



 Resolver: x + S(x) + S(S(x)) = 1993.



 --

 Abraços


 oɾnɐɹɐ ǝp oıɔıɹnɐɯ




 --
 Esta mensagem foi verificada pelo sistema de antiv?s e
 acredita-se estar livre de perigo.


 --
 Esta mensagem foi verificada pelo sistema de antivírus e
 acredita-se estar livre de perigo.




 --
 Abraços

 oɾnɐɹɐ ǝp oıɔıɹnɐɯ


 --
 Esta mensagem foi verificada pelo sistema de antivírus e
 acredita-se estar livre de perigo.

-- 
Esta mensagem foi verificada pelo sistema de antiv�rus e
 acredita-se estar livre de perigo.



RES: [obm-l] Arquivo

2014-08-18 Por tôpico Albert Bouskela
Pô!!! Foi justamente nesse arquivo (o que foi desabilitado) que eu postei a 
demonstração completa da Conjectura de Goldbach! Agora só resta o choro e o 
ranger de dentes…

 

  _  

Albert Bouskelá

 mailto:bousk...@ymail.com bousk...@ymail.com

 

De: owner-ob...@mat.puc-rio.br [mailto:owner-ob...@mat.puc-rio.br] Em nome de 
Vanderlei Nemitz
Enviada em: segunda-feira, 18 de agosto de 2014 09:03
Para: OBM
Assunto: Re: [obm-l] Arquivo

 

Obrigado!

Essa era minha dúvida. Só está funcionando o arquivo com postagens mais 
recentes. As bem antigas estavam no outro, que pelo visto foi desabilitado.

 

Em 18 de agosto de 2014 08:55, Hermann ilhadepaqu...@bol.com.br escreveu:

funcionando!

http://www.mail-archive.com/obm-l@mat.puc-rio.br/

 

não funcionando

http://www.mat.puc-rio.br/~nicolau/olimp/obm-l.arquivo.html

 

 

- Original Message - 

From: Vanderlei Nemitz mailto:vanderma...@gmail.com  

To: OBM mailto:obm-l@mat.puc-rio.br  

Sent: Sunday, August 17, 2014 9:57 PM

Subject: [obm-l] Arquivo

 

Boa noite, alguém sabe dizer se o arquivo com mensagens antigas foi desativado? 
Tentei acessar, mas não carrega. 

Obrigado,

Vanderlei

 

-- 
Esta mensagem foi verificada pelo sistema de antiv�rus e 
acredita-se estar livre de perigo. 


-- 
Esta mensagem foi verificada pelo sistema de antivírus e 
acredita-se estar livre de perigo. 

 


-- 
Esta mensagem foi verificada pelo sistema de antiv�s e 
acredita-se estar livre de perigo. 


-- 
Esta mensagem foi verificada pelo sistema de antiv�rus e
 acredita-se estar livre de perigo.



RES: [obm-l] Primos entre si

2014-08-09 Por tôpico benedito
Experimente  b = a+1



De: owner-ob...@mat.puc-rio.br [mailto:owner-ob...@mat.puc-rio.br] Em nome
de marcone augusto araújo borges
Enviada em: sexta-feira, 8 de agosto de 2014 19:58
Para: obm-l@mat.puc-rio.br
Assunto: [obm-l] Primos entre si



Mostre que existem infinitos n tais que a + n e b + n são primos entre si




--
Esta mensagem foi verificada pelo sistema de antivírus e
acredita-se estar livre de perigo.



---
Este email está limpo de vírus e malwares porque a proteção do avast! Antivírus 
está ativa.
http://www.avast.com

-- 
Esta mensagem foi verificada pelo sistema de antivírus e
 acredita-se estar livre de perigo.



RES: [obm-l] Quantas partidas?

2014-05-05 Por tôpico Manoel Cesar Valente Lopes
21
O primeiro jogou 5 com o segundo
O primeiro jogou 5 com o terceiro
O segundo jogou 16 com o terceiro.



From: marconeborge...@hotmail.commailto:marconeborge...@hotmail.com
To: obm-l@mat.puc-rio.brmailto:obm-l@mat.puc-rio.br
Subject: [obm-l] Quantas partidas?
Date: Sun, 20 Apr 2014 22:37:14 +
Três jogadores estão jogando ping-pong e o que não está jogando em uma 
determinada
partida joga com o vencedor na próxima partida.No final do dia, o primeiro 
jogador tinha
jogado 10 partidas e o segundo,21.Quantas partidas jogou o terceiro jogador?

--
Esta mensagem foi verificada pelo sistema de antivírus e
acredita-se estar livre de perigo.

--
Esta mensagem foi verificada pelo sistema de antivírus e
acredita-se estar livre de perigo.

-- 
Esta mensagem foi verificada pelo sistema de antivírus e
 acredita-se estar livre de perigo.



RES: [obm-l] Problema do Cavalo

2014-02-19 Por tôpico Benedito
OK Bernado.
Vou dar uma olhada.
Obrigado.
Benedito

-Mensagem original-
De: owner-ob...@mat.puc-rio.br [mailto:owner-ob...@mat.puc-rio.br] Em nome
de Bernardo Freitas Paulo da Costa
Enviada em: terça-feira, 18 de fevereiro de 2014 18:00
Para: Lista de E-mails da OBM
Assunto: Re: [obm-l] Problema do Cavalo

2014-02-18 14:30 GMT-03:00 Benedito bened...@ufrnet.br:

 É infinito nos quatro quadrantes, que é para permitir muitos movimentos.

 De: owner-ob...@mat.puc-rio.br [mailto:owner-ob...@mat.puc-rio.br] Em
 nome de terence thirteen Enviada em: segunda-feira, 17 de fevereiro de
 2014 08:16
 Para: obm-l
 Assunto: Re: [obm-l] Problema do Cavalo

 Ele é infinito nos quatro quadrantes?

 Eu tentaria algo como construir um grafo infinito, mas vou pensar antes...

Eu tenho uma idéia de solução no braço. Supondo que a questão seja:
Qual é o número de casas diferentes em que um cavalo pode terminar uma
seqüência de N movimentos. Assim, para n = 1, temos 8 casas (brancas), e
para n = 2 temos 33 casas (pretas, incluindo a casa preta original!).

Para n maior, a seqüência fica assim (feito num computador, na marra):

8; 33; 76; 129; 196; 277; 372; 481; 604; 741; 892; 1057; 1236; 1429; ...

Agora, vem o chute principal (que é o que vai ajudar a gente a fazer
indução): Calcule as diferenças sucessivas dos elementos! Isso dá:

25; 43; 53; 67; 81; 95; 109; 123; 137; 151; 165; 179; 193; ...

Ainda não parece bom ? Não tem problema... Mais uma vez, faça as diferenças:

18; 10; 14; 14; 14; 14; 14; 14; 14; 14; 14; 14; ...

Ah ! Parece que é uma PA de segunda ordem, a partir de um certo
ponto...

Vamos entender essa idéia. No longo prazo, o cavalo vai se afastando do
centro, e portanto ele pode cobrir uma área no máximo proporcional a N^2.
Isso por si só já justifica tentar achar uma PA de segunda ordem. O que é
interessante é que a parte perto do centro (depois do início, onde ainda há
um monte de buracos meio aleatórios) estará completamente coberta depois de
um certo tempo, e o que interessa é o que acontece nas coroas. Agora, tem
que justificar que as coroas têm uma espessura constante depois de passada a
parte transiente
inicial.

Como eu usei um computador, e posso calcular mais do que n = 10 (por exemplo
n = 100) e os 14 continuam até esse ponto. Para mim, isso é mais do que
suficiente para eu ter certeza que a resposta é essa, mas admito que falta
um argumento garantindo que basta observar um número finito de passos para
acertar a recorrência. Eu diria que, como um cavalo completa a vizinhança
do ponto inicial (o 3x3 em volta da
origem) em uma quantidade finita de passos (basta chegar na profundidade 3
do grafo do Torres) a recorrência não pode ser de ordem muito maior do que
isso. Para melhorar, veja que a partir de 3 passos, o que temos é um
octógono, TODO preenchido, dos quadrados brancos (que são os únicos em que o
cavalo pode estar!). Daí pra frente, não é difícil ver que a cada etapa
teremos um octógono com lado aumentando de 1 a cada vez. Veja também que a
partir do 3o termo da segunda diferença, só tem 14. Não é coincidência.

Agora, eu deixo a indução para você completar!

Abraços,
--
Bernardo Freitas Paulo da Costa

--
Esta mensagem foi verificada pelo sistema de antivírus e  acredita-se estar
livre de perigo.


=
Instruções para entrar na lista, sair da lista e usar a lista em
http://www.mat.puc-rio.br/~obmlistas/obm-l.html
=


---
Este email está limpo de vírus e malwares porque a proteção do avast! Antivírus 
está ativa.
http://www.avast.com


-- 
Esta mensagem foi verificada pelo sistema de antivírus e
 acredita-se estar livre de perigo.


=
Instruções para entrar na lista, sair da lista e usar a lista em
http://www.mat.puc-rio.br/~obmlistas/obm-l.html
=


RES: [obm-l] Problema do Cavalo

2014-02-18 Por tôpico Benedito
É infinito nos quatro quadrantes, que é para permitir muitos movimentos.



De: owner-ob...@mat.puc-rio.br [mailto:owner-ob...@mat.puc-rio.br] Em nome de 
terence thirteen
Enviada em: segunda-feira, 17 de fevereiro de 2014 08:16
Para: obm-l
Assunto: Re: [obm-l] Problema do Cavalo



Ele é infinito nos quatro quadrantes?

Eu tentaria algo como construir um grafo infinito, mas vou pensar antes...





Em 10 de fevereiro de 2014 09:11, Benedito bened...@ufrnet.br 
mailto:bened...@ufrnet.br  escreveu:

Estou tentando uma solução para o problema seguinte, usando Indução. Alguém 
pode me ajudar?

Problema

Num tabuleiro infinito, um cavalo (peça do jogo de xadrez) está situado na 
origem, digamos numa casa preta, e começa a se movimentar.

No total, quantas casas possíveis o cavalo pode atingir depois de n movimentos?

Nota - O movimento de um cavalo no jogo de xadrez é em forma de L (formado por 
4 casas, a partir da casa em que se encontra)



  _


 http://www.avast.com/

Este email está limpo de vírus e malwares porque a proteção do avast! Antivírus 
http://www.avast.com/  está ativa.



--
Esta mensagem foi verificada pelo sistema de antivírus e
acredita-se estar livre de perigo.




--
/**/
神が祝福

Torres


--
Esta mensagem foi verificada pelo sistema de antiv�s e
acredita-se estar livre de perigo.



---
Este email está limpo de vírus e malwares porque a proteção do avast! Antivírus 
está ativa.
http://www.avast.com

-- 
Esta mensagem foi verificada pelo sistema de antiv�rus e
 acredita-se estar livre de perigo.



RES: [obm-l] Teorema da Incompletude de Godel

2014-02-03 Por tôpico Albert Bouskela
Olá a todos!

 

Bem, vou sugerir dois livros sobre o assunto (o primeiro é mais do que clássico 
‒ é a bíblia do tema!):

 

1) O Teorema de Gödel e a Hipótese do Contínuo ‒ Antologia organizada, 
prefaciada e traduzida por Manuel Lourenço | Fundação Calouste Gulbenkian, 
Lisboa, Fevereiro de 1979

 

É um calhamaço de 1000 páginas, muito raro (eu tenho um exemplar!), mais do que 
árido, mas plenamente exaustivo, pelo menos até a data da sua publicação. É uma 
leitura só para os “iniciados”!

 

2) Incompletude (A prova e o paradoxo de Kurt Gödel) ‒ Rebecca Goldstein | 
Companhia das Letras, 2008.

 

Este é bem digerível (250 páginas) e é um bom começo.

 

  _  

Albert Bouskelá

 

De: owner-ob...@mat.puc-rio.br [mailto:owner-ob...@mat.puc-rio.br] Em nome de 
Carlos Nehab
Enviada em: domingo, 2 de fevereiro de 2014 07:24
Para: obm-l@mat.puc-rio.br
Assunto: Re: [obm-l] Teorema da Incompletude de Godel

 

Oi, Luiz

Mande sua análise.
Você é da patota da Ciência da Computação, Matemática ou Filosofia?

Abs
Nehab

On 01/02/2014 23:46, luiz silva wrote:

É que eu estou querendo saber se tenho como formalizar uma análise que fiz.

 

Abs

Felipe

 

Em Sábado, 1 de Fevereiro de 2014 23:21, Francisco Barreto 
costadutrabarr...@gmail.com escreveu:

Não sou especialista, perdoe-me. Meu nome é Francisco Costa Barreto e este 
assunto me interessa. Estou acompanhando, quem sabe não torno-me útil em alguns 
dias para ajudá-lo neste aspecto, em tempo. 

Eu costumo dizer incompleteza, mas podem me acusar de ser um Stickler neste 
caso.

=)

Abraços,

Francisco.

 

2014-02-01 luiz silva luizfelipec...@yahoo.com.br:

Pessoal,

 

Alguem aqui é especialista em logica-matematica, e conhece bem o teorema de 
godel?

 

Abs

Felipe


-- 
Esta mensagem foi verificada pelo sistema de antivírus e 
acredita-se estar livre de perigo. 


-- 
Esta mensagem foi verificada pelo sistema de antiv�rus e
 acredita-se estar livre de perigo.



Re: RES: [obm-l] Teorema da Incompletude de Godel

2014-02-03 Por tôpico luiz silva
Pessoal,

Jé enviei duas vezes a análise para a lista, e os emails não chegaram(não estão 
nos arquivos da lista). 

Prezado Carlos, dada  a dificuldade, enviei diretamente para seu email pessoal. 
Vc poderia confirmar o recebimento ?

Desde já, agradeço.

Abs
Felipe





Em Segunda-feira, 3 de Fevereiro de 2014 11:03, Albert Bouskela 
bousk...@ymail.com escreveu:
 
Olá a todos!
 
Bem, vou sugerir dois livros sobre o assunto (o primeiro é mais do que clássico 
‒ é a bíblia do tema!):
 
1) O Teorema de Gödel e a Hipótese do Contínuo ‒ Antologia organizada, 
prefaciada e traduzida por Manuel Lourenço | Fundação Calouste Gulbenkian, 
Lisboa, Fevereiro de 1979
 
É um calhamaço de 1000 páginas, muito raro (eu tenho um exemplar!), mais do que 
árido, mas plenamente exaustivo, pelo menos até a data da sua publicação. É uma 
leitura só para os “iniciados”!
 
2) Incompletude (A prova e o paradoxo de Kurt Gödel) ‒ Rebecca Goldstein | 
Companhia das Letras, 2008.
 
Este é bem digerível (250 páginas) e é um bom começo.
 



Albert Bouskelá
 
De:owner-ob...@mat.puc-rio.br [mailto:owner-ob...@mat.puc-rio.br] Em nome de 
Carlos Nehab
Enviada em: domingo, 2 de fevereiro de 2014 07:24
Para: obm-l@mat.puc-rio.br
Assunto: Re: [obm-l] Teorema da Incompletude de Godel
 
Oi, Luiz

Mande sua análise.
Você é da patota da Ciência da Computação, Matemática ou Filosofia?

Abs
Nehab

On 01/02/2014 23:46, luiz silva wrote:
É que eu estou querendo saber se tenho como formalizar uma análise que fiz.
 
Abs
Felipe
 
Em Sábado, 1 de Fevereiro de 2014 23:21, Francisco Barreto 
costadutrabarr...@gmail.com escreveu:
Não sou especialista, perdoe-me. Meu nome é Francisco Costa Barreto e este 
assunto me interessa. Estou acompanhando, quem sabe não torno-me útil em 
alguns dias para ajudá-lo neste aspecto, em tempo. 
Eu costumo dizer incompleteza, mas podem me acusar de ser um Stickler neste 
caso.
=)
Abraços,
Francisco.
 
2014-02-01 luiz silva luizfelipec...@yahoo.com.br:
Pessoal,
 
Alguem aqui é especialista em logica-matematica, e conhece bem o teorema de 
godel?
 
Abs
Felipe

-- 
Esta mensagem foi verificada pelo sistema de antivírus e 
acredita-se estar livre de perigo. 
-- 
Esta mensagem foi verificada pelo sistema de antiv�us e 
acredita-se estar livre de perigo. 
-- 
Esta mensagem foi verificada pelo sistema de antiv�rus e
 acredita-se estar livre de perigo.



[obm-l] RES: [obm-l] Re: [obm-l] Combinatória 2014

2014-01-22 Por tôpico Antonio Paschoal
Muitíssimo obrigado pelas referências.

O problema é bastante difícil!

Antonio Paschoal.



  _

De: owner-ob...@mat.puc-rio.br [mailto:owner-ob...@mat.puc-rio.br] Em nome
de Kelvin Anjos
Enviada em: terça-feira, 21 de janeiro de 2014 23:21
Para: obm-l@mat.puc-rio.br
Assunto: [obm-l] Re: [obm-l] Combinatória 2014



Problema de desarranjo, conhecido como Non-sexist solution of the ménage
problem.
Sem o principal empecilho de que casais não podem estar sentados em cadeiras
adjacentes, teríamos a forma permutativa de 2(n!)^2. Mas com as condições
expostas temos um caso de desarranjo.

A solução do problema passo a passo é muito extensa, te passo dois links
onde você encontra o problema solucionado, são bem similares as fontes.
http://www.doc88.com/p-998978336884.html
http://www.math.dartmouth.edu/~doyle/docs/menage/menage/menage.html



Em 14 de janeiro de 2014 16:53, Antonio Paschoal barz...@dglnet.com.br
escreveu:

Olá.

Se possível for gostaria de uma ajuda com o seguinte problema de
combinatória:

“ Seis casais estão sentados ao redor de uma mesa circular. Quantas são as
distribuições nas quais há alternância de homem e mulher porém

não há nenhum casal sentado lado a lado.”



Me parece claro que o número de distribuições alternadas é dada por
PC(6)=5! x 6! .

Acho que agora há que utilizar o princípio da Inclusão-Exclusão para filtrar
os casais pareados.

Essa é parte difícil do problema.



Agradeço qualquer ajuda.



Um abraço.



Antonio Paschoal







  _


 http://www.avast.com/

Este email está limpo de vírus e malwares porque a proteção do avast!
http://www.avast.com/  Antivírus está ativa.



--
Esta mensagem foi verificada pelo sistema de antivírus e
acredita-se estar livre de perigo.




--
Esta mensagem foi verificada pelo sistema de antivírus e
acredita-se estar livre de perigo.



---
Este email está limpo de vírus e malwares porque a proteção do avast! Antivírus 
está ativa.
http://www.avast.com

-- 
Esta mensagem foi verificada pelo sistema de antivírus e
 acredita-se estar livre de perigo.



RES: [obm-l] problema

2013-12-13 Por tôpico Albert Bouskela
Olá!

 

Faça o gráfico das 2 funções [ f(x)=2^x; g(x)=x ] e você verá o que
acontece…

 

  _  

Albert Bouskela

 mailto:bousk...@ymail.com bousk...@ymail.com

 

De: owner-ob...@mat.puc-rio.br [mailto:owner-ob...@mat.puc-rio.br] Em nome
de saulo nilson
Enviada em: sexta-feira, 13 de dezembro de 2013 17:58
Para: obm-l@mat.puc-rio.br
Assunto: [obm-l] problema

 

encontre todas as soluçoes de 2^x=x


-- 
Esta mensagem foi verificada pelo sistema de antivírus e 
acredita-se estar livre de perigo. 


-- 
Esta mensagem foi verificada pelo sistema de antivírus e
 acredita-se estar livre de perigo.



RES: RES: [obm-l] 2,345 = 2,345000... = 2,34999...

2013-12-04 Por tôpico Albert Bouskela
Olá!

Há várias formas de provar que 0,999...=1. A que eu prefiro é a seguinte:

Na base de numeração 10:
Eq. A: 1/9 + 8/9 = 0,111... + 0,888... = 0,999...

Na base de numeração 9:
Eq. B: 1/10 + 8/10 = 0,1 + 0,8 = 1

Eq. C: (1/9 + 8/9) [base 10] = (1/10 + 8/10) [base 9] = (0,1 + 0,8 = 1) [base 
9] = 1 [base 10]

A Eq. C prova que a Eq. A é equivalente à Eq. B. Logo:
0,999... (base 10) = 1 (base 9) = 1 (base 10)

Albert Bouskela
bousk...@ymail.com

 -Mensagem original-
 De: owner-ob...@mat.puc-rio.br [mailto:owner-ob...@mat.puc-rio.br] Em
 nome de Artur Steiner
 Enviada em: terça-feira, 3 de dezembro de 2013 23:27
 Para: obm-l@mat.puc-rio.br
 Assunto: Re: RES: [obm-l] 2,345 = 2,345000... = 2,34999...
 
 Uma forma rigorosa de provar que 0,999 = 1 é considerar que, por
 definição, 0,999..,é o limite da série geométrica
 
 0,9 + 0,09 + 0,009...
 
 Uma série geométrica cuja razão é 0,1. Logo,
 
 0,999... =  0,9/(1 -0,1) = 0,9/0,9 = 1
 
 Artur Costa Steiner
 
  Em 03/12/2013, às 21:46, Albert Bouskela bousk...@ymail.com
 escreveu:
 
  Ennius,
 
  Existe um procedimento padrão, muito utilizado para transformar dízimas 
  periódicas em frações, que resolve problemas desse tipo ― ver abaixo:
 
  x = 2,344999...
  10x = 23,44999... = 21,105 + 2,344999... = 21,105 + x
  9x = 21105/1000
  x = 21105/9000 = 2,345
 
  Caso queira ser mais elegante:
 
  x = 2,344999... = 2,344 + 0,000999... = (2344+0,999...)/1000 (Eq. 1)
 
  Basta provar que 0,999... = 1
  y = 0,999...
  10y = 9,999... = 9 + 0,999... = 9 + y
  9y = 9
  y = 1
  Voltando à Eq. 1: x = (2344+0,999...)/1000 = (2344+1)/1000 = 2,345
 
  Albert Bouskela
  bousk...@ymail.com
 
  -Mensagem original-
  De: owner-ob...@mat.puc-rio.br [mailto:owner-ob...@mat.puc-rio.br]
 Em
  nome de Ennius Lima Enviada em: terça-feira, 3 de dezembro de 2013
  16:39
  Para: obm-l@mat.puc-rio.br
  Assunto: Re: [obm-l] 2,345 = 2,345000... = 2,34999...
 
  Na verdade, eu quis dizer 2,344999...
  Creio que falta algo na demonstração dada pelo Pedro José, a
 quem
  muito agradeço.
  Gostaria de um exame melhor da questão, se possível for.
  Abraços do Ennius!
  
 
 
 
 
 
 
 
 
  De: Pedro José  petroc...@gmail.com 
  Enviada: Quinta-feira, 28 de Novembro de 2013 17:04
  Para: obm-l@mat.puc-rio.br
  Assunto: Re: [obm-l] 2,345 = 2,345000... = 2,34999...
 
 
 
 
 
 
 
  Bom dia!
 
 
  A primeira é fácil demais:
 
  2 + 3*10^ -1 +4*10^-2 + 5*10^ -3 = 2 + 3*10^ -1 +4*10^-2 + 5*10^
 -3+
  0* 10^-4 + 0*10^-5 + 0*10^-6...
 
 
  A segunda é simples também:
 
  2 + 3*10^ -1 +4*10^-2 + 5*10^ -3= 2 + 3*10^ -1 +4*10^-2 + 9 *10^-
 3 +
  9* 10^-4 + 9*10^-5 + 9*10^-6... Simplificando as parcelas iguias em
  ambos os lados da iguldade teremos:
 
  5*10^ -3= 9 *10^-3 + 9* 10^-4 + 9*10^-5 + 9*10^-6..
 
  o lado direito é o limite de uma soma de PG de razão 1/10 e a1 =
  9*10^-3 quando o número de termos tende a infinito
 
 
  donde 5*10^ -3 = 5*10^-3
 
 
 
 
 
 
 
 
 
 
  Em 28 de novembro de 2013 16:39, Ennius Lima enn...@bol.com.br
  escreveu:
 
  Caros Colegas,
 
 
  Como provar que 2,345 = 2,345... = 2,34999... ?
 
  Desde já, muitíssimo grato!
 
  Ennius Lima
  __
 
  Â
  Â
 
  --
  Esta mensagem foi verificada pelo sistema de antivírus e
  acredita-se estar livre de perigo.
 
 
 
 
  =
  Instruções para entrar na lista, sair da lista e usar a lista em
  http://www.mat.puc-rio.br/~obmlistas/obm-l.html
 
 
 
  =
 
 
 
 
  --
  Esta mensagem foi verificada pelo sistema de antivírus e
  acredita-se estar livre de perigo.
 
  --
  Esta mensagem foi verificada pelo sistema de antiv rus e  acredita-se
  estar livre de perigo.
 
 
 
  =
  Instru  es para entrar na lista, sair da lista e usar a lista em
  http://www.mat.puc-rio.br/~obmlistas/obm-l.html
 
 
  =
 
 
  --
  Esta mensagem foi verificada pelo sistema de antivírus e acredita-se
  estar livre de perigo.
 
 
 
 
 ==
  === Instruções para entrar na lista, sair da lista e usar a lista em
  http://www.mat.puc-rio.br/~obmlistas/obm-l.html
 
 
 ==
  ===
 
 --
 Esta mensagem foi verificada pelo sistema de antiv rus e  acredita-se estar
 livre de perigo.
 
 
 
 =
 Instru  es para entrar na lista, sair da lista e usar a lista em
 http://www.mat.puc-rio.br/~obmlistas/obm-l.html
 
 =


-- 
Esta mensagem foi verificada pelo sistema de antiv�rus e
 acredita-se estar livre de perigo

RES: RES: [obm-l] 2,345 = 2,345000... = 2,34999...

2013-12-04 Por tôpico Albert Bouskela
Olá!

Há várias formas de provar que 0,999...=1. A que eu prefiro é a seguinte:

Na base de numeração 10:
Eq. A: 1/9 + 8/9 = 0,111... + 0,888... = 0,999...

Na base de numeração 9:
Eq. B: 1/10 + 8/10 = 0,1 + 0,8 = 1

Eq. C: (1/9 + 8/9) [base 10] = (1/10 + 8/10) [base 9] = (0,1 + 0,8 = 1) [base 
9] = 1 [base 10]

A Eq. C prova que a Eq. A é equivalente à Eq. B. Logo:
0,999... (base 10) = 1 (base 9) = 1 (base 10)

Albert Bouskela
bousk...@ymail.com

 -Mensagem original-
 De: owner-ob...@mat.puc-rio.br [mailto:owner-ob...@mat.puc-rio.br] Em
 nome de Artur Steiner
 Enviada em: terça-feira, 3 de dezembro de 2013 23:27
 Para: obm-l@mat.puc-rio.br
 Assunto: Re: RES: [obm-l] 2,345 = 2,345000... = 2,34999...
 
 Uma forma rigorosa de provar que 0,999 = 1 é considerar que, por
 definição, 0,999..,é o limite da série geométrica
 
 0,9 + 0,09 + 0,009...
 
 Uma série geométrica cuja razão é 0,1. Logo,
 
 0,999... =  0,9/(1 -0,1) = 0,9/0,9 = 1
 
 Artur Costa Steiner
 
  Em 03/12/2013, às 21:46, Albert Bouskela bousk...@ymail.com
 escreveu:
 
  Ennius,
 
  Existe um procedimento padrão, muito utilizado para transformar dízimas 
  periódicas em frações, que resolve problemas desse tipo ― ver abaixo:
 
  x = 2,344999...

  10x = 23,44999... = 21,105 + 2,344999... = 21,105 + x

  9x = 21105/1000

  x = 21105/9000 = 2,345
 
  Caso queira ser mais elegante:
 
  x = 2,344999... = 2,344 + 0,000999... = (2344+0,999...)/1000 (Eq. 1)
 
  Basta provar que 0,999... = 1

  y = 0,999...

  10y = 9,999... = 9 + 0,999... = 9 + y

  9y = 9

  y = 1

  Voltando à Eq. 1: x = (2344+0,999...)/1000 = (2344+1)/1000 = 2,345
 
  Albert Bouskela
  bousk...@ymail.com


-- 
Esta mensagem foi verificada pelo sistema de antiv�rus e
 acredita-se estar livre de perigo.


=
Instru��es para entrar na lista, sair da lista e usar a lista em
http://www.mat.puc-rio.br/~obmlistas/obm-l.html
=


RES: [obm-l] 2,345 = 2,345000... = 2,34999...

2013-12-03 Por tôpico Albert Bouskela
Ennius,

Existe um procedimento padrão, muito utilizado para transformar dízimas 
periódicas em frações, que resolve problemas desse tipo – ver abaixo:

x = 2,344999...
10x = 23,44999... = 21,105 + 2,344999... = 21,105 + x
9x = 21105/1000
x = 21105/9000 = 2,345

Caso queira ser mais elegante:

x = 2,344999... = 2,344 + 0,000999... = (2344+0,999...)/1000

Basta provar que 0,999... = 1
y = 0,999...
10y = 9,999... = 9 + 0,999... = 9 + y
9y = 9
y = 1
Voltando: x = (2344+1)/1000 = 2,345

Albert Bouskela
bousk...@ymail.com

 -Mensagem original-
 De: owner-ob...@mat.puc-rio.br [mailto:owner-ob...@mat.puc-rio.br] Em
 nome de Ennius Lima
 Enviada em: terça-feira, 3 de dezembro de 2013 16:39
 Para: obm-l@mat.puc-rio.br
 Assunto: Re: [obm-l] 2,345 = 2,345000... = 2,34999...
 
 Na verdade, eu quis dizer 2,344999...
 Creio que falta algo na demonstração dada pelo Pedro José, a quem muito
 agradeço.
 Gostaria de um exame melhor da questão, se possível for.
 Abraços do Ennius!
 
 
 
 
 
 
 
 
 
 De: Pedro José  petroc...@gmail.com 
 Enviada: Quinta-feira, 28 de Novembro de 2013 17:04
 Para: obm-l@mat.puc-rio.br
 Assunto: Re: [obm-l] 2,345 = 2,345000... = 2,34999...
 
 
 
 
 
 
 
 Bom dia!
 
 
 A primeira é fácil demais:
 
 2 + 3*10^ -1 +4*10^-2 + 5*10^ -3 = 2 + 3*10^ -1 +4*10^-2 + 5*10^ -3+
 0* 10^-4 + 0*10^-5 + 0*10^-6...
 
 
 A segunda é simples também:
 
 2 + 3*10^ -1 +4*10^-2 + 5*10^ -3= 2 + 3*10^ -1 +4*10^-2 + 9 *10^-3 +
 9* 10^-4 + 9*10^-5 + 9*10^-6... Simplificando as parcelas iguias em ambos
 os lados da iguldade teremos:
 
 5*10^ -3= 9 *10^-3 + 9* 10^-4 + 9*10^-5 + 9*10^-6..
 
 o lado direito é o limite de uma soma de PG de razão 1/10 e a1 = 9*10^-3
 quando o número de termos tende a infinito
 
 
 donde 5*10^ -3 = 5*10^-3
 
 
 
 
 
 
 
 
 
 
 Em 28 de novembro de 2013 16:39, Ennius Lima enn...@bol.com.br
 escreveu:
 
 Caros Colegas,
 
 
  Como provar que 2,345 = 2,345... = 2,34999... ?
 
  Desde já, muitíssimo grato!
 
  Ennius Lima
  __
 
  Â
  Â
 
  --
  Esta mensagem foi verificada pelo sistema de antivírus e
   acredita-se estar livre de perigo.
 
 
 
 =
  Instruções para entrar na lista, sair da lista e usar a lista em
 http://www.mat.puc-rio.br/~obmlistas/obm-l.html
 
 
 =
 
 
 
 
 --
 Esta mensagem foi verificada pelo sistema de antivírus e  acredita-se estar
 livre de perigo.
 
 --
 Esta mensagem foi verificada pelo sistema de antiv rus e  acredita-se estar
 livre de perigo.
 
 
 =
 Instru  es para entrar na lista, sair da lista e usar a lista em
 http://www.mat.puc-rio.br/~obmlistas/obm-l.html
 
 =


-- 
Esta mensagem foi verificada pelo sistema de antiv�rus e
 acredita-se estar livre de perigo.


=
Instru��es para entrar na lista, sair da lista e usar a lista em
http://www.mat.puc-rio.br/~obmlistas/obm-l.html
=


RES: [obm-l] 2,345 = 2,345000... = 2,34999...

2013-12-03 Por tôpico Albert Bouskela
Corrigindo a formatação!

Ennius,

Existe um procedimento padrão, muito utilizado para transformar dízimas 
periódicas em frações, que resolve problemas desse tipo – ver abaixo:

x = 2,344999...

10x = 23,44999... = 21,105 + 2,344999... = 21,105 + x

9x = 21105/1000

x = 21105/9000 = 2,345

Caso queira ser mais elegante:

x = 2,344999... = 2,344 + 0,000999... = (2344+0,999...)/1000

Basta provar que 0,999... = 1
y = 0,999...
10y = 9,999... = 9 + 0,999... = 9 + y
9y = 9
y = 1
Voltando: x = (2344+1)/1000 = 2,345

Albert Bouskela
bousk...@ymail.com

 -Mensagem original-
 De: owner-ob...@mat.puc-rio.br [mailto:owner-ob...@mat.puc-rio.br] Em
 nome de Ennius Lima
 Enviada em: terça-feira, 3 de dezembro de 2013 16:39
 Para: obm-l@mat.puc-rio.br
 Assunto: Re: [obm-l] 2,345 = 2,345000... = 2,34999...
 
 Na verdade, eu quis dizer 2,344999...
 Creio que falta algo na demonstração dada pelo Pedro José, a quem muito
 agradeço.
 Gostaria de um exame melhor da questão, se possível for.
 Abraços do Ennius!
 
 
 
 
 
 
 
 
 
 De: Pedro José  petroc...@gmail.com 
 Enviada: Quinta-feira, 28 de Novembro de 2013 17:04
 Para: obm-l@mat.puc-rio.br
 Assunto: Re: [obm-l] 2,345 = 2,345000... = 2,34999...
 
 
 
 
 
 
 
 Bom dia!
 
 
 A primeira é fácil demais:
 
 2 + 3*10^ -1 +4*10^-2 + 5*10^ -3 = 2 + 3*10^ -1 +4*10^-2 + 5*10^ -3+
 0* 10^-4 + 0*10^-5 + 0*10^-6...
 
 
 A segunda é simples também:
 
 2 + 3*10^ -1 +4*10^-2 + 5*10^ -3= 2 + 3*10^ -1 +4*10^-2 + 9 *10^-3 +
 9* 10^-4 + 9*10^-5 + 9*10^-6... Simplificando as parcelas iguias em ambos
 os lados da iguldade teremos:
 
 5*10^ -3= 9 *10^-3 + 9* 10^-4 + 9*10^-5 + 9*10^-6..
 
 o lado direito é o limite de uma soma de PG de razão 1/10 e a1 = 9*10^-3
 quando o número de termos tende a infinito
 
 
 donde 5*10^ -3 = 5*10^-3
 
 
 
 
 
 
 
 
 
 
 Em 28 de novembro de 2013 16:39, Ennius Lima enn...@bol.com.br
 escreveu:
 
 Caros Colegas,
 
 
  Como provar que 2,345 = 2,345... = 2,34999... ?
 
  Desde já, muitíssimo grato!
 
  Ennius Lima
  __
 
  Â
  Â
 
  --
  Esta mensagem foi verificada pelo sistema de antivírus e
   acredita-se estar livre de perigo.
 
 
 
 =
  Instruções para entrar na lista, sair da lista e usar a lista em
 http://www.mat.puc-rio.br/~obmlistas/obm-l.html
 
 
 =
 
 
 
 
 --
 Esta mensagem foi verificada pelo sistema de antivírus e  acredita-se estar
 livre de perigo.
 
 --
 Esta mensagem foi verificada pelo sistema de antiv rus e  acredita-se estar
 livre de perigo.
 
 
 =
 Instru  es para entrar na lista, sair da lista e usar a lista em
 http://www.mat.puc-rio.br/~obmlistas/obm-l.html
 
 =


-- 
Esta mensagem foi verificada pelo sistema de antiv�rus e
 acredita-se estar livre de perigo.


=
Instru��es para entrar na lista, sair da lista e usar a lista em
http://www.mat.puc-rio.br/~obmlistas/obm-l.html
=


Re: RES: [obm-l] 2,345 = 2,345000... = 2,34999...

2013-12-03 Por tôpico Artur Steiner
Uma forma rigorosa de provar que 0,999 = 1 é considerar que, por definição, 
0,999..,é o limite da série geométrica

0,9 + 0,09 + 0,009...

Uma série geométrica cuja razão é 0,1. Logo,

0,999... =  0,9/(1 -0,1) = 0,9/0,9 = 1

Artur Costa Steiner

 Em 03/12/2013, às 21:46, Albert Bouskela bousk...@ymail.com escreveu:
 
 Ennius,
 
 Existe um procedimento padrão, muito utilizado para transformar dízimas 
 periódicas em frações, que resolve problemas desse tipo – ver abaixo:
 
 x = 2,344999...
 10x = 23,44999... = 21,105 + 2,344999... = 21,105 + x
 9x = 21105/1000
 x = 21105/9000 = 2,345
 
 Caso queira ser mais elegante:
 
 x = 2,344999... = 2,344 + 0,000999... = (2344+0,999...)/1000
 
 Basta provar que 0,999... = 1
 y = 0,999...
 10y = 9,999... = 9 + 0,999... = 9 + y
 9y = 9
 y = 1
 Voltando: x = (2344+1)/1000 = 2,345
 
 Albert Bouskela
 bousk...@ymail.com
 
 -Mensagem original-
 De: owner-ob...@mat.puc-rio.br [mailto:owner-ob...@mat.puc-rio.br] Em
 nome de Ennius Lima
 Enviada em: terça-feira, 3 de dezembro de 2013 16:39
 Para: obm-l@mat.puc-rio.br
 Assunto: Re: [obm-l] 2,345 = 2,345000... = 2,34999...
 
 Na verdade, eu quis dizer 2,344999...
 Creio que falta algo na demonstração dada pelo Pedro José, a quem muito
 agradeço.
 Gostaria de um exame melhor da questão, se possível for.
 Abraços do Ennius!
 
 
 
 
 
 
 
 
 
 De: Pedro José  petroc...@gmail.com 
 Enviada: Quinta-feira, 28 de Novembro de 2013 17:04
 Para: obm-l@mat.puc-rio.br
 Assunto: Re: [obm-l] 2,345 = 2,345000... = 2,34999...
 
 
 
 
 
 
 
 Bom dia!
 
 
 A primeira é fácil demais:
 
 2 + 3*10^ -1 +4*10^-2 + 5*10^ -3 = 2 + 3*10^ -1 +4*10^-2 + 5*10^ -3+
 0* 10^-4 + 0*10^-5 + 0*10^-6...
 
 
 A segunda é simples também:
 
 2 + 3*10^ -1 +4*10^-2 + 5*10^ -3= 2 + 3*10^ -1 +4*10^-2 + 9 *10^-3 +
 9* 10^-4 + 9*10^-5 + 9*10^-6... Simplificando as parcelas iguias em ambos
 os lados da iguldade teremos:
 
 5*10^ -3= 9 *10^-3 + 9* 10^-4 + 9*10^-5 + 9*10^-6..
 
 o lado direito é o limite de uma soma de PG de razão 1/10 e a1 = 9*10^-3
 quando o número de termos tende a infinito
 
 
 donde 5*10^ -3 = 5*10^-3
 
 
 
 
 
 
 
 
 
 
 Em 28 de novembro de 2013 16:39, Ennius Lima enn...@bol.com.br
 escreveu:
 
 Caros Colegas,
 
 
 Como provar que 2,345 = 2,345... = 2,34999... ?
 
 Desde já, muitíssimo grato!
 
 Ennius Lima
 __
 
 Â
 Â
 
 --
 Esta mensagem foi verificada pelo sistema de antivírus e
  acredita-se estar livre de perigo.
 
 
 
 =
 Instruções para entrar na lista, sair da lista e usar a lista em
 http://www.mat.puc-rio.br/~obmlistas/obm-l.html
 
 
 =
 
 
 
 
 --
 Esta mensagem foi verificada pelo sistema de antivírus e  acredita-se estar
 livre de perigo.
 
 --
 Esta mensagem foi verificada pelo sistema de antiv rus e  acredita-se estar
 livre de perigo.
 
 
 =
 Instru  es para entrar na lista, sair da lista e usar a lista em
 http://www.mat.puc-rio.br/~obmlistas/obm-l.html
 
 =
 
 
 -- 
 Esta mensagem foi verificada pelo sistema de antivírus e
 acredita-se estar livre de perigo.
 
 
 =
 Instruções para entrar na lista, sair da lista e usar a lista em
 http://www.mat.puc-rio.br/~obmlistas/obm-l.html
 =

-- 
Esta mensagem foi verificada pelo sistema de antiv�rus e
 acredita-se estar livre de perigo.


=
Instru��es para entrar na lista, sair da lista e usar a lista em
http://www.mat.puc-rio.br/~obmlistas/obm-l.html
=


[obm-l] RES: [obm-l] definição de volumes de sólidos

2013-09-30 Por tôpico EPVN
Olá.

Mas é claro que o conceito de volume é essencial, aparece, historicamente,
muito cedo. Os povos antigos, egípcios, babilônios, chineses, hindus, sempre

tiveram necessidade de calcular o volume de alguns sólidos relacionados ao
armazenamento de alimentos. Os egípcios sabiam calcular muito bem

o volume de um tronco de pirâmide.

Para efeitos didáticos podemos sim explicar que o volume de um prisma é
gerado como uma área varrendo uma aresta lateral do referido prisma, isso
ajuda

a fixar o conceito de volume.

Agora dizer que a fórmula V=Bxh  é desnecessária já é um disparate completo.

Pode continuar explicando a seus alunos essa fórmula, eles vão precisar
dela.

Espero ter colaborado.

Um abraço.

Claudio.

 

  _  

De: owner-ob...@mat.puc-rio.br [mailto:owner-ob...@mat.puc-rio.br] Em nome
de Marco Antonio Leal
Enviada em: sábado, 28 de setembro de 2013 22:14
Para: obm-l@mat.puc-rio.br
Assunto: [obm-l] definição de volumes de sólidos

 

Ola ! ensinando geometria espacial em uma instituição nova, na qual comecei
a trabalhar a pouco tempo, me deparei com um colega que ensina que não é
necessário que o aluno saiba que o volume de um prisma, é área da base vezes
altura, que este conceito é completamente falho, não existe área da base
vezes altura, o que existe é escolher uma área lateral determinada e varrer
seu comprimento. Tal maneira milagrosa de ensinar me deixou inquieto, já
pesquisei em vários sites, livros e ex-professores meus e resolvi pedir
ajuda a vocês. O conceito de volume é desnecessário mesmo 

 


-- 
Esta mensagem foi verificada pelo sistema de antivírus e 
acredita-se estar livre de perigo. 


-- 
Esta mensagem foi verificada pelo sistema de antivírus e
 acredita-se estar livre de perigo.



Re: [obm-l] RES: [obm-l] definição de volumes de sólidos

2013-09-30 Por tôpico marcoantonio_elementar
Muito obrigado, estava preocupado de estar ensinando errado ou estar obsoleto


Enviado por Samsung Mobile

 Mensagem original 
De : EPVN barz...@dglnet.com.br 
Data:  
Para: obm-l@mat.puc-rio.br 
Assunto: [obm-l] RES: [obm-l] definição de volumes de sólidos 
 
Olá.
Mas é claro que o conceito de volume é essencial, aparece, historicamente, 
muito cedo. Os povos antigos, egípcios, babilônios, chineses, hindus, sempre
tiveram necessidade de calcular o volume de alguns sólidos relacionados ao 
armazenamento de alimentos. Os egípcios sabiam calcular muito bem
o volume de um tronco de pirâmide.
Para efeitos didáticos podemos sim explicar que o volume de um prisma é gerado 
como uma área varrendo uma aresta lateral do referido prisma, isso ajuda
a fixar o conceito de volume.
Agora dizer que a fórmula V=Bxh  é desnecessária já é um disparate completo.
Pode continuar explicando a seus alunos essa fórmula, eles vão precisar dela.
Espero ter colaborado.
Um abraço.
Claudio.
 
De: owner-ob...@mat.puc-rio.br [mailto:owner-ob...@mat.puc-rio.br] Em nome de 
Marco Antonio Leal
Enviada em: sábado, 28 de setembro de 2013 22:14
Para: obm-l@mat.puc-rio.br
Assunto: [obm-l] definição de volumes de sólidos
 
Ola ! ensinando geometria espacial em uma instituição nova, na qual comecei  a 
trabalhar a pouco tempo, me deparei com um colega que ensina que não é 
necessário que o aluno saiba que o volume de um prisma, é área da base vezes 
altura, que este conceito é completamente falho, não existe área da base vezes 
altura, o que existe é escolher uma área lateral determinada e varrer seu 
comprimento. Tal maneira milagrosa de ensinar me deixou inquieto, já pesquisei 
em vários sites, livros e ex-professores meus e resolvi pedir ajuda a vocês. O 
conceito de volume é desnecessário mesmo 
 

-- 
Esta mensagem foi verificada pelo sistema de antivírus e 
acredita-se estar livre de perigo.

-- 
Esta mensagem foi verificada pelo sistema de antivírus e 
acredita-se estar livre de perigo.
-- 
Esta mensagem foi verificada pelo sistema de antiv�rus e
 acredita-se estar livre de perigo.



[obm-l] RE: RES: [obm-l] Fatores 3

2013-09-15 Por tôpico dnasimento
O teorema de lagrange ajuda a responder esse problema!Seja n! = n.(n-1).(n-2). ... .2.1 e k o fator primo que queremos determinar a quantidade, entãoLk = n/k + n/k^2 + n/k^3 +...Devemos parar as divisões quando a potência do denominador for maior que o denominador. Caso a divisão não seja exata, tomamos a parte inteira .Em hoje 00:03 Benedito  escreveu:Observe que, no produto 3.9.15...99 existem 17 fatores, pois 3,9,15,...,99 estão em progressão aritmética de razão 6.De: owner-ob...@mat.puc-rio.br [mailto:owner-ob...@mat.puc-rio.br] Em nome de faraujoco...@yahoo.com.brEnviada em: sábado, 7 de setembro de 2013 20:52Para: obm-l@mat.puc-rio.brAssunto: Re: [obm-l] Fatores 3Perdão. Sao nos inteiros. A única coisa que não entendi foi o expoente 17. Enviado via iPhoneEm 07/09/2013, às 20:30, terence thirteen peterdirich...@gmail.com escreveu:Um terço tem o fator 3Um nono tem o fator 9Um 27-avos tem o fator 27E assim por diante...Em 7 de setembro de 2013 18:24, Benedito bened...@ufrnet.br escreveu:Resposta 32.( 1.3.5.7.9. ... .99 ) = (3.9.15...99)(1.5.7...97) = 3^17(1.3.5...33). (1.5.7...97) = 3^17.(3.9.15...33)(1.5.7...97)                  = 3^17.3^11.(1.3.5.7.9.10.11).(1.5.7...97) = 3^28.(3.6.9).(1.5.7.10.11). )(1.5.7...97) = 3^28. 3^3(1.2.3).(1.5.7.10.11). )(1.5.7...97) =                  = 3^32 (1.2) .(1.5.7.10.11). )(1.5.7...97-Mensagem original-De: owner-ob...@mat.puc-rio.br [mailto:owner-ob...@mat.puc-rio.br] Em nome de faraujoco...@yahoo.com.brEnviada em: sábado, 7 de setembro de 2013 13:32Para: obm-l@mat.puc-rio.brAssunto: [obm-l] Fatores 3Olá.Tenho uma duvida p. discutirmos.Fatorando o produto dos 100 primeiros impares qual quantidade máxima de fatores 3?( 1.3.5.7.9. ... .99 ) = 3^k [k max.]Enviado via iPhone--Esta mensagem foi verificada pelo sistema de antiv rus e Âacredita-se estar livre de perigo.=Instru Âes para entrar na lista, sair da lista e usar a lista emhttp://www.mat.puc-rio.br/~obmlistas/obm-l.html=--Esta mensagem foi verificada pelo sistema de antivírus eÂacredita-se estar livre de perigo.=Instruções para entrar na lista, sair da lista e usar a lista emhttp://www.mat.puc-rio.br/~obmlistas/obm-l.html=-- /**/神が祝福Torres -- Esta mensagem foi verificada pelo sistema de antivírus e acredita-se estar livre de perigo. -- Esta mensagem foi verificada pelo sistema de antiv�s e acredita-se estar livre de perigo. --
Esta mensagem foi verificada pelo sistema de antivrus e 
 acredita-se estar livre de perigo.


--
Esta mensagem foi verificada pelo sistema de antivírus e 
 acredita-se estar livre de perigo.

Instruções para entrar na lista, sair da lista e usar a lista em
http://www.mat.puc-rio.br/~obmlistas/obm-l.html
===

Re: [obm-l] RE: RES: [obm-l] Fatores 3

2013-09-15 Por tôpico faraujocosta
Acontece que o produto não é dos inteiros consecutivos e so  dos impares. 

Enviado via iPhone

Em 16/09/2013, às 00:08, dnasime...@terra.com.br escreveu:

 O teorema de lagrange ajuda a responder esse problema!
 
 Seja n! = n.(n-1).(n-2). ... .2.1 e k o fator primo que queremos determinar a 
 quantidade, então 
 
 Lk = n/k + n/k^2 + n/k^3 +...
 
 Devemos parar as divisões quando a potência do denominador for maior que o 
 denominador. Caso a divisão  não seja exata, tomamos a parte inteira . 
 
 
 Em hoje 00:03 Benedito escreveu:
 Observe que, no produto 3.9.15...99 existem 17 fatores, pois 3,9,15,...,99  
 estão em progressão aritmética de razão 6.
 
  
 
 De: owner-ob...@mat.puc-rio.br [mailto:owner-ob...@mat.puc-rio.br] Em nome de 
 faraujoco...@yahoo.com.br
 Enviada em: sábado, 7 de setembro de 2013 20:52
 Para: obm-l@mat.puc-rio.br
 Assunto: Re: [obm-l] Fatores 3
 
  
 
 Perdão.  Sao nos inteiros.  
 
 A única coisa que não entendi foi o expoente 17.  
 
 Enviado via iPhone
 
 
 Em 07/09/2013, Ã s 20:30, terence thirteen peterdirich...@gmail.com 
 escreveu:
 
 Um terço tem o fator 3
 
 Um nono tem o fator 9
 
 Um 27-avos tem o fator 27
 
  
 
 E assim por diante...
 
  
 
 Em 7 de setembro de 2013 18:24, Benedito bened...@ufrnet.br escreveu:
 
 Resposta 32.
 ( 1.3.5.7.9. ... .99 ) = (3.9.15...99)(1.5.7...97) = 3^17(1.3.5...33). 
 (1.5.7...97) = 3^17.(3.9.15...33)(1.5.7...97)
 Â  Â  Â  Â  Â  Â  Â  Â  Â  Â  Â  Â  Â  Â  Â  Â  Â  Â  = 
 3^17.3^11.(1.3.5.7.9.10.11).(1.5.7...97) = 3^28.(3.6.9).(1.5.7.10.11). 
 )(1.5.7...97) = 3^28. 3^3(1.2.3).(1.5.7.10.11). )(1.5.7...97) =
 Â  Â  Â  Â  Â  Â  Â  Â  Â  Â  Â  Â  Â  Â  Â  Â  Â  Â  = 
 3^32 (1.2) .(1.5.7.10.11). )(1.5.7...97
 
 -Mensagem original-
 De: owner-ob...@mat.puc-rio.br [mailto:owner-ob...@mat.puc-rio.br] Em nome de 
 faraujoco...@yahoo.com.br
 Enviada em: sábado, 7 de setembro de 2013 13:32
 Para: obm-l@mat.puc-rio.br
 Assunto: [obm-l] Fatores 3
 
 
 Olá.
 Tenho uma duvida p. discutirmos.
 Fatorando o produto dos 100 primeiros impares qual quantidade máxima 
 de fatores 3?
 
 ( 1.3.5.7.9. ... .99 ) = 3^k [k max.]
 
 Enviado via iPhone
 --
 
 Esta mensagem foi verificada pelo sistema de antiv rus e  acredita-se estar 
 livre de perigo.
 
 
 =
 Instru  es para entrar na lista, sair da lista e usar a lista em
 
 http://www.mat.puc-rio.br/~obmlistas/obm-l.html
 =
 
 
 --
 Esta mensagem foi verificada pelo sistema de antivírus e
 Â acredita-se estar livre de perigo.
 
 
 =
 Instruções para entrar na lista, sair da lista e usar a lista em
 http://www.mat.puc-rio.br/~obmlistas/obm-l.html
 =
 
 
 
 
  
 
 -- 
 /**/
 神が祝福
 
 Torres
 
 
 -- 
 Esta mensagem foi verificada pelo sistema de antivírus e 
 acredita-se estar livre de perigo.
 
 
 -- 
 Esta mensagem foi verificada pelo sistema de antiv�s e 
 acredita-se estar livre de perigo.
 
 
 -- 
 Esta mensagem foi verificada pelo sistema de antivrus e 
 acredita-se estar livre de perigo.
 
 -- 
 Esta mensagem foi verificada pelo sistema de antivírus e 
 acredita-se estar livre de perigo. 
 Instruções
  para entrar na lista, sair da lista e usar a lista em 
 http://www.mat.puc-rio.br/~obmlistas/obm-l.html 
 

-- 
Esta mensagem foi verificada pelo sistema de antiv�rus e
 acredita-se estar livre de perigo.



RES: [obm-l] Fatores 3

2013-09-08 Por tôpico Benedito
Observe que, no produto 3.9.15...99 existem 17 fatores, pois 3,9,15,...,99  
estão em progressão aritmética de razão 6.

 

De: owner-ob...@mat.puc-rio.br [mailto:owner-ob...@mat.puc-rio.br] Em nome de 
faraujoco...@yahoo.com.br
Enviada em: sábado, 7 de setembro de 2013 20:52
Para: obm-l@mat.puc-rio.br
Assunto: Re: [obm-l] Fatores 3

 

Perdão.  Sao nos inteiros.  

A única coisa que não entendi foi o expoente 17.  

Enviado via iPhone


Em 07/09/2013, às 20:30, terence thirteen peterdirich...@gmail.com 
mailto:peterdirich...@gmail.com  escreveu:

Um terço tem o fator 3

Um nono tem o fator 9

Um 27-avos tem o fator 27

 

E assim por diante...

 

Em 7 de setembro de 2013 18:24, Benedito bened...@ufrnet.br 
mailto:bened...@ufrnet.br  escreveu:

Resposta 32.
( 1.3.5.7.9. ... .99 ) = (3.9.15...99)(1.5.7...97) = 3^17(1.3.5...33). 
(1.5.7...97) = 3^17.(3.9.15...33)(1.5.7...97)
                                    = 
3^17.3^11.(1.3.5.7.9.10.11).(1.5.7...97) = 3^28.(3.6.9).(1.5.7.10.11). 
)(1.5.7...97) = 3^28. 3^3(1.2.3).(1.5.7.10.11). )(1.5.7...97) =
                                    = 3^32 (1.2) 
.(1.5.7.10.11). )(1.5.7...97

-Mensagem original-
De: owner-ob...@mat.puc-rio.br mailto:owner-ob...@mat.puc-rio.br  
[mailto:owner-ob...@mat.puc-rio.br mailto:owner-ob...@mat.puc-rio.br ] Em 
nome de faraujoco...@yahoo.com.br mailto:faraujoco...@yahoo.com.br 
Enviada em: sábado, 7 de setembro de 2013 13:32
Para: obm-l@mat.puc-rio.br mailto:obm-l@mat.puc-rio.br 
Assunto: [obm-l] Fatores 3


Olá.
Tenho uma duvida p. discutirmos.
Fatorando o produto dos 100 primeiros impares qual quantidade máxima de 
fatores 3?

( 1.3.5.7.9. ... .99 ) = 3^k [k max.]

Enviado via iPhone
--

Esta mensagem foi verificada pelo sistema de antiv rus e  acredita-se estar 
livre de perigo.


=
Instru  es para entrar na lista, sair da lista e usar a lista em

http://www.mat.puc-rio.br/~obmlistas/obm-l.html
=


--
Esta mensagem foi verificada pelo sistema de antivírus e
 acredita-se estar livre de perigo.


=
Instruções para entrar na lista, sair da lista e usar a lista em
http://www.mat.puc-rio.br/~obmlistas/obm-l.html
=





 

-- 
/**/
神が祝福

Torres 


-- 
Esta mensagem foi verificada pelo sistema de antivírus e 
acredita-se estar livre de perigo. 


-- 
Esta mensagem foi verificada pelo sistema de antiv�s e 
acredita-se estar livre de perigo. 


-- 
Esta mensagem foi verificada pelo sistema de antiv�rus e
 acredita-se estar livre de perigo.



RES: [obm-l] Fatores 3

2013-09-07 Por tôpico Benedito
Resposta 32.
( 1.3.5.7.9. ... .99 ) = (3.9.15...99)(1.5.7...97) = 3^17(1.3.5...33). 
(1.5.7...97) = 3^17.(3.9.15...33)(1.5.7...97)
= 3^17.3^11.(1.3.5.7.9.10.11).(1.5.7...97) 
= 3^28.(3.6.9).(1.5.7.10.11). )(1.5.7...97) = 3^28. 3^3(1.2.3).(1.5.7.10.11). 
)(1.5.7...97) =
= 3^32 (1.2) .(1.5.7.10.11). )(1.5.7...97

-Mensagem original-
De: owner-ob...@mat.puc-rio.br [mailto:owner-ob...@mat.puc-rio.br] Em nome de 
faraujoco...@yahoo.com.br
Enviada em: sábado, 7 de setembro de 2013 13:32
Para: obm-l@mat.puc-rio.br
Assunto: [obm-l] Fatores 3

Olá.  
Tenho uma duvida p. discutirmos. 
Fatorando o produto dos 100 primeiros impares qual quantidade máxima de fatores 
3?

( 1.3.5.7.9. ... .99 ) = 3^k [k max.]

Enviado via iPhone
--
Esta mensagem foi verificada pelo sistema de antiv rus e  acredita-se estar 
livre de perigo.


=
Instru  es para entrar na lista, sair da lista e usar a lista em
http://www.mat.puc-rio.br/~obmlistas/obm-l.html
=


-- 
Esta mensagem foi verificada pelo sistema de antiv�rus e
 acredita-se estar livre de perigo.


=
Instru��es para entrar na lista, sair da lista e usar a lista em
http://www.mat.puc-rio.br/~obmlistas/obm-l.html
=


RES: [obm-l] O Paradoxo da Flecha - Off Topic

2013-08-26 Por tôpico Albert Bouskela
Olá!

 

Zenão – é claro! – não conhecia a Transformada de Galileu, base do Movimento
Relativo da mecânica newtoniana.

 

Muito mais interessante é o Paradoxo dos Gêmeos, cuja solução ainda não é um
consenso entre os físicos relativistas.

 

  _  

Albert Bouskela

 mailto:bousk...@ymail.com bousk...@ymail.com

 

De: owner-ob...@mat.puc-rio.br [mailto:owner-ob...@mat.puc-rio.br] Em nome
de Henrique Rennó
Enviada em: segunda-feira, 26 de agosto de 2013 15:40
Para: obm-l
Assunto: Re: [obm-l] O Paradoxo da Flecha - Off Topic

 

Acho que depende da referência.

 

2013/8/26 luiz silva luizfelipec...@yahoo.com.br

 

Pessoal,

 

Vcs consiceram este paradoxo resolvido ?

 


. O paradoxo da flecha*


Para um objeto se mover, sua posição no espaço deve mudar, certo? Pois bem,
esse paradoxo do filósofo grego Zeno de Eleia (495 a.C 430 a.C) diz que os
objetos não se movem. Considere um instante como uma fotografia, cada espaço
de tempo é uma fotografia na qual o objeto está parado. O exemplo usado por
Zeno é o de uma flecha voando pelo ar. Se pudéssemos pegar o máximo de
fotografias possíveis durante o movimento, em todas elas o objeto está
parado, ou seja, ele jamais se moveu.


 

*retirado do site BuleVoador.

 

Abs

Felipe

 


-- 
Esta mensagem foi verificada pelo sistema de antivírus e 
acredita-se estar livre de perigo. 





 

-- 
Henrique


-- 
Esta mensagem foi verificada pelo sistema de antivírus e 
acredita-se estar livre de perigo. 


-- 
Esta mensagem foi verificada pelo sistema de antivírus e
 acredita-se estar livre de perigo.



RES: [obm-l] O Paradoxo da Flecha - Off Topic

2013-08-26 Por tôpico Albert Bouskela
Olá!

Não é tão simples assim: – É necessário apelar para a Transformada de
Galileu! Você tira uma foto da flecha e depois tira outra – há duas
possibilidades:

1) As fotos são idênticas: – (i) A flecha está parada e você também; e (ii)
A sua velocidade é – vetorialmente – idêntica a da flecha, o que engloba o
item «i»;
2) As fotos não são idênticas: – A sua velocidade NÃO é – vetorialmente –
idêntica a da flecha. Logo, se você está parado, então a flecha está se
movendo.

E ainda há mais a discutir: – (i) Parado significa não ter velocidade em
relação a um referencial inercial comum ao observador e à flecha; e (ii)
Admitir que tanto você, como a flecha, se movem em velocidades
insignificantes em relação a da luz.

E, por aí, vai...

A propósito, caso tenham interesse em bizarrices sobre o tempo (ou
espaço-tempo), leiam Sonhos de Einstein (Alan Lightman).

Albert Bouskela
bousk...@ymail.com

 -Mensagem original-
 De: owner-ob...@mat.puc-rio.br [mailto:owner-ob...@mat.puc-rio.br] Em
 nome de Ralph Teixeira
 Enviada em: segunda-feira, 26 de agosto de 2013 21:47
 Para: obm-l@mat.puc-rio.br
 Assunto: Re: [obm-l] O Paradoxo da Flecha - Off Topic
 
 2013/8/26 Bernardo Freitas Paulo da Costa bernardo...@gmail.com:
  Já temos um problema aqui. Ao considerar que num instante, a flecha
  está parada. Nada disso, ela TEM velocidade, mas observar o movimento
  só faz sentido AO LONGO do tempo, não numa fotografia, e é por isso
  que você não vê a flecha se mexer nesta dita fotografia.
 
 zero!
 
 Oi, Paulo Cesar, eu ia responder... mas, po, o Bernardo expressou
 exatamente o que eu penso do assunto nesse paragrafo ai em cima, de
 maneira mais clara do que eu seria capaz de dizer. :) :)
 
 Abraco,
 Ralph
 
 --
 Esta mensagem foi verificada pelo sistema de antivírus e  acredita-se
estar
 livre de perigo.
 
 
 
 =
 Instruções para entrar na lista, sair da lista e usar a lista em
 http://www.mat.puc-rio.br/~obmlistas/obm-l.html
 
 =


-- 
Esta mensagem foi verificada pelo sistema de antivírus e
 acredita-se estar livre de perigo.


=
Instruções para entrar na lista, sair da lista e usar a lista em
http://www.mat.puc-rio.br/~obmlistas/obm-l.html
=


RES: [obm-l] Problemas interessantes

2013-08-25 Por tôpico Benedito
Você está correto quando à sua observação.  Foi um equívoco.  O certo é:

Um triângulo equilátero de lado 2012 está dividido em 2012  triângulos
equiláteros  de lado 1.

 

 

De: owner-ob...@mat.puc-rio.br [mailto:owner-ob...@mat.puc-rio.br] Em nome
de Eduardo Wilner
Enviada em: sábado, 24 de agosto de 2013 21:31
Para: obm-l@mat.puc-rio.br
Assunto: Re: [obm-l] Problemas interessantes

 

Um triângulo  equilátero de lado n se divide em n triângulos de lado 1
???!!!

 

 

  _  

De: Benedito bened...@ufrnet.br mailto:bened...@ufrnet.br 
Para: obm-l@mat.puc-rio.br mailto:obm-l@mat.puc-rio.br  
Enviadas: Quinta-feira, 22 de Agosto de 2013 4:39
Assunto: [obm-l] Problemas interessantes

 

Segue dois problemas interessantes.

Benedito

 

Problema 1

Um triângulo equilátero de lado 2012 está dividido em 2012  triângulos
equiláteros menores de lado 1

mediante paralelas ao seus lados. Em cada vértice de um triângulo menor há
uma formiga. No mesmo instante, 

todas as formigas começam a caminhar com a mesma velocidade pelas retas da
triangulação.

Ao chegar  a outro vértice giram 60º ou 120º à esquerda ou à direita e
seguem movendo-se.

Determinar se é possível que este movimento se desenvolva para sempre sem
ter  nunca duas

formigas em um mesmo vértice de um triângulo menor.

 

Problema 2

Associar aos vértices de um polígono convexo de 33 lados os números inteiros
de 1 a 33, sem repetir, e em seguida, associar aos lados do polígono a soma
dos números de seus extremos. 

O objetivo é que os números associados aos lados sejam 33 inteiros
consecutivos ordenados.


-- 
Esta mensagem foi verificada pelo sistema de antivírus e 
acredita-se estar livre de perigo. 

 


-- 
Esta mensagem foi verificada pelo sistema de antivírus e 
acredita-se estar livre de perigo. 


-- 
Esta mensagem foi verificada pelo sistema de antivírus e
 acredita-se estar livre de perigo.



RES: [obm-l] Problemas interessantes

2013-08-25 Por tôpico Benedito
Eduardo,

A sua observação faz sentido. O que falta é a vírgula !!!:

Um triângulo equilátero de lado 2012 está dividido em 2012  triângulos
equiláteros menores, de lado 1.

 

Obrigado.

Benedito

De: owner-ob...@mat.puc-rio.br [mailto:owner-ob...@mat.puc-rio.br] Em nome
de Eduardo Wilner
Enviada em: sábado, 24 de agosto de 2013 21:31
Para: obm-l@mat.puc-rio.br
Assunto: Re: [obm-l] Problemas interessantes

 

Um triângulo  equilátero de lado n se divide em n triângulos de lado 1
???!!!:



 

 

  _  

De: Benedito bened...@ufrnet.br mailto:bened...@ufrnet.br 
Para: obm-l@mat.puc-rio.br mailto:obm-l@mat.puc-rio.br  
Enviadas: Quinta-feira, 22 de Agosto de 2013 4:39
Assunto: [obm-l] Problemas interessantes

 

Segue dois problemas interessantes.

Benedito

 

Problema 1

Um triângulo equilátero de lado 2012 está dividido em 2012  triângulos
equiláteros menores de lado 1

mediante paralelas ao seus lados. Em cada vértice de um triângulo menor há
uma formiga. No mesmo instante, 

todas as formigas começam a caminhar com a mesma velocidade pelas retas da
triangulação.

Ao chegar  a outro vértice giram 60º ou 120º à esquerda ou à direita e
seguem movendo-se.

Determinar se é possível que este movimento se desenvolva para sempre sem
ter  nunca duas

formigas em um mesmo vértice de um triângulo menor.

 

Problema 2

Associar aos vértices de um polígono convexo de 33 lados os números inteiros
de 1 a 33, sem repetir, e em seguida, associar aos lados do polígono a soma
dos números de seus extremos. 

O objetivo é que os números associados aos lados sejam 33 inteiros
consecutivos ordenados.


-- 
Esta mensagem foi verificada pelo sistema de antivírus e 
acredita-se estar livre de perigo. 

 


-- 
Esta mensagem foi verificada pelo sistema de antivírus e 
acredita-se estar livre de perigo. 


-- 
Esta mensagem foi verificada pelo sistema de antivírus e
 acredita-se estar livre de perigo.



[obm-l] RES: [obm-l] Re: Dízimas periódicas

2013-08-19 Por tôpico Albert Bouskela
Olá! Ótimo que tenha gostado, entretanto, para ficar “direitinho”, faltou uma 
passagem:

Equação A: [ 1/9 + 8/9 ] « na base 10 » = [ 1/10 + 8/10 ] « na base 9 » = [ 0,1 
+ 0,8 = 1 ] « na base 9 » = [ 1 ] « na base 10 »

A passagem que falta:

 

Na base 10: 1/9 + 8/9 = 0,111… + 0,888… = 0,999…

 

Na base 9: 1/10 + 8/10 = 0,1 + 0,8 = 1

 

Na equação A, provamos que as 2 expressões acima são equivalentes. Logo: 0,999… 
« na base 10 » = 1 « na base 9 » = 1 « na base 10 » 

 

  _  

Albert Bouskela

 mailto:bousk...@ymail.com bousk...@ymail.com

 

De: owner-ob...@mat.puc-rio.br [mailto:owner-ob...@mat.puc-rio.br] Em nome de 
Luiz Antonio Rodrigues
Enviada em: segunda-feira, 19 de agosto de 2013 16:45
Para: obm-l@mat.puc-rio.br
Assunto: [obm-l] Re: Dízimas periódicas

 

Oi, Albert!

Achei genial essa saída!

Um abraço e obrigado!

Luiz

On Wednesday, August 14, 2013, Albert Bouskela wrote:

Olá a todos!

 

Há uma maneira de dar um tiro de canhão (ou jogar uma bomba atômica) para matar 
essa mosca: ― Provar que 0,999… = 1. Igual MESMO, só escrito de outra forma.

 

É simples: ― Basta adotar uma base de numeração, na qual as frações envolvidas 
não sejam dízimas. Neste caso, vou adotar a base “9”.

 

Então: [ 1/9 + 8/9 ] « na base 10 » = [ 1/10 + 8/10 ] « na base 9 » = [ 0,1 + 
0,8 = 1 ] « na base 9 » = [ 1 ] « na base 10 »

 

Ficou bem legal!

 

  _  

Albert Bouskela

 javascript:_e(%7b%7d,%20'cvml',%20'bousk...@ymail.com'); bousk...@ymail.com

 

De: owner-ob...@mat.puc-rio.br 
javascript:_e(%7b%7d,%20'cvml',%20'owner-ob...@mat.puc-rio.br');  
[mailto:owner-ob...@mat.puc-rio.br 
javascript:_e(%7b%7d,%20'cvml',%20'owner-ob...@mat.puc-rio.br'); ] Em nome de 
Luiz Antonio Rodrigues
Enviada em: quarta-feira, 14 de agosto de 2013 09:20
Para: obm-l@mat.puc-rio.br 
javascript:_e(%7b%7d,%20'cvml',%20'obm-l@mat.puc-rio.br'); 
Assunto: [obm-l] Re: Dízimas periódicas

 

Olá, Ralph!

Gostei muito do texto!

Obrigado e um abraço!

Luiz

On Tuesday, August 13, 2013, Ralph Teixeira wrote:

Oi, Luiz.

Argumento interessante? Que tal...

http://www.mat.puc-rio.br/~nicolau/olimp/obm-l.24/msg00074.html

Abraco, Ralph

On Aug 13, 2013 1:25 PM, Luiz Antonio Rodrigues rodrigue...@gmail.com wrote:

Olá, pessoal!

Tudo bem?

Na semana passada eu propus a seguinte discussão para os meus alunos: se 
0,111... + 0,888... = 0,999... então 1/9 + 8/9 = 0,999...   o que implica que 
1= 0,999... 

Consegui despertar a curiosidade dos alunos, mas muitos deles não aceitaram o 
que acabamos concluindo. Alguém poderia me ajudar com algum argumento 
interessante sobre a estranha conclusão?

Obrigado e um abraço!

Luiz


-- 
Esta mensagem foi verificada pelo sistema de antivírus e 
acredita-se estar livre de perigo. 


-- 
Esta mensagem foi verificada pelo sistema de antivírus e 
acredita-se estar livre de perigo. 


-- 
Esta mensagem foi verificada pelo sistema de antivírus e 
acredita-se estar livre de perigo. 


-- 
Esta mensagem foi verificada pelo sistema de antivírus e 
acredita-se estar livre de perigo. 


-- 
Esta mensagem foi verificada pelo sistema de antivírus e 
acredita-se estar livre de perigo. 


-- 
Esta mensagem foi verificada pelo sistema de antiv�rus e
 acredita-se estar livre de perigo.



[obm-l] Re: [obm-l] RES: [obm-l] Re: Dízimas periódicas

2013-08-15 Por tôpico regis barros
Olá Luis e aos demais.
Pensei na seguinte solução.
0,111...=1/9 então
0,999...= 9*0,111... logo
9*1/9=1 assim podemos falar que 0,999...=1

Regis



 De: Albert Bouskela bousk...@ymail.com
Para: obm-l@mat.puc-rio.br 
Enviadas: Quarta-feira, 14 de Agosto de 2013 12:57
Assunto: [obm-l] RES: [obm-l] Re: Dízimas periódicas
 


Olá a todos!
 
Há uma maneira de dar um tiro de canhão (ou jogar uma bomba atômica) para matar 
essa mosca: ― Provar que 0,999… = 1. Igual MESMO, só escrito de outra forma.
 
É simples: ― Basta adotar uma base de numeração, na qual as frações envolvidas 
não sejam dízimas. Neste caso, vou adotar a base “9”.
 
Então: [ 1/9 + 8/9 ] « na base 10 » = [ 1/10 + 8/10 ] « na base 9 » = [ 0,1 + 
0,8 = 1 ] « na base 9 » = [ 1 ] « na base 10 »
 
Ficou bem legal!
 



Albert Bouskela
bousk...@ymail.com
 
De:owner-ob...@mat.puc-rio.br [mailto:owner-ob...@mat.puc-rio.br] Em nome de 
Luiz Antonio Rodrigues
Enviada em: quarta-feira, 14 de agosto de 2013 09:20
Para: obm-l@mat.puc-rio.br
Assunto: [obm-l] Re: Dízimas periódicas
 
Olá, Ralph!
Gostei muito do texto!
Obrigado e um abraço!
Luiz

On Tuesday, August 13, 2013, Ralph Teixeira wrote:
Oi, Luiz.
Argumento interessante? Que tal...
http://www.mat.puc-rio.br/~nicolau/olimp/obm-l.24/msg00074.html
Abraco, Ralph
On Aug 13, 2013 1:25 PM, Luiz Antonio Rodrigues rodrigue...@gmail.com wrote:
Olá, pessoal!
Tudo bem?
Na semana passada eu propus a seguinte discussão para os meus alunos: se 
0,111... + 0,888... = 0,999... então 1/9 + 8/9 = 0,999...   o que implica que 
1= 0,999... 
Consegui despertar a curiosidade dos alunos, mas muitos deles não aceitaram o 
que acabamos concluindo. Alguém poderia me ajudar com algum argumento 
interessante sobre a estranha conclusão?
Obrigado e um abraço!
Luiz

-- 
Esta mensagem foi verificada pelo sistema de antivírus e 
acredita-se estar livre de perigo. 

-- 
Esta mensagem foi verificada pelo sistema de antivírus e 
acredita-se estar livre de perigo. 

-- 
Esta mensagem foi verificada pelo sistema de antivírus e 
acredita-se estar livre de perigo. 
-- 
Esta mensagem foi verificada pelo sistema de antiv�us e 
acredita-se estar livre de perigo. 
-- 
Esta mensagem foi verificada pelo sistema de antiv�rus e
 acredita-se estar livre de perigo.



[obm-l] RES: [obm-l] Re: Dízimas periódicas

2013-08-14 Por tôpico Albert Bouskela
Olá a todos!

 

Há uma maneira de dar um tiro de canhão (ou jogar uma bomba atômica) para matar 
essa mosca: ― Provar que 0,999… = 1. Igual MESMO, só escrito de outra forma.

 

É simples: ― Basta adotar uma base de numeração, na qual as frações envolvidas 
não sejam dízimas. Neste caso, vou adotar a base “9”.

 

Então: [ 1/9 + 8/9 ] « na base 10 » = [ 1/10 + 8/10 ] « na base 9 » = [ 0,1 + 
0,8 = 1 ] « na base 9 » = [ 1 ] « na base 10 »

 

Ficou bem legal!

 

  _  

Albert Bouskela

 mailto:bousk...@ymail.com bousk...@ymail.com

 

De: owner-ob...@mat.puc-rio.br [mailto:owner-ob...@mat.puc-rio.br] Em nome de 
Luiz Antonio Rodrigues
Enviada em: quarta-feira, 14 de agosto de 2013 09:20
Para: obm-l@mat.puc-rio.br
Assunto: [obm-l] Re: Dízimas periódicas

 

Olá, Ralph!

Gostei muito do texto!

Obrigado e um abraço!

Luiz

On Tuesday, August 13, 2013, Ralph Teixeira wrote:

Oi, Luiz.

Argumento interessante? Que tal...

http://www.mat.puc-rio.br/~nicolau/olimp/obm-l.24/msg00074.html

Abraco, Ralph

On Aug 13, 2013 1:25 PM, Luiz Antonio Rodrigues rodrigue...@gmail.com 
javascript:_e(%7b%7d,%20'cvml',%20'rodrigue...@gmail.com');  wrote:

Olá, pessoal!

Tudo bem?

Na semana passada eu propus a seguinte discussão para os meus alunos: se 
0,111... + 0,888... = 0,999... então 1/9 + 8/9 = 0,999...   o que implica que 
1= 0,999... 

Consegui despertar a curiosidade dos alunos, mas muitos deles não aceitaram o 
que acabamos concluindo. Alguém poderia me ajudar com algum argumento 
interessante sobre a estranha conclusão?

Obrigado e um abraço!

Luiz


-- 
Esta mensagem foi verificada pelo sistema de antivírus e 
acredita-se estar livre de perigo. 


-- 
Esta mensagem foi verificada pelo sistema de antivírus e 
acredita-se estar livre de perigo. 


-- 
Esta mensagem foi verificada pelo sistema de antivírus e 
acredita-se estar livre de perigo. 


-- 
Esta mensagem foi verificada pelo sistema de antiv�rus e
 acredita-se estar livre de perigo.



RES: [obm-l] Dois problemas legais

2013-07-10 Por tôpico Benedito
Uma sugestão para o problema 2: 

Divida o tabuleiro 10 por 10 em dois sub- tabuleiros 5 por 10. 

Com 25 movimentos ou menos você coloca todos os pares em um dos
sub-tabuleiros e no outro os ímpares.

Questão: Como resolver o problema para as duas colunas (5 e 6), do encontro
dos pares com os ímpares, em, no máximo, 10 movimentos?

Benedito

 

De: owner-ob...@mat.puc-rio.br [mailto:owner-ob...@mat.puc-rio.br] Em nome
de Henrique Rennó
Enviada em: quarta-feira, 10 de julho de 2013 13:06
Para: obm-l@mat.puc-rio.br
Assunto: Re: [obm-l] Dois problemas legais

 

Eu havia pensado que o 35 teria uma relação (e deve ter) com a quantidade de
primos máxima (cada primo seria uma soma), mas a quantidade de primos
possíveis é 45 e não 35 (desconsiderando o 2, já que não é possível
representá-lo pela soma de dois números no tabuleiro).

2013/7/10 saulo nilson saulo.nil...@gmail.com
mailto:saulo.nil...@gmail.com 

os numeros primos possiveis de se encontrar tem o valor e no maximo 199, que
contando tudo da 35 numeros entao vc tem que fazer no maximo 35 operaçoes
para nao enconrar eles.

 

2013/7/6 Benedito bened...@ufrnet.br mailto:bened...@ufrnet.br 

Problema 1

Divide-se as faces de um cubo de dimensões 9 por 9 por  9 em quadradinhos
unitários. Dispõe-se de 243 cartões na forma retangular 2 por 1, com os
quais vamos cobrir todas a superfície do cubo, sem deixar espaços livres, e
sem sobreposição de cartões. Para poder fazer isto, alguns cartões devem ser
dobrados ao meio.

Prove que a quantidade de cartões dobrados é ímpar. 

 

Problema 2

Escrevem-se os números 1,2,3,...,100  nas casas de um tabuleiro 10 por 10,
sem repetir qualquer um deles e colocando um só número em cada casa. Uma
operação permitida é escolher duas casas e trocar de posição os números que
estão escritos nelas. 

Demonstre que é possível realizar 35 operações ou menos, de maneira tal que
se consiga que para duas casas vizinhas quaisquer a soma dos números nelas
escritas seja um número composto.

OBS.: Duas casas são vizinhas se possuem um lado em comum.

  


-- 
Esta mensagem foi verificada pelo sistema de antivírus e 
acredita-se estar livre de perigo. 

 


-- 
Esta mensagem foi verificada pelo sistema de antivírus e 
acredita-se estar livre de perigo. 





 

-- 
Henrique

-- 
Esta mensagem foi verificada pelo sistema de antivírus e 
acredita-se estar livre de perigo. 


-- 
Esta mensagem foi verificada pelo sistema de antivírus e
 acredita-se estar livre de perigo.



RES: [obm-l] Problema 4 Cone Sul 1996

2013-04-30 Por tôpico EPVN
Se entendi seu argumento podemos trocar os primeiros 998 números pela média
dos primeiros 998 números. O enunciado claramente não permite essa operação.
Apenas um deles deve ser trocado pela média.
Sem querer abusar da sua bondade, poderia esclarecer esse ponto.
Abraço.
Osmundo.

-Mensagem original-
De: owner-ob...@mat.puc-rio.br [mailto:owner-ob...@mat.puc-rio.br] Em nome
de Carlos Yuzo Shine
Enviada em: terça-feira, 30 de abril de 2013 13:11
Para: obm-l@mat.puc-rio.br
Assunto: Re: [obm-l] Problema 4 Cone Sul 1996

O erro foi supor que na situação anterior os números na sequência ficariam
a,b,b,b,...,b.
 
Poderia muito bem ser, digamos, 997/998, 1, 1995/1996, 1995/1996, ...,
1995/1996.
 
Se você ainda quer pensar no problema, pare de ler aqui. Caso contrário,
continue.
 
O que você pode fazer para resolver o problema é fazer a média dos primeiros
998 números, obtendo 998 números iguais a 997/998 e depois fazer pares com
997/998 e 1 (fazendo a operação mais 998 vezes). Note que esse argumento
funciona para qualquer número composto no lugar do 1996.
 
E no caso em que trocamos 1996 por um primo p (um 0 e p-1 uns)? Aí não dá,
porque no final o denominador tem que p (todo mundo teria que ser igual a
(p-1)/p, já que a soma de todos nunca muda), e isso obrigaria a gente a, em
algum momento, dividir tudo por p, o que não é possível.
 
Mas e se a soma dos p números é múltiplo de p? Mais uma boa pergunta, não?
 
[]'s
Shine
 

From: EPVN barz...@dglnet.com.br
To: obm-l@mat.puc-rio.br 
Sent: Monday, April 22, 2013 11:57 AM
Subject: [obm-l] Problema 4 Cone Sul 1996




O enunciado é: 

A seqüência 0, 1, 1, 1,
... , 1 contém 1996 números, sendo o primeiro zero e todos os demais um. Se
escolhem
dois ou mais números da seqüência (mas não todos) e se sustitui um deles
pela
média aritmética dos números escolhidos, obtendo-se assim uma nova seqüência
de
1996 números.  
Provar que, com a
repetição desta operação, é possível obter uma seqüência na qual os 1996
números são iguais.  

NOTA: Não é necessário
escolher a mesma quantidade de números em cada operação. 

Um colega apresentou a seguinte argumentação: 

Se essa operação levasse a uma seqüência com todos os números
idênticos então no penúltimo estágio teríamos algo assim: 

a,b,b,b,..,b , com um único número diferente que
deve ser tornado igual aos demais com mais um passo. Bem, se tomarmos p
números
b e mais o número a, 
obteremos o número (a + pb)/ (p + 1 ), igualando a b teríamos
a=b. 
Parece que isso prova que esse penúltimo estágio nunca é
atingido e, portanto, o último também não. 

Se algum colega puder nos ajudar a esclarecer a situação
ficamos muito gratos. 

Um abraço. 

Osmundo Bragança.   

=
Instruções para entrar na lista, sair da lista e usar a lista em
http://www.mat.puc-rio.br/~obmlistas/obm-l.html
=


=
Instruções para entrar na lista, sair da lista e usar a lista em
http://www.mat.puc-rio.br/~obmlistas/obm-l.html
=


RES: [obm-l] Problema 4 Cone Sul 1996

2013-04-30 Por tôpico EPVN
Então há, de fato, um erro na tradução.
Isso, é claro, muda tudo.
Agora vamos trabalhar com essa versão e mais as suas perguntas.
Obrigado pela atenção.
Osmundo Bragança

-Mensagem original-
De: owner-ob...@mat.puc-rio.br [mailto:owner-ob...@mat.puc-rio.br] Em nome
de Carlos Yuzo Shine
Enviada em: terça-feira, 30 de abril de 2013 16:34
Para: obm-l@mat.puc-rio.br
Assunto: Re: [obm-l] Problema 4 Cone Sul 1996

Ah, no enunciado original trocamos cada um deles pela média aritmética
(talvez houve algum erro na hora de transcrever o problema para o site). Eu
sei porque eu fui nessa Cone Sul, e exatamente por isso eu nem li o
enunciado que foi enviado pela lista. A solução que postei foi a que dei na
prova.

[]'s
Shine


- Original Message -
From: Bernardo Freitas Paulo da Costa bernardo...@gmail.com
To: obm-l@mat.puc-rio.br
Cc: 
Sent: Tuesday, April 30, 2013 1:24 PM
Subject: Re: [obm-l] Problema 4 Cone Sul 1996

  Se escolhem
  dois ou mais números da seqüência (mas não todos) e se sustitui um deles
pela
  média aritmética dos números escolhidos, obtendo-se assim uma nova
seqüência de
  1996 números.
 
2013/4/30 Carlos Yuzo Shine cysh...@yahoo.com:
 já que a soma de todos nunca muda

Confesso que não entendi direito. Imagine que você escolhe os 4
primeiros números, 0, 1, 1, 1. Qual é o resultado da operação? Da
forma como o enunciado parece indicar, isso DEPENDE de uma OUTRA
escolha, a saber a do um deles a ser substituído por 3/4. Ou seja,
podemos ficar com 3/4, 1, 1, 1; ou 0, 3/4, 1, 1 (ou uma permutação). O
Shine parece argumentar que o resultado será 3/4, 3/4, 3/4, 3/4. Mas
isso só seria o caso se estivesse escrito .. se substitui CADA UM
deles ..., não?

-- 
Bernardo Freitas Paulo da Costa

=
Instruções para entrar na lista, sair da lista e usar a lista em
http://www.mat.puc-rio.br/~obmlistas/obm-l.html
=


=
Instruções para entrar na lista, sair da lista e usar a lista em
http://www.mat.puc-rio.br/~obmlistas/obm-l.html
=


=
Instruções para entrar na lista, sair da lista e usar a lista em
http://www.mat.puc-rio.br/~obmlistas/obm-l.html
=


[obm-l] RES: [obm-l] Re: [obm-l] sobre a resolução de problemas em geral

2013-04-21 Por tôpico Benedito
Além dos livros citados pelo Marcelo, vale a pena ler:

1)  A Matemática do Ensino Médio; Vol 4 – Elon Lages Lima e outros - SBM

2)  Techniques of Problem Solving – Steven G. Karntz – MAS

3)  2) The Art of Problem Solving – Editado por Alfred S. Posamentier – 
Corwin Press

4)  First Steps for Math Olympians – MAA

5)   The Art and Craft of Problem Solving; Second Edition – Paul Zeitz – 
Wiley

6)  The Heart of Mathematics – Edward B. Burger  Michael Starbird

7)  The Inquisitive Problem Solver – Paul Vaderlind; Richard Guy; Loren 
Larson – MAA

8)  A Decade of the Berkeley Math Circle – Zvezdeline Stankov; Tom Rike - 
AMS

 

Benedito

 

De: owner-ob...@mat.puc-rio.br [mailto:owner-ob...@mat.puc-rio.br] Em nome de 
Marcelo de Moura Costa
Enviada em: domingo, 21 de abril de 2013 08:08
Para: obm-l@mat.puc-rio.br
Assunto: [obm-l] Re: [obm-l] sobre a resolução de problemas em geral

 

Há um livro interessante: 21 aulas de Matemática Olímpica, da SBM.

Não sei se ele irá atender suas necessidades e há o famoso: A Arte de Resolver 
Problemas, do G.Polya.

Não sei se ajudei, mas é o que vem na minha memória.

Abraços

Marcelo

 

Em 21 de abril de 2013 01:13, Listeiro 037 listeiro_...@yahoo.com.br 
mailto:listeiro_...@yahoo.com.br  escreveu:


Bom dia a todos.

Tenho acompanhado discretamente há algum tempo esta lista. Pensei um
pouco antes dessa dúvida.

No momento não viso a meta desportiva/competitiva, mas aprender melhor
como seria uma demonstração adequada de uma inadequada através de
observação.

Longe de conseguir resolver qualquer questão de pronto, mas entender
alguns mecanismos de solução, o problema seria expressar melhor no pouco
ou no muito, até para adquirir maior confiança posteriormente.

Há algum texto que trabalhe essas características? Desde já agradeço.

--
Esta mensagem foi verificada pelo sistema de antivírus e
 acredita-se estar livre de perigo.


=
Instru�ões para entrar na lista, sair da lista e usar a lista em
http://www.mat.puc-rio.br/~obmlistas/obm-l.html
=

 


-- 
Esta mensagem foi verificada pelo sistema de antiv�s e 
acredita-se estar livre de perigo. 


-- 
Esta mensagem foi verificada pelo sistema de antiv�rus e
 acredita-se estar livre de perigo.



RES: [obm-l] Torneio das Cidades 94

2013-04-12 Por tôpico Benedito
Seguindo a idéia do Ponce, a resposta é 14.

 

De: owner-ob...@mat.puc-rio.br [mailto:owner-ob...@mat.puc-rio.br] Em nome de 
PONCE
Enviada em: quinta-feira, 11 de abril de 2013 18:00
Para: Jeferson Almir
Cc: obm-l@mat.puc-rio.br
Assunto: Re: [obm-l] Torneio das Cidades 94

 

Jeferson,

 

O enunciado do problema sugere que voce pense no principio da casa dos pombos.

Com isto na cabeça a prova é relativamente simples.

Qualquer duvida, entre em contato que envio um  esboço de uma prova.

PONCE.





On Qua 10/04/13 20:05 , Jeferson Almir jefersonram...@gmail.com 
mailto:jefersonram...@gmail.com  sent:

Existem 20 alunos em uma escola. Quaisquer dois deles possui um avó em comum. 
Prove que pelo menos 14 deles possui um avó em comum.

estou tentando fazer por grafos ..  alguma ajuda ou sugestão??


-- 
Esta mensagem foi verificada pelo sistema de antivrus e 
acredita-se estar livre de perigo. 



-- 
Esta mensagem foi verificada pelo sistema de antiv�s e 
acredita-se estar livre de perigo. 


-- 
Esta mensagem foi verificada pelo sistema de antiv�rus e
 acredita-se estar livre de perigo.



RES: [obm-l] Problema

2013-03-22 Por tôpico Benedito
A idéia é usar Cálculo (Coordenadas Polares). Mas, fazer na região descrita
no problema eu acho mais interessante.

Benedito

 

De: owner-ob...@mat.puc-rio.br [mailto:owner-ob...@mat.puc-rio.br] Em nome
de João Maldonado
Enviada em: sexta-feira, 22 de março de 2013 17:13
Para: obm-l@mat.puc-rio.br
Assunto: RE: [obm-l] Problema

 

Eu consegui fazer para o caso geral (M e Q pode estar em qualquer região do
círculo, não apenas em regiões opostas determinadas por um diâmetro)
E a resolução ficou bem feia também (tive que usar cálculo)

*Sendo P1 um ponto a uma distância x fixa do centro do círculo, qual a
probabilidade de escolhermos outro ponto no círculo tal que a distância
entre P1 e P2 seja menor que um?
Podemos tracejar um círculo de raio 1 em torno de P1. A intersecção desse
círculo com o círculo original é a região dos pontos cuja a distância a P1 é
 1.
A área dessa região sobre a área do círculo simboliza a probabilidade de
escolhermos outro ponto P2 no círculo tal que a distância entre P1 e P2 seja
menor que um.
A área pode ser facilmente calculada por matemática básica

A/Atotal = 1/Pi (2 ArcCos[x/2] - x sqrt (1- (x/2)²))

O peso dessa probabilidade é proporcional à área que ela ocupa (temos
muito mais pontos a uma distância 1 do que a uma distância 1/2 por exemplo)
O peso vale 2 Pi x dx/Pi = 2 x dx
Integrando de 0 a 1

P = Integral[ 2 x dx/Pi (2 ArcCos[x/2] - x sqrt (1- (x/2)²))]  de 0 a 1

P = 58.6%

[]'s
João

  _  

From: bened...@ufrnet.br mailto:bened...@ufrnet.br 
To: obm-l@mat.puc-rio.br mailto:obm-l@mat.puc-rio.br 
Subject: [obm-l] Problema
Date: Fri, 22 Mar 2013 05:16:50 -0300

Problema

Dois pontos, M e Q, são escolhidos aleatoriamente num disco unitário, mas em
regiões opostas, determinadas por um diâmetro AB. 

Qual é a probabilidade de que a distância entre M e Q seja menor do que 1?



Re: RES: [obm-l] problema

2013-02-17 Por tôpico grego
Obrigado, grande mestre!
A coisa é, de fato, violenta.
Um abraço!
Grego




 De: Albert Bouskela bousk...@msn.com
Para: obm-l@mat.puc-rio.br 
Enviadas: Sábado, 16 de Fevereiro de 2013 0:54
Assunto: RES: [obm-l] problema
 

Olá!
 
Este é um problema da Teoria dos Números bastante conhecido. Acredito (a 
confirmar!) que não exista uma solução analítica – o jeito é fazer “no braço” 
(“brute force”).
 
Bem, na Internet, encontrei a solução abaixo (bastante “arrumadinha”): – 
http://www.prise2tete.fr/forum/viewtopic.php?id=6370 
 
Bonjour,
Je n'ai trouvé qu'une méthode empirique et dichotomique aboutissant à 14 
solutions.
Supposons 1  a ≤ b ≤ c ≤ d.
Alors 1/a + 1/b + 1/c + 1/d ≤ 4/a donc 1 ≤ 4/a donc a ≤ 4 (et a  1). D'où 3 
cas à analyser : a=4, a=3 et a=2.

1) a = 4
Alors 1/4 + 1/b + 1/c + 1/d = 1 ≤ 1/4 + 3/b donc 3/4 ≤ 3/b donc b ≤ 4.
Et comme b ≥ a, on voit que b=4. De la même façon c=4 et d=4.
Solution1 : 1/4 + 1/4 + 1/4 + 1/4 = 1

2) a = 3 
Alors 1/3 + 1/b + 1/c + 1/d = 1 ≤ 1/3 + 3/b donc 2/3 ≤ 3/b donc b ≤ 9/2.
Et comme b ≥ a, on voit que b=4 ou b=3.

2a) b = 4
Alors 1/3 + 1/4 + 1/c + 1/d = 7/12 + 1/c + 1/d = 1 ≤ 7/12 + 2/c donc 5/12 ≤ 2/c 
donc c ≤ 24/5.
Et comme c ≥ b, on voit que c=4
Solution2 : 1/3 + 1/4 + 1/4 + 1/6 = 1

2b) b = 3
Alors 1/3 + 1/3 + 1/c + 1/d = 2/3 + 1/c + 1/d = 1 ≤ 2/3 + 2/c donc 1/3 ≤ 2/c 
donc c ≤ 6.
Et comme c ≥ b, on voit que c=3, c=4, c=5 ou c=6.
Solution3 : 1/3 + 1/3 + 1/4 + 1/12 = 1
Solution4 : 1/3 + 1/3 + 1/6 + 1/6 = 1

3) a = 2
Alors 1/2 + 1/b + 1/c + 1/d = 1 ≤ 1/2 + 3/b donc 1/2 ≤ 3/b donc b ≤ 6.
Et comme b ≥ a, on voit que b=6, b=5, b=4 ou b=3 (b=2 ne marche pas).

3a) b = 6
Alors 1/2 + 1/6 + 1/c + 1/d = 2/3 + 1/c + 1/d = 1 ≤ 2/3 + 2/c donc 1/3 ≤ 2/c 
donc c ≤ 6.
Et comme c ≥ b, on voit que c=6 et par suite que d=6.
Solution5 : 1/2 + 1/6 + 1/6 + 1/6 = 1

3b) b = 5
Alors 1/2 + 1/5 + 1/c + 1/d = 7/10 + 1/c + 1/d = 1 ≤ 7/10 + 2/c donc 3/10 ≤ 2/c 
donc c ≤ 20/3.
Et comme c ≥ b, on voit que c=5 ou c=6.
Si c=5, alors 1/d = 1 - 1/2 - 1/5 - 1/5 = 1/10
Solution6 : 1/2 + 1/5 + 1/5 + 1/10 = 1
Si c= 6, alors 1/d = 1 - 1/2 - 1/5 - 1/6 = 4/30 = 2/15 non réductible

3c) b = 4
Alors 1/2 + 1/4 + 1/c + 1/d = 3/4 + 1/c + 1/d = 1 ≤ 3/4 + 2/c donc 1/4 ≤ 2/c 
donc c ≤ 8.
Et comme c ≥ b, on voit que c=8, c=7, c=6 ou c=5 (c=4 ne marche pas).
Si c=8, alors 1/d = 1 - 1/2 - 1/4 - 1/8 = 1/8
Solution7 : 1/2 + 1/4 + 1/8 + 1/8 = 1
Si c=7, alors 1/d = 1 - 1/2 - 1/4 - 1/7 = 3/28 non réductible
Si c=6, alors 1/d = 1 - 1/2 - 1/4 - 1/6 = 1/12
Solution8 : 1/2 + 1/4 + 1/6 + 1/12 = 1
Si c=5, alors 1/d = 1 - 1/2 - 1/4 - 1/5 = 1/20
Solution9 : 1/2 + 1/4 + 1/5 + 1/20 = 1

3d) b = 3
Alors 1/2 + 1/3 + 1/c + 1/d = 5/6 + 1/c + 1/d = 1 ≤ 5/6 + 2/c donc 1/6 ≤ 2/c 
donc c ≤ 12.
Et comme 1/2 + 1/3 + 1/c  1 donc 1/c  1/6 donc c  6, on voit que c=12, c= 
11, c=10, c= 9, c=8 ou c=7.
Si c=12, alors 1/d = 1 - 1/2 - 1/3 - 1/12 = 1/12
Solution10 : 1/2 + 1/3 + 1/12 + 1/12 = 1
Si c=11, alors 1/d = 1 - 1/2 - 1/3 - 1/11 = 5/66 non réductible
Si c=10, alors 1/d = 1 - 1/2 - 1/3 - 1/10 = 2/30 = 1/15
Solution11 : 1/2 + 1/3 + 1/10 + 1/15 = 1
Si c=9, alors 1/d = 1 - 1/2 - 1/3 - 1/9 = 1/18
Solution12 : 1/2 + 1/3 + 1/9 + 1/18 = 1
Si c=8, alors 1/d = 1 - 1/2 - 1/3 - 1/8 = 1/24
Solution13 : 1/2 + 1/3 + 1/8 + 1/24 = 1
Si c=7, alors 1/d = 1 - 1/2 - 1/3 - 1/7 = 1/42
Solution14 : 1/2 + 1/3 + 1/7 + 1/42 = 1

FIN de la démonstration *** ouf... ***



J'ai tellement besoin de temps pour ne rien faire, qu'il ne m'en reste plus 
assez pour travailler.
 



Albert Bouskela
bousk...@msn.com
 
De:owner-ob...@mat.puc-rio.br [mailto:owner-ob...@mat.puc-rio.br] Em nome de 
grego
Enviada em: sexta-feira, 15 de fevereiro de 2013 22:31
Para: obm-l@mat.puc-rio.br
Assunto: [obm-l] problema
 
Olá, companheiros!
Um aluno me perguntou o seguinte:
a =b=c=d
1/a+1/b+1/c+1/d=1
Quantas quádruplas ordenadas (a, b, c, d) de naturais satisfazem a igualdade?
Um abraço!
Grego

[obm-l] RES: [obm-l] Último Teorema de Fermat

2013-02-17 Por tôpico Albert Bouskela
Olá, João!

 

Sim, eu tenho! Acho, até, que já a coloquei aqui, na Lista – vou procurar e
enviá-la.

 

Abraço,

  _  

Albert Bouskela

 mailto:bousk...@msn.com bousk...@msn.com

 

De: owner-ob...@mat.puc-rio.br [mailto:owner-ob...@mat.puc-rio.br] Em nome
de João Maldonado
Enviada em: sábado, 16 de fevereiro de 2013 14:00
Para: obm-l@mat.puc-rio.br
Assunto: [obm-l] Último Teorema de Fermat

 

Alguém tem uma prova (em inglês ou português) para o caso n=3 do último
teorema de fermat? 



[obm-l] RES: [obm-l] Último Teorema de Fermat

2013-02-17 Por tôpico Albert Bouskela
Olá!

 

Encontrei! Está em meu post de 21ABR2009 « Assunto: [obm-l] RE: [obm-l] Uma
demonstração interessante – Equação do 3º grau e o Último Teorema de Fermat
»

 

Para: obm-l@mat.puc-rio.br
Data: Terça-feira, 21 de Abril de 2009, 17:35
 
Olá! 
 
  
 
Lamento
não ter respondido antes... 
 
  
 
Felizmente,
o caso particular  x^3 + y^3 = z^3  do chamado Último Teorema de Fermat é
muito simples. 
 
  
 
Veja, por exemplo, o item 10.1 - El caso p=3 no livro Teoría de Números do
Carlos Ivorra Castillo (  http://www.uv.es/ivorra/Libros/Numeros.pdf
http://www.uv.es/ivorra/Libros/Numeros.pdf ).

 

  _  

Albert Bouskela

 mailto:bousk...@msn.com bousk...@msn.com

 

De: owner-ob...@mat.puc-rio.br [mailto:owner-ob...@mat.puc-rio.br] Em nome
de João Maldonado
Enviada em: sábado, 16 de fevereiro de 2013 14:00
Para: obm-l@mat.puc-rio.br
Assunto: [obm-l] Último Teorema de Fermat

 

Alguém tem uma prova (em inglês ou português) para o caso n=3 do último
teorema de fermat? 



RES: [obm-l] problema

2013-02-15 Por tôpico Albert Bouskela
Olá!

 

Este é um problema da Teoria dos Números bastante conhecido. Acredito (a 
confirmar!) que não exista uma solução analítica – o jeito é fazer “no braço” 
(“brute force”).

 

Bem, na Internet, encontrei a solução abaixo (bastante “arrumadinha”): – 
http://www.prise2tete.fr/forum/viewtopic.php?id=6370 

 

Bonjour,
Je n'ai trouvé qu'une méthode empirique et dichotomique aboutissant à 14 
solutions.
Supposons 1  a ≤ b ≤ c ≤ d.
Alors 1/a + 1/b + 1/c + 1/d ≤ 4/a donc 1 ≤ 4/a donc a ≤ 4 (et a  1). D'où 3 
cas à analyser : a=4, a=3 et a=2.

1) a = 4
Alors 1/4 + 1/b + 1/c + 1/d = 1 ≤ 1/4 + 3/b donc 3/4 ≤ 3/b donc b ≤ 4.
Et comme b ≥ a, on voit que b=4. De la même façon c=4 et d=4.
Solution1 : 1/4 + 1/4 + 1/4 + 1/4 = 1

2) a = 3 
Alors 1/3 + 1/b + 1/c + 1/d = 1 ≤ 1/3 + 3/b donc 2/3 ≤ 3/b donc b ≤ 9/2.
Et comme b ≥ a, on voit que b=4 ou b=3.

2a) b = 4
Alors 1/3 + 1/4 + 1/c + 1/d = 7/12 + 1/c + 1/d = 1 ≤ 7/12 + 2/c donc 5/12 ≤ 2/c 
donc c ≤ 24/5.
Et comme c ≥ b, on voit que c=4
Solution2 : 1/3 + 1/4 + 1/4 + 1/6 = 1

2b) b = 3
Alors 1/3 + 1/3 + 1/c + 1/d = 2/3 + 1/c + 1/d = 1 ≤ 2/3 + 2/c donc 1/3 ≤ 2/c 
donc c ≤ 6.
Et comme c ≥ b, on voit que c=3, c=4, c=5 ou c=6.
Solution3 : 1/3 + 1/3 + 1/4 + 1/12 = 1
Solution4 : 1/3 + 1/3 + 1/6 + 1/6 = 1

3) a = 2
Alors 1/2 + 1/b + 1/c + 1/d = 1 ≤ 1/2 + 3/b donc 1/2 ≤ 3/b donc b ≤ 6.
Et comme b ≥ a, on voit que b=6, b=5, b=4 ou b=3 (b=2 ne marche pas).

3a) b = 6
Alors 1/2 + 1/6 + 1/c + 1/d = 2/3 + 1/c + 1/d = 1 ≤ 2/3 + 2/c donc 1/3 ≤ 2/c 
donc c ≤ 6.
Et comme c ≥ b, on voit que c=6 et par suite que d=6.
Solution5 : 1/2 + 1/6 + 1/6 + 1/6 = 1

3b) b = 5
Alors 1/2 + 1/5 + 1/c + 1/d = 7/10 + 1/c + 1/d = 1 ≤ 7/10 + 2/c donc 3/10 ≤ 2/c 
donc c ≤ 20/3.
Et comme c ≥ b, on voit que c=5 ou c=6.
Si c=5, alors 1/d = 1 - 1/2 - 1/5 - 1/5 = 1/10
Solution6 : 1/2 + 1/5 + 1/5 + 1/10 = 1
Si c= 6, alors 1/d = 1 - 1/2 - 1/5 - 1/6 = 4/30 = 2/15 non réductible

3c) b = 4
Alors 1/2 + 1/4 + 1/c + 1/d = 3/4 + 1/c + 1/d = 1 ≤ 3/4 + 2/c donc 1/4 ≤ 2/c 
donc c ≤ 8.
Et comme c ≥ b, on voit que c=8, c=7, c=6 ou c=5 (c=4 ne marche pas).
Si c=8, alors 1/d = 1 - 1/2 - 1/4 - 1/8 = 1/8
Solution7 : 1/2 + 1/4 + 1/8 + 1/8 = 1
Si c=7, alors 1/d = 1 - 1/2 - 1/4 - 1/7 = 3/28 non réductible
Si c=6, alors 1/d = 1 - 1/2 - 1/4 - 1/6 = 1/12
Solution8 : 1/2 + 1/4 + 1/6 + 1/12 = 1
Si c=5, alors 1/d = 1 - 1/2 - 1/4 - 1/5 = 1/20
Solution9 : 1/2 + 1/4 + 1/5 + 1/20 = 1

3d) b = 3
Alors 1/2 + 1/3 + 1/c + 1/d = 5/6 + 1/c + 1/d = 1 ≤ 5/6 + 2/c donc 1/6 ≤ 2/c 
donc c ≤ 12.
Et comme 1/2 + 1/3 + 1/c  1 donc 1/c  1/6 donc c  6, on voit que c=12, c= 
11, c=10, c= 9, c=8 ou c=7.
Si c=12, alors 1/d = 1 - 1/2 - 1/3 - 1/12 = 1/12
Solution10 : 1/2 + 1/3 + 1/12 + 1/12 = 1
Si c=11, alors 1/d = 1 - 1/2 - 1/3 - 1/11 = 5/66 non réductible
Si c=10, alors 1/d = 1 - 1/2 - 1/3 - 1/10 = 2/30 = 1/15
Solution11 : 1/2 + 1/3 + 1/10 + 1/15 = 1
Si c=9, alors 1/d = 1 - 1/2 - 1/3 - 1/9 = 1/18
Solution12 : 1/2 + 1/3 + 1/9 + 1/18 = 1
Si c=8, alors 1/d = 1 - 1/2 - 1/3 - 1/8 = 1/24
Solution13 : 1/2 + 1/3 + 1/8 + 1/24 = 1
Si c=7, alors 1/d = 1 - 1/2 - 1/3 - 1/7 = 1/42
Solution14 : 1/2 + 1/3 + 1/7 + 1/42 = 1

FIN de la démonstration *** ouf... ***

  _  

J'ai tellement besoin de temps pour ne rien faire, qu'il ne m'en reste plus 
assez pour travailler.

 

  _  

Albert Bouskela

 mailto:bousk...@msn.com bousk...@msn.com

 

De: owner-ob...@mat.puc-rio.br [mailto:owner-ob...@mat.puc-rio.br] Em nome de 
grego
Enviada em: sexta-feira, 15 de fevereiro de 2013 22:31
Para: obm-l@mat.puc-rio.br
Assunto: [obm-l] problema

 

Olá, companheiros!

Um aluno me perguntou o seguinte:

a =b=c=d

1/a+1/b+1/c+1/d=1

Quantas quádruplas ordenadas (a, b, c, d) de naturais satisfazem a igualdade?

Um abraço!

Grego



[obm-l] RES: [obm-l] Re: [obm-l] Existe um único k

2013-01-18 Por tôpico Albert Bouskela
Olá!

 

Repare que a abordagem do Artur demonstra a unicidade para R, logo, a 
demonstração para Z (um subconjunto de R) é automática.

 

A respeito da sua questão: Se x.y=0, então “x” ou “y” é igual a zero, repare 
que:

 

a+0=a Isto pela própria definição de zero, i.e., não é passível de demonstração.

Logo: a-a=0 daí a(1-1)=0 daí a.0=0 

 

Sds.,

  _  

Albert Bouskela

 mailto:bousk...@msn.com bousk...@msn.com

 

De: owner-ob...@mat.puc-rio.br [mailto:owner-ob...@mat.puc-rio.br] Em nome de 
ennius
Enviada em: quinta-feira, 17 de janeiro de 2013 17:20
Para: obm-l@mat.puc-rio.br
Assunto: [obm-l] Re: [obm-l] Existe um único k

 

Caro Artur,
A questão trata somente de números inteiros. No universo Z, precisamos então 
mostrar que a igualdade
x.y = 0 implica x = 0 ou y = 0.
Gostaria de ajuda nesse ponto.
Ennius Lima
___

 

  _  

Em 16/01/2013 21:24, Artur Costa Steiner   mailto:steinerar...@gmail.com 
steinerar...@gmail.com  escreveu:
Na realidade, se a e b  0 são reais quaisquer, então existe um único c tal 
que a = bc. De fato, se c e cd satisfazem a esta condição, então 

a = cb
a = db
0 = cb - db = (c -d)b 

Como os reais formam um corpo, logo um anel de integridade, com relação à soma 
e a multiplicação, segue-se, como b não é nulo, que c - d = 0 e que c = d.



Artur Costa Steiner

Em 16/01/2013, às 19:44, Pedro Chaves brped...@hotmail.com escreveu:

 
 Amigos da Lista,
 
 Usando-se a definição:  Um número inteiro d (diferente de zero) é divisor de 
 um inteiro n, quando existe um inteiro k, tal que
 n = kd, como provar que o número k, quando existe, é único?
 
 Desde já, muito obrigado pela atenção.
 Pedro Chaves
 
 _ 
 =
 Instruções para entrar na lista, sair da lista e usar a lista em
 http://www.mat.puc-rio.br/~obmlistas/obm-l.html
 =

=
Instru��es para entrar na lista, sair da lista e usar a lista em
http://www.mat.puc-rio.br/~obmlistas/obm-l.html
=

= 
Instru絥s para entrar na lista, sair da lista e usar a lista em 
http://www.mat.puc-rio.br/~obmlistas/obm-l.html 
= 



RES: [obm-l] link?

2012-11-26 Por tôpico Albert Bouskela
A lista da OBM está hospedada no seguinte link:

 

http://www.mail-archive.com/obm-l@mat.puc-rio.br/ 

 

  _  

Albert Bouskela

bousk...@msn.com

 

De: owner-ob...@mat.puc-rio.br [mailto:owner-ob...@mat.puc-rio.br] Em nome
de Hermann
Enviada em: segunda-feira, 26 de novembro de 2012 08:57
Para: obm-l@mat.puc-rio.br
Assunto: [obm-l] link?

 

Não consigo acessar essa página, alguém sabe qual o novo link?

 

http://www.mat.puc-rio.br/~obmlistas/obm-l.html



RES: [obm-l] Pergunta Boba

2012-10-15 Por tôpico bouskela
Olá!

 

Bem, primeiro é necessário verificar que (-a)(b) = -(a)(b) = -(ab) e,
depois, fica fácil verificar que (-a)(-b) = (a)(b) = (ab).

 

(-a)(b) =

   (-a)(b) + (a)(b) – (a)(b) =

   (b)[(-a)+(a)] – (a)(b) =

   (b)[0] – (a)(b) =

   -(a)(b) 

 

 

(-a)(-b) =

   (-a)(-b) + (a)(b) – (a)(b) =

   (-a)(-b) + (a)(b) + (-a)(b) =

   (-a)[(-b)+(b)] + (a)(b) =

   (-a)[0] + (a)(b) = 

   (a)(b)   

 

 

Albert Bouskela

 mailto:bousk...@gmail.com bousk...@gmail.com

 

De: owner-ob...@mat.puc-rio.br [mailto:owner-ob...@mat.puc-rio.br] Em nome
de luiz silva
Enviada em: segunda-feira, 15 de outubro de 2012 12:08
Para: Matematica Lista
Assunto: [obm-l] Pergunta Boba

 

Pessoal,

 

Pq a multiplicação de dois números negativos dá um número positivo?
Lembro-me de, a muito tempo atras, um professor usar o seguinte argumento:
se eu nego uma negação, estou fazendo uma afirmação!

 

O argumento até faz sentido, mas afirmação está longe de ser um numero
positivo, assim como negação está longe de ser um número negativo.Existe uma
demonstração para isso ?

 

Abs

Felipe



[obm-l] RES: [obm-l] Recíproca do teorema de Pitot

2012-10-02 Por tôpico bouskela
Olá!

Sim, a recíproca do Teorema de Pitot foi demonstrada por Steiner em 1850
(aproximadamente).

No seguinte artigo [ http://jl.ayme.pagesperso-orange.fr/Docs/Demir.pdf ]
você pode encontrar a prova do próprio teorema e da sua recíproca.

Albert Bouskela
bousk...@gmail.com


-Mensagem original-
De: owner-ob...@mat.puc-rio.br [mailto:owner-ob...@mat.puc-rio.br] Em nome
de Pedro Chaves
Enviada em: terça-feira, 2 de outubro de 2012 16:25
Para: obm-l@mat.puc-rio.br
Assunto: [obm-l] Recíproca do teorema de Pitot


Amigos da lista,

Gostaria de saber se vale a recíproca do Teorema de Pitot, abaixo enunciado.

Teorema de Pitot:  Em todo quadrilátero circunscritível, a soma de dois
lados opostos é igual à soma dos outros dois.


Muito obrigado!
Pedro Chaves

  
=
Instruções para entrar na lista, sair da lista e usar a lista em
http://www.mat.puc-rio.br/~obmlistas/obm-l.html
=


=
Instruções para entrar na lista, sair da lista e usar a lista em
http://www.mat.puc-rio.br/~obmlistas/obm-l.html
=


[obm-l] RES: [obm-l] Re: [obm-l] Análise Combinatória

2012-09-16 Por tôpico Osmundo Bragança
Muitíssimo obrigado caro Ralph.

Esta lista continua utilíssima para muitos professores.

Um abraço.

Osmundo.

 

  _  

De: owner-ob...@mat.puc-rio.br [mailto:owner-ob...@mat.puc-rio.br] Em nome
de Ralph Teixeira
Enviada em: domingo, 16 de setembro de 2012 12:22
Para: obm-l@mat.puc-rio.br
Assunto: [obm-l] Re: [obm-l] Análise Combinatória

 

Ah, errei uma bobagem. Era:

 

R(a,b,c)=R(a,c,b)=B(b,a,c)=B(c,a,b)=U(b,c,a)=U(c,b,a)

 

a chave eh que o numero a tem que ficar na mesma posicao relativa em cada
funcao. Mas dali para frente, estah correto assim mesmo.

 

Abraco,

  Ralph

2012/9/16 Ralph Teixeira ralp...@gmail.com

Certamente nao eh a segunda resposta... :)

 

Digo, para arrumar as nacionalidades, voce tem 3 opcoes para o primeiro, 2
para o segundo, etc., para um total de 3.2^8=768 possibilidades.

 

Mas isto estah errado, eh claro -- muitas dessas escolhas sao impossiveis,
como por exemplo RBRBRBRUR, que teria 5 russos -- nao vale.

 

Entao estou dizendo que sao MENOS que 768 possibilidades para a ordenacao
das nacionalidades. Portanto, sao menos que 768.3!.3!.3! filas (permutando
os individuos dentro de cada nacionalidade).

 

Nao estou resolvendo o problema, mas sei que a resposta eh (bem!) menos que
768.6^3=165888. Faltou exclusao na inclusao-exclusao. :) :) :)

 

Abraco,

   Ralph

 

P.S.: Vou resolver o problema de um jeito computacional feio. Faco isso para
mostrar que aas vezes vale a pena botar um pouco de algebra, fazer tudo
ficar mecanico, e mandar brasa!

R(a,b,c)=numero de filas COMECANDO COM UM RUSSO que tem a russos, b
bielorussos e c ucranianos, contando soh nacionalidades, sem ter
nacionalidades consecutivas

B(a,b,c)=numero de filas COMECANDO COM UM BIELO etc etc

U(a,b,c)=comecando com UCRANIANO

Por outro lado, por simetria,
R(a,b,c)=R(a,c,b)=B(b,a,c)=B(b,c,a)=U(c,a,b)=U(c,b,a), certo?

 

Entao, uma fila comecando por R tem que continuar com B ou com U, usando um
russo a menos:

R(a,b,c)=B(a-1,b,c)+U(a-1,b,c)=R(b,a-1,c)+R(c,b,a-1)

Esta recorrencia nao eh das piores se os numeros forem pequenos! Com
coragem, isto mata o problema:

R(3,3,3)=R(3,2,3)+R(3,3,2)=2R(3,3,2)=2.(R(3,2,2)+R(2,3,2))=

=2.(2.R(2,2,2)+R(3,1,2)+R(2,3,1))=2.(4.R(2,2,1)+R(1,2,2)+R(2,1,2)+R(3,1,1)+R
(1,3,1))=

=2.(R(3,1,1)+5.R(2,2,1)+R(1,3,1)+R(1,2,2))

 

Agora que soh tem 5 fulanos na fila, acho que jah dah para calcular cada um
pensando direto:

R(3,1,1)=2 porque soh tem 3 lugares para por os russos nas 5 posicoes. Entao
eh RURBR ou RBRUR.

R(2,2,1)=R(2,1,1)+R(1,2,1)=4+1=5 (4=permutacoes de RBU sem comecar por R;
1=RBUB).

R(1,3,1)=0 (haveria dois B consecutivos!)

R(1,2,2)=2 (RBUBU ou RUBUB, soh)

Entao R(3,3,3)=2.(2+25+2)=58

 

O que queremos eh R(3,3,3)+U(3,3,3)+B(3,3,3)=3.58=174

 

Minto, o que REALMENTE queremos eh isso vezes 3!.3!.3!. Eh, concordo com a
primeira resposta.

 

2012/9/16 Osmundo Bragança barz...@dglnet.com.br

Caros colegas solicito ajuda na resolução do seguinte problema:

Três russos, três biolerussos e três ucranianos vão ser organizados em uma
fila.

Determine quantas filas existem que não contêm dois conterrâneos em posição
consecutiva.

Dois colegas apresentaram resolução, um encontrou, para resposta,
174x3!x3!x3!=37.584,

outro colega chegou a:283 824 (via o Princípio da Inclusão-Exclusão)

Qualquer ajuda será muito útil.

Obrigado.

Osmundo Bragança

 

 



[obm-l] RES: [obm-l] Análise Combinatória

2012-09-16 Por tôpico Osmundo Bragança
Muitíssimo obrigado caro Douglas Oliveira.

Um abraço do colega

Osmundo Bragança.

 

  _  

De: owner-ob...@mat.puc-rio.br [mailto:owner-ob...@mat.puc-rio.br] Em nome
de douglas.olive...@grupoolimpo.com.br
Enviada em: domingo, 16 de setembro de 2012 12:33
Para: obm-l@mat.puc-rio.br
Assunto: Re: [obm-l] Análise Combinatória

 

Bom podemos fazer por inclusão e exclusão sim , mas acho que fica um pouco
grande olha:

Vamos considerar que sejam AAABBBCCC e façamos todos os anagramas onde nao
existam letras iguais juntas e ao final multiplicaremos por 3!x3!x3!.

Vamos contar todas as permutações que possuem dois AA juntos , que é só
considerar que os dois AA sejam um único bloco. aí dará

8!/3!x3!=1120 e tirar os casos em que aparecem 3 A's juntos o tipo AAA que
do mesmo jeito fica 7!/3!x3!=140

1120-140=980.

Vamos contar aquelas em que aparecem dois A e dois B juntos contaremos
(AA)A(BB)BCCC 7!/3!=840, e retiramos aqueles em que aparecem AAA e BB que
dará 6!/3!=120 e depois retiramos aqueles em que aparecem AA e BBB que
tambem dará 120 e acrescentaremos aqueles em que aparecem AAA e BBB que é
(AAA)(BBB)CCC e dará 5!/3!=20 , assim dará 840-2x120+20=620.

Agora amos ao procedimento final contar quantos anagramas aparecem AA BB e
CC (AA)(BB)(CC)ABC que dará  6! e precisamos retirar

os que ocorrem AAA ,BB e CC e depois o mesmo para AA,BBB e CC e o mesmo para
AA , BB e CCC 5!x3=360 assim fica 720-360=360, porém precisamos colocar os
que aparecem AA, BBB, e CCC, o mesmo para AAA,BBB e CC  e os que contém AAA,
BB e CCC que dará 4!x3=72 entao fica 360+72=432 e finalizando precisamos
retirar aqueles em que aparecem AAA, BBB e CCC que é 3!=6 assim resultado dá
432-6=426.

 

Agora podemos finalizar o problema fazendo 3x980-3x620+426=1506, e retiramos
de todos os anagrams possíveis que são 9!/3!x3!x3!=1680, resposta final será
1680-1506=174 ou seja 174x(3!)^3=37584 modos distintos.Um dos seus colegas
acertou o resultado

Valeu, um abraço do

Douglas Oliveira!! 

On Sun, 16 Sep 2012 10:08:40 -0300, Osmundo Bragança wrote:

Caros colegas solicito ajuda na resolução do seguinte problema:

Três russos, três biolerussos e três ucranianos vão ser organizados em uma
fila.

Determine quantas filas existem que não contêm dois conterrâneos em posição
consecutiva.

Dois colegas apresentaram resolução, um encontrou, para resposta,
174x3!x3!x3!=37.584,

outro colega chegou a:283 824 (via o Princípio da Inclusão-Exclusão)

Qualquer ajuda será muito útil.

Obrigado.

Osmundo Bragança

 

 



[obm-l] RES: [obm-l] Fwd: [obm-l] Solução única

2012-08-30 Por tôpico Judah Ben Hur
Apareceu sim, só falta o desafiante enviar o numerário para você.

Judah

 

  _  

De: owner-ob...@mat.puc-rio.br [mailto:owner-ob...@mat.puc-rio.br] Em nome
de Ralph Teixeira
Enviada em: quarta-feira, 29 de agosto de 2012 10:34
Para: obm-l@mat.puc-rio.br
Assunto: [obm-l] Fwd: [obm-l] Solução única

 

Nao sei porque esta mensagem nao apareceu na lista Estou tentando de
novo, para ver se ganho os R$50.

-- Forwarded message --
From: Ralph Teixeira ralp...@gmail.com
Date: 2012/8/28
Subject: Re: [obm-l] Solução única
To: obm-l@mat.puc-rio.br



Hmmm Veja se voce conhece este fato:

 

FATO: f(x)=(1+r/x)^x eh uma funcao crescente (para x=1) que tende para e^r
quando x-+Inf. Ou seja, (1+r/x)^x  e^r sempre que x=1.

 

Agora sim!

 

i) Nao ha solucao com ba=3. De fato, escrevendo b=a+r, vem:

 

a^b-b^a = a^a.(a^r-(1+r/a)^a)  a^a.(a^r-e^r)  a^a.(a-e)  3^3.(0.2)  1

 

onde usei que r=1 para sumir com r (note que
(a^r-e^r)=(a-e)(a^(r-1)+a^(r-2)+...+e^(r-1)) e este termo imenso eh =1), e
que a=3 e que e=2.781828... no finzinho. 

 

ii) Nao ha solucao com 3=ba. De fato, escrevendo a=b+r, vem:

 

a^b-b^a = b^b.((1+r/b)^b-b^r)  b^b.(e^r-b^r)  0 pois be e r=1.

 

Como obviamente tambem nao ha solucao com a=b, as unicas possiveis solucoes
tem pelo menos um dos numeros menores do que 3. Agora eh soh analisar os
casos a=1, a=2, b=1 e b=2:

 

-- Se a=1 entao 1-b=1, nao presta.

-- Se a=2, entao 2^b-b^2=1, isto eh, 2^b=b^2+1. Entao b eh impar, digamos,
b=2k+1, entao 2^b=4k^2+4k+2 nao eh divisivel por 4, entao b=1. De fato
(a,b)=(2,1) serve.

-- Se b=1, entao a-1=1, isto eh, a=2, o que jah vimos acima.

-- Se b=2, entao a^2-2^a=1, isto eh, 2^a=a^2-1=(a+1)(a-1). Entao ambos a-1 e
a+1 tem que ser potencias de 2 (cuja diferenca eh 2!), ou seja, a-1=2 e
a+1=4. Assim, a=3.

 

Cade meus 50 reais? ;)

 

Abraco,

  Ralph

 

2012/8/28 João Maldonado joao_maldona...@hotmail.com

Meu amigo me passou um desafio anteontem, falou que se eu resolvesse até
ontem a meia-noite, ele me dava 50 reais.
Acontece que por mais insistente que eu tenha sido não saiu muita coisa :)

A aposta já acabou e ele também não sabe a resolução, e eu quero muito saber
como se resolve isso!
Se alguém puder me dar uma ajuda eu agradeço
 
Prove que a^b - b^a = 1  admite única e exclusivamente a solução (3, 2),
para a e b naturais maiores de 0.
 
 
[]'s
João
 

 

 



[obm-l] RES: [obm-l] Re: [obm-l] Fwd: [obm-l] Solução única

2012-08-30 Por tôpico bouskela
Olá!

 

Sim, veja a minha mensagem « A Conjectura de Catalan ».

 

Albert Bouskela

 mailto:bousk...@gmail.com bousk...@gmail.com

 

De: owner-ob...@mat.puc-rio.br [mailto:owner-ob...@mat.puc-rio.br] Em nome
de luiz silva
Enviada em: quinta-feira, 30 de agosto de 2012 09:34
Para: obm-l@mat.puc-rio.br
Assunto: [obm-l] Re: [obm-l] Fwd: [obm-l] Solução única

 

Este problema não é um caso específico do problema que ficou 150 anos para
ser resolvido (esqueci o nome dele, mas acho que enviaram para cá) ?

 

Mas se não me engano, o enunciado geral era 

 

Mostrar que a única solução inteira para equação x^p - y^q = 1 é  (x=3, p=2,
y=2, q=3).

 

Abs

Felipe

 

 

 

De: Ralph Teixeira ralp...@gmail.com
Para: obm-l@mat.puc-rio.br 
Enviadas: Quarta-feira, 29 de Agosto de 2012 10:33
Assunto: [obm-l] Fwd: [obm-l] Solução única

 

Nao sei porque esta mensagem nao apareceu na lista Estou tentando de
novo, para ver se ganho os R$50.

-- Forwarded message --
From: Ralph Teixeira ralp...@gmail.com
Date: 2012/8/28
Subject: Re: [obm-l] Solução única
To: obm-l@mat.puc-rio.br



Hmmm Veja se voce conhece este fato:

 

FATO: f(x)=(1+r/x)^x eh uma funcao crescente (para x=1) que tende para e^r
quando x-+Inf. Ou seja, (1+r/x)^x  e^r sempre que x=1.

 

Agora sim!

 

i) Nao ha solucao com ba=3. De fato, escrevendo b=a+r, vem:

 

a^b-b^a = a^a.(a^r-(1+r/a)^a)  a^a.(a^r-e^r)  a^a.(a-e)  3^3.(0.2)  1

 

onde usei que r=1 para sumir com r (note que
(a^r-e^r)=(a-e)(a^(r-1)+a^(r-2)+...+e^(r-1)) e este termo imenso eh =1), e
que a=3 e que e=2.781828... no finzinho. 

 

ii) Nao ha solucao com 3=ba. De fato, escrevendo a=b+r, vem:

 

a^b-b^a = b^b.((1+r/b)^b-b^r)  b^b.(e^r-b^r)  0 pois be e r=1.

 

Como obviamente tambem nao ha solucao com a=b, as unicas possiveis solucoes
tem pelo menos um dos numeros menores do que 3. Agora eh soh analisar os
casos a=1, a=2, b=1 e b=2:

 

-- Se a=1 entao 1-b=1, nao presta.

-- Se a=2, entao 2^b-b^2=1, isto eh, 2^b=b^2+1. Entao b eh impar, digamos,
b=2k+1, entao 2^b=4k^2+4k+2 nao eh divisivel por 4, entao b=1. De fato
(a,b)=(2,1) serve.

-- Se b=1, entao a-1=1, isto eh, a=2, o que jah vimos acima.

-- Se b=2, entao a^2-2^a=1, isto eh, 2^a=a^2-1=(a+1)(a-1). Entao ambos a-1 e
a+1 tem que ser potencias de 2 (cuja diferenca eh 2!), ou seja, a-1=2 e
a+1=4. Assim, a=3.

 

Cade meus 50 reais? ;)

 

Abraco,

  Ralph

 

2012/8/28 João Maldonado joao_maldona...@hotmail.com

Meu amigo me passou um desafio anteontem, falou que se eu resolvesse até
ontem a meia-noite, ele me dava 50 reais.
Acontece que por mais insistente que eu tenha sido não saiu muita coisa :)

A aposta já acabou e ele também não sabe a resolução, e eu quero muito saber
como se resolve isso!
Se alguém puder me dar uma ajuda eu agradeço
 
Prove que a^b - b^a = 1  admite única e exclusivamente a solução (3, 2),
para a e b naturais maiores de 0.
 
 
[]'s
João
 

 

 

 



[obm-l] RES: [obm-l] Fwd: [obm-l] Solução única

2012-08-29 Por tôpico bouskela
Nananinanão!!!

 

Eu resolvi primeiro! Mandei ver Catalan e pronto! Reconheço que é tiro de
canhão pra matar mosca, mas as 50 pratas são minhas!

 

Ralph, não me leve a mal, é que eu tô precisando dessa grana, pô!

 

Albert Bouskela

 mailto:bousk...@gmail.com bousk...@gmail.com

 

De: owner-ob...@mat.puc-rio.br [mailto:owner-ob...@mat.puc-rio.br] Em nome
de Ralph Teixeira
Enviada em: quarta-feira, 29 de agosto de 2012 10:34
Para: obm-l@mat.puc-rio.br
Assunto: [obm-l] Fwd: [obm-l] Solução única

 

Nao sei porque esta mensagem nao apareceu na lista Estou tentando de
novo, para ver se ganho os R$50.

-- Forwarded message --
From: Ralph Teixeira ralp...@gmail.com
Date: 2012/8/28
Subject: Re: [obm-l] Solução única
To: obm-l@mat.puc-rio.br



Hmmm Veja se voce conhece este fato:

 

FATO: f(x)=(1+r/x)^x eh uma funcao crescente (para x=1) que tende para e^r
quando x-+Inf. Ou seja, (1+r/x)^x  e^r sempre que x=1.

 

Agora sim!

 

i) Nao ha solucao com ba=3. De fato, escrevendo b=a+r, vem:

 

a^b-b^a = a^a.(a^r-(1+r/a)^a)  a^a.(a^r-e^r)  a^a.(a-e)  3^3.(0.2)  1

 

onde usei que r=1 para sumir com r (note que
(a^r-e^r)=(a-e)(a^(r-1)+a^(r-2)+...+e^(r-1)) e este termo imenso eh =1), e
que a=3 e que e=2.781828... no finzinho. 

 

ii) Nao ha solucao com 3=ba. De fato, escrevendo a=b+r, vem:

 

a^b-b^a = b^b.((1+r/b)^b-b^r)  b^b.(e^r-b^r)  0 pois be e r=1.

 

Como obviamente tambem nao ha solucao com a=b, as unicas possiveis solucoes
tem pelo menos um dos numeros menores do que 3. Agora eh soh analisar os
casos a=1, a=2, b=1 e b=2:

 

-- Se a=1 entao 1-b=1, nao presta.

-- Se a=2, entao 2^b-b^2=1, isto eh, 2^b=b^2+1. Entao b eh impar, digamos,
b=2k+1, entao 2^b=4k^2+4k+2 nao eh divisivel por 4, entao b=1. De fato
(a,b)=(2,1) serve.

-- Se b=1, entao a-1=1, isto eh, a=2, o que jah vimos acima.

-- Se b=2, entao a^2-2^a=1, isto eh, 2^a=a^2-1=(a+1)(a-1). Entao ambos a-1 e
a+1 tem que ser potencias de 2 (cuja diferenca eh 2!), ou seja, a-1=2 e
a+1=4. Assim, a=3.

 

Cade meus 50 reais? ;)

 

Abraco,

  Ralph

 

2012/8/28 João Maldonado joao_maldona...@hotmail.com

Meu amigo me passou um desafio anteontem, falou que se eu resolvesse até
ontem a meia-noite, ele me dava 50 reais.
Acontece que por mais insistente que eu tenha sido não saiu muita coisa :)

A aposta já acabou e ele também não sabe a resolução, e eu quero muito saber
como se resolve isso!
Se alguém puder me dar uma ajuda eu agradeço
 
Prove que a^b - b^a = 1  admite única e exclusivamente a solução (3, 2),
para a e b naturais maiores de 0.
 
 
[]'s
João
 

 

 



[obm-l] Re: [obm-l] RES: [obm-l] Fwd: [obm-l] Solução única

2012-08-29 Por tôpico Ralph Teixeira
30 p/ mim, 20 p/ voce, e nao se fala mais nisso. ;) ;) ;)
2012/8/29 bousk...@gmail.com

 Nananinanão!!!

 ** **

 Eu resolvi primeiro! Mandei ver Catalan e pronto! Reconheço que é tiro de
 canhão pra matar mosca, mas as 50 pratas são minhas!

 ** **

 Ralph, não me leve a mal, é que eu tô precisando dessa grana, pô!

 ** **

 *Albert Bouskela*

 bousk...@gmail.com

 ** **

 *De:* owner-ob...@mat.puc-rio.br [mailto:owner-ob...@mat.puc-rio.br] *Em
 nome de *Ralph Teixeira
 *Enviada em:* quarta-feira, 29 de agosto de 2012 10:34
 *Para:* obm-l@mat.puc-rio.br
 *Assunto:* [obm-l] Fwd: [obm-l] Solução única

 ** **

 Nao sei porque esta mensagem nao apareceu na lista Estou tentando de
 novo, para ver se ganho os R$50.

 -- Forwarded message --
 From: *Ralph Teixeira* ralp...@gmail.com
 Date: 2012/8/28
 Subject: Re: [obm-l] Solução única
 To: obm-l@mat.puc-rio.br

 

 Hmmm Veja se voce conhece este fato:

  

 FATO: f(x)=(1+r/x)^x eh uma funcao crescente (para x=1) que tende para
 e^r quando x-+Inf. Ou seja, (1+r/x)^x  e^r sempre que x=1.

  

 Agora sim!

  

 i) Nao ha solucao com ba=3. De fato, escrevendo b=a+r, vem:

  

 a^b-b^a = a^a.(a^r-(1+r/a)^a)  a^a.(a^r-e^r)  a^a.(a-e)  3^3.(0.2)  1*
 ***

  

 onde usei que r=1 para sumir com r (note que
 (a^r-e^r)=(a-e)(a^(r-1)+a^(r-2)+...+e^(r-1)) e este termo imenso eh =1), e
 que a=3 e que e=2.781828... no finzinho. 

  

 ii) Nao ha solucao com 3=ba. De fato, escrevendo a=b+r, vem:

  

 a^b-b^a = b^b.((1+r/b)^b-b^r)  b^b.(e^r-b^r)  0 pois be e r=1.

  

 Como obviamente tambem nao ha solucao com a=b, as unicas possiveis
 solucoes tem pelo menos um dos numeros menores do que 3. Agora eh soh
 analisar os casos a=1, a=2, b=1 e b=2:

  

 -- Se a=1 entao 1-b=1, nao presta.

 -- Se a=2, entao 2^b-b^2=1, isto eh, 2^b=b^2+1. Entao b eh impar, digamos,
 b=2k+1, entao 2^b=4k^2+4k+2 nao eh divisivel por 4, entao b=1. De fato
 (a,b)=(2,1) serve.

 -- Se b=1, entao a-1=1, isto eh, a=2, o que jah vimos acima.

 -- Se b=2, entao a^2-2^a=1, isto eh, 2^a=a^2-1=(a+1)(a-1). Entao ambos a-1
 e a+1 tem que ser potencias de 2 (cuja diferenca eh 2!), ou seja, a-1=2 e
 a+1=4. Assim, a=3.

  

 Cade meus 50 reais? ;)

  

 Abraco,

   Ralph

  

 2012/8/28 João Maldonado joao_maldona...@hotmail.com

 Meu amigo me passou um desafio anteontem, falou que se eu resolvesse até
 ontem a meia-noite, ele me dava 50 reais.
 Acontece que por mais insistente que eu tenha sido não saiu muita coisa :)

 A aposta já acabou e ele também não sabe a resolução, e eu quero muito
 saber como se resolve isso!
 Se alguém puder me dar uma ajuda eu agradeço

 Prove que a^b - b^a = 1  admite única e exclusivamente a solução (3, 2),
 para a e b naturais maiores de 0.


 []'s
 João
  

 ** **

 ** **



[obm-l] RES: [obm-l] Solução única

2012-08-28 Por tôpico bouskela
Olá!

 

Trata-se da Conjectura de Catalan, quase tão famosa quanto o Último Teorema de 
Fermat. A Conjectura de Catalan, lançada em 1844 pelo matemático belga Eugène 
Charles Catalan (1814-1894), propõe que 8 (=23) e 9 (=32) sejam as duas únicas 
potências inteiras e consecutivas (excetuando, é claro, 0 e 1).

 

A Conjectura de Catalan resistiu incólume por mais de 150 anos, até ser 
provada, em 2002, pelo matemático alemão (nascido na Romênia) Preda Mihăilescu.

 

Caso tenha interesse, posso lhe enviar a respectiva demonstração.

 

Albert Bouskela

 mailto:bousk...@gmail.com bousk...@gmail.com

 

De: owner-ob...@mat.puc-rio.br [mailto:owner-ob...@mat.puc-rio.br] Em nome de 
João Maldonado
Enviada em: terça-feira, 28 de agosto de 2012 13:26
Para: obm-l@mat.puc-rio.br
Assunto: [obm-l] Solução única

 

Meu amigo me passou um desafio anteontem, falou que se eu resolvesse até ontem 
a meia-noite, ele me dava 50 reais.
Acontece que por mais insistente que eu tenha sido não saiu muita coisa :)

A aposta já acabou e ele também não sabe a resolução, e eu quero muito saber 
como se resolve isso!
Se alguém puder me dar uma ajuda eu agradeço
 
Prove que a^b - b^a = 1  admite única e exclusivamente a solução (3, 2), para a 
e b naturais maiores de 0.
 
 
[]'s
João
 



[obm-l] RES: [obm-l] Solução única

2012-08-28 Por tôpico bouskela
Olá!

 

Trata-se da Conjectura de Catalan, quase tão famosa quanto o Último Teorema de 
Fermat. A Conjectura de Catalan, lançada em 1844 pelo matemático belga Eugène 
Charles Catalan (1814-1894), propõe que 8 (=2^3) e 9 (=3^2) sejam as duas 
únicas potências inteiras e consecutivas (excetuando, é claro, 0 e 1).

 

A Conjectura de Catalan resistiu incólume por mais de 150 anos, até ser 
provada, em 2002, pelo matemático alemão (nascido na Romênia) Preda Mihăilescu.

 

Caso tenha interesse, posso lhe enviar a respectiva demonstração.

 

Albert Bouskela

 mailto:bousk...@gmail.com bousk...@gmail.com

 

De: owner-ob...@mat.puc-rio.br [mailto:owner-ob...@mat.puc-rio.br] Em nome de 
João Maldonado
Enviada em: terça-feira, 28 de agosto de 2012 13:26
Para: obm-l@mat.puc-rio.br
Assunto: [obm-l] Solução única

 

Meu amigo me passou um desafio anteontem, falou que se eu resolvesse até ontem 
a meia-noite, ele me dava 50 reais.
Acontece que por mais insistente que eu tenha sido não saiu muita coisa :)

A aposta já acabou e ele também não sabe a resolução, e eu quero muito saber 
como se resolve isso!
Se alguém puder me dar uma ajuda eu agradeço
 
Prove que a^b - b^a = 1  admite única e exclusivamente a solução (3, 2), para a 
e b naturais maiores de 0.
 
 
[]'s
João
 



[obm-l] RES: [obm-l] Re: [obm-l] RES: [obm-l] Solução única

2012-08-28 Por tôpico bouskela
Olá!

 

É verdade! Eu li rapidamente e “vi” Catalan: – A eq. diofantina a^b - c^d = 1 
tem uma única solução: 3^2 - 2^3 = 1.

 

Bem, provar que a eq. diofantina a^b - b^a = 1 tem uma única solução (2, 3) é 
MUITO mais fácil:

 

Para começar, sugiro provar que a eq. diofantina a^b = b^a tem uma única 
solução (não trivial): 2^4 = 4^2 (acho que já fiz esta demonstração nesta 
lista).

 

Repare que: – Se abe então b^a  a^b (esta, certamente, eu já demonstrei 
nesta lista). Daí, b^a  a^b + 1 se ab3.

 

Repare, também, que: – Se eab então a^b  b^a (óbvio!).

 

Daí...

 

Mas o que interessa mesmo é que: EU GANHEI 50 PRATAS!!!

 

Albert Bouskela

 mailto:bousk...@gmail.com bousk...@gmail.com

 

De: owner-ob...@mat.puc-rio.br [mailto:owner-ob...@mat.puc-rio.br] Em nome de 
Tiago
Enviada em: terça-feira, 28 de agosto de 2012 15:56
Para: obm-l@mat.puc-rio.br
Assunto: [obm-l] Re: [obm-l] RES: [obm-l] Solução única

 

Na verdade, o que ele quer é um pouco mais fraco que a conjectura de catalan. 
Na conjectura de Catalan, você quer inteiros x,y,a,b tais que x^b - y^a=1. O 
que ele pede, é com a restrição adicional de x=a e y=b. Então talvez tenha uma 
resposta mais simples neste caso!

2012/8/28 bousk...@gmail.com

Olá!

 

Trata-se da Conjectura de Catalan, quase tão famosa quanto o Último Teorema de 
Fermat. A Conjectura de Catalan, lançada em 1844 pelo matemático belga Eugène 
Charles Catalan (1814-1894), propõe que 8 (=23) e 9 (=32) sejam as duas únicas 
potências inteiras e consecutivas (excetuando, é claro, 0 e 1).

 

A Conjectura de Catalan resistiu incólume por mais de 150 anos, até ser 
provada, em 2002, pelo matemático alemão (nascido na Romênia) Preda Mihăilescu.

 

Caso tenha interesse, posso lhe enviar a respectiva demonstração.

 

Albert Bouskela

 mailto:bousk...@gmail.com bousk...@gmail.com

 

De: owner-ob...@mat.puc-rio.br [mailto:owner-ob...@mat.puc-rio.br] Em nome de 
João Maldonado
Enviada em: terça-feira, 28 de agosto de 2012 13:26
Para: obm-l@mat.puc-rio.br
Assunto: [obm-l] Solução única

 

Meu amigo me passou um desafio anteontem, falou que se eu resolvesse até ontem 
a meia-noite, ele me dava 50 reais.
Acontece que por mais insistente que eu tenha sido não saiu muita coisa :)

A aposta já acabou e ele também não sabe a resolução, e eu quero muito saber 
como se resolve isso!
Se alguém puder me dar uma ajuda eu agradeço
 
Prove que a^b - b^a = 1  admite única e exclusivamente a solução (3, 2), para a 
e b naturais maiores de 0.
 
 
[]'s
João
 




-- 
Tiago J. Fonseca
http://legauss.blogspot.com



RES: [obm-l] Ajuda

2012-07-16 Por tôpico bouskela
Olá!

 

Uma boa alternativa é o C.a.R. (
http://zirkel.sourceforge.net/doc_en/index.html ), também gratuito. Eu,
assim, como o Ponce, prefiro o GeoGebra.

 

Albert Bouskela

 mailto:bousk...@gmail.com bousk...@gmail.com

 

De: owner-ob...@mat.puc-rio.br [mailto:owner-ob...@mat.puc-rio.br] Em nome
de Rogerio Ponce
Enviada em: segunda-feira, 16 de julho de 2012 20:34
Para: obm-l@mat.puc-rio.br
Assunto: Re: [obm-l] Ajuda

 

Ola' Marcelo,
ambos sao parecidos, mas o geogebra e' gratuito, e o sketchpad e' pago.
Alem disso, o sketchpad funciona somente em Windows ou Macs, enquanto o
geogebra roda em Windows, Mac, e Linux.

Como eu so' uso Linux, nao tenho muito mais a acrescentar...
[]'s
Rogerio Ponce



Em 14 de julho de 2012 07:28, Marcelo de Moura Costa mat.mo...@gmail.com
escreveu:

Alguém sabe dizer algo sobre o software geometer's sketchpad? É gratuito,
compatível com linux ou windows? Melhor que geogebra? Vantagens e
desvantagens em relação a outros de geometria dinâmica? 

 



RES: [obm-l] Prove que...

2012-07-05 Por tôpico bouskela
Olá!

 

Um resultado muito importante da Teoria dos Números é:

 

Todo quadrado de um par é par e múltiplo de 4 ( i.e., pode ser escrito assim: 
4(n^2) ou 4m ). Todo quadrado de um impar é impar e pode ser escrito da 
seguinte forma: 8n+1 (i.e., deixa resto 1, na divisão por 8). Obviamente, 
também deixa resto 1, na divisão por 4.

 

Obs.: ― Provar o resultado acima é muito fácil. Para o quadrado de um par, é 
óbvio. Para o quadrado de um impar, basta fazer o quadrado desse impar (2n+1) 
e, depois, analisar as 2 hipóteses possíveis: ― (i) “n” é par; e (ii) “n” é 
impar.

 

Agora, o problema:

 

Pitágoras: a^2 = b^2 + c^2 (neste problema, “a”, “b” e “c” são inteiros e 
positivos).

 

Logo, “b” e “c” não podem ser, ambos, ímpares, porque 8n+1 + 8m+1 = 8p+2, i.e., 
deixa resto 2, na divisão por 8, logo não é um quadrado.

 

1A. PARTE: um dos catetos é múltiplo de 4.

 

Hipótese 1: “b” é par (=2n) e “c” é impar. Nesta hipótese, a hipotenusa tem que 
ser impar (é óbvio!):

8m+1 = 4(n^2) + 8p+1. Logo: m = (n^2)/2 + 1. Logo: n^2 é par. Logo: “n” é par.

n=2q. Logo: b = 2n = 4q. Logo, “b” é múltiplo de 4.

 

Hipótese 2: “b” é par (=2n) e “c” também é par (=2p). Nesta hipótese, a 
hipotenusa tem que ser par (é óbvio!):

4(m^2) = 4(n^2) + 4(p^2). Logo: m^2 = n^2 + p^2.

Já foi visto que a soma de 2 quadrados ímpares não é um quadrado. Logo, “n” ou 
“p” é par. Já que há simetria, que seja, p.ex., “n”.

n=2q. Logo: b = 2n = 4q. Logo, “b” é múltiplo de 4.

 

2A. PARTE: “b” e “c” não podem ser, ambos, ímpares ― já foi visto!

 

3A. PARTE: b = m^2-n^2  e  c = 2mn  é, de fato, suficiente.

 

b^2 + c^2 = m^4 + 2(m^2)(n^2) + n^4 = (m^2 + n^2)^2 = a^2 (que é um quadrado).

 

Obs.: ― Esta é uma condição suficiente, não é uma condição necessária!

 

Albert Bouskela

 mailto:bousk...@gmail.com bousk...@gmail.com

 

De: owner-ob...@mat.puc-rio.br [mailto:owner-ob...@mat.puc-rio.br] Em nome de 
marcone augusto araújo borges
Enviada em: quarta-feira, 4 de julho de 2012 19:17
Para: obm-l@mat.puc-rio.br
Assunto: [obm-l] Prove que...

 

As medidas dos lados de um triângulo retângulo são representadas por números 
inteiros.Prove que a medida de um dos catetos
é representada por um múltiplo de 4.
 
Mostrar que as medidads dos catetos não podem ser ambas números ímpares e 
considerar essas medidas sendo
b = m^2 - n^2 e c = 2mn é suficiente?
 



RES: [obm-l] CIDADES

2012-07-03 Por tôpico bouskela
Olá!

 

VC = velocidade (carro); VM = velocidade (motocicleta); VB = velocidade 
(bicicleta)

 

D = distância (AB)

 

Após um determinado tempo “t1”, o carro encontra a motocicleta. Nesse ponto de 
encontro, o carro percorreu uma distância igual a D+a e a motocicleta D-a. Logo:

 

(D+a)/VC = (D-a)/VM

 

VM/VC = (D-a)/(D+a) ... Eq. 1

 

Após um determinado tempo “t2”, o carro encontra a bicicleta. Nesse ponto de 
encontro, o carro percorreu uma distância igual a D+b e a bicicleta D-b. Logo:

 

(D+b)/VC = (D-b)/VB

 

Daí: VB = VC(D-b)/(D+b)

 

Após um determinado tempo “t3”, a motocicleta encontra a bicicleta. Nesse ponto 
de encontro, a motocicleta percorreu uma distância igual a D+c e a bicicleta 
D-c. Logo:

 

(D+c)/VM = (D-c)/VB

 

Daí: VM = VB(D+c)/(D-c) = VC [(D-b)/(D+b)] [(D+c)/(D-c)]

 

VM/VC = [(D-b)/(D+b)] [(D+c)/(D-c)]

 

Voltando à Eq. 1:

 

(D-a)/(D+a) = [(D-b)/(D+b)] [(D+c)/(D-c)]

 

É só resolver a equação acima...

 

Albert Bouskela

 mailto:bousk...@gmail.com bousk...@gmail.com

 

De: owner-ob...@mat.puc-rio.br [mailto:owner-ob...@mat.puc-rio.br] Em nome de 
arkon
Enviada em: terça-feira, 3 de julho de 2012 14:45
Para: obm-l@mat.puc-rio.br
Assunto: [obm-l] CIDADES

 

Alguém pode resolver essa questão sinistra, por favor. 

Da cidade A partem, simultaneamente, para a cidade B, um carro, uma motocicleta 
e uma bicicleta. Alcançando B, o carro retorna à cidade A e encontra a 
motocicleta a “a” quilômetros de B e a bicicleta a “b” quilômetros de B. A 
motocicleta, ao chegar a B, retorna também, encontrando a bicicleta a “c” 
quilômetros de B. Determine a distância entre as cidades A e B.

(Suponha todos os movimentos uniformes).
[Gab.: \sqrt{\frac{abc}{a+c-b}}] 
= 
Instru絥s para entrar na lista, sair da lista e usar a lista em 
http://www.mat.puc-rio.br/~obmlistas/obm-l.html 
= 



[obm-l] RES: [obm-l] Re:[obm-l] A função e^x

2012-06-11 Por tôpico bouskela
Olá!

 

Sim, “a” e “b” são reais.

 

 

Albert Bouskela

 mailto:bousk...@gmail.com bousk...@gmail.com 

 

De: owner-ob...@mat.puc-rio.br [mailto:owner-ob...@mat.puc-rio.br] Em nome
de Eduardo Wilner
Enviada em: segunda-feira, 11 de junho de 2012 11:19
Para: obm-l@mat.puc-rio.br
Assunto: [obm-l] Re:[obm-l] A função e^x

 


No item 1) a e b são reais?

 



[obm-l] RES: [obm-l] A função e^x

2012-06-11 Por tôpico bouskela
Olá!

 

Quando escrevi o item “1”, não o fiz corretamente ― consertando:

 

Considere o número “a”, real, tal que: a1 e a=/e.

Obs.: =/ significa “diferente de”.

 

Mostre que a equação:

 

x^a=a^x

 

Possui uma única solução real e não trivial (x=b), sendo x=a a solução trivial. 
E mais:

 

Se 1ae, então be; e

Se ae, então 1be.

 

Albert Bouskela

 mailto:bousk...@gmail.com bousk...@gmail.com 

 

De: owner-ob...@mat.puc-rio.br [mailto:owner-ob...@mat.puc-rio.br] Em nome de 
douglas.olive...@grupoolimpo.com.br
Enviada em: segunda-feira, 11 de junho de 2012 15:05
Para: obm-l@mat.puc-rio.br
Assunto: Re: [obm-l] A função e^x

 

Na questão número 1 não vejo outra solução além de e fazendo o gráfico 
pode-se perceber, mas façamos

aplicando ln em ambos os lados teríamos, x=e.ln(x) , ou x/e=ln(x) , ou ainda 
f(x)=x/e-ln(x)=0 e analisando esta função fazendo a sua derivada primeira

daria f'(x)=1/e-1/x o que nos forneceria o ponto de mínimo x=e pois 
f''(e)=1/e^2 0 e f(e)=0, o que nos mostra que a única raíz desta equação será 
x=e e que o gráfico possui concavidade para cima e valor mínimo f(e)=0.

e logo responde a número 2 fazendo o mesmo procedimento de ln em ambos os lados 
 percebendo que o gráfico de eˆx-x^e é sempre positivo com exceçao da sua raiz 
x=e.

Agora a número 3, podemos fazer o seguinte sem perda de generalidade 
começaremos xy ,logo x/y1 , o que nos fornece, que x/y=1+t, x=y(1+t), e 
substituindo na primeira expressão teríamos

[y(1+t)]ˆy=yˆ[y(1+t)] e simplificando vem y^(1+t)=y(1+t), y^t=1+t, agora pausa 
aqui isso me lembra uma outra expressão o qual já usei muito que é e^x1+x para 
x0, mas isso é fácil mostrar numa expansão de série e^x=1+x+(x^2)/2! + 
...1+x. pronto agora vamos fazer uma comparação se y^te^t, ye ai teríamos 
y=1 ou y=2, mas testando y=1 teremos

uma contradição pois t seria 0, e testando y=2 fica 2^t=1+t. e na equação 
inicial fica 2^x=x^2 o que seria um quadrado perfeito aí fica fácil ver por 
indução que  2^x=2,4,8,... e 1+x=2,3,4,..., logo a solução será t=1 e 
x=2(1+1)=4 logo 2^4=4^2, e para y^te^t  teremos ye tipo 3,4,  e pelo 
gráfico fica fácil ver que não existem mais soluções.

Valeu cara espero ter ajudado ai . um abraço do Douglas Oliveira

 

On Sun, 10 Jun 2012 13:08:33 -0300,  mailto:bousk...@gmail.com 
bousk...@gmail.com wrote:

Olá!

 

Considere a função f(x)=e^x

 

1) A equação   e^a = a^e   (a1 e “a” diferente de “e”)

Mostre que essa equação tem uma segunda raiz “b” (diferente de “a”), tal que: 

Se ae, então b

Se ae.

 

2) Mostre que   e^x  x^e   para qualquer que seja “x” real e positivo (e 
diferente de “e”).

 

3) Mostre que a equação   m^n = n^m   tem uma única solução não trivial no 
domínio dos naturais: 2^4=4^2.

 

 

Albert Bouskela

 mailto:bousk...@gmail.com bousk...@gmail.com 

 

 

 



[obm-l] RES: [obm-l] Dados n naturais consecutivos, um é múltiplo de n

2012-06-09 Por tôpico Judah Ben Hur
Sejam k+1, k+2, ..., k+n os tais n naturais consecutivos.
Dividamos o primeiro deles, k+1, por n. Se o resto for zero terminamos, caso
contrário, seja r o resto da divisão. Então o número ( k+1 ) + (n - r )=
=nx(q + 1), está na lista acima e é divisível por n. O próximo múltiplo de n
está fora da lista.
Aplicação: Encontre um múltiplo de 17 que comece com 2012.
Procuremos na lista: 201201, 201202,...,201217.
Dividindo 201201 por 17 encontramos quociente 11835 e resto 6, então somamos
11 ao resto e somamos 11 ao número 201201. A divisão desse número por 17 dá
resto 0 e quociente 11836.
Espero ter ajudado.
Judah.

-Mensagem original-
De: owner-ob...@mat.puc-rio.br [mailto:owner-ob...@mat.puc-rio.br] Em nome
de Paulo Argolo
Enviada em: sábado, 9 de junho de 2012 19:48
Para: obm-l@mat.puc-rio.br
Assunto: [obm-l] Dados n naturais consecutivos, um é múltiplo de n

Caríssimos Colegas,

Como posso provar o teorema seguinte?

--- Dados n números naturais consecutivos, um deles (e somente um) é
múltiplo de n. ---


Abraços do Paulo.

=
Instrues para entrar na lista, sair da lista e usar a lista em
http://www.mat.puc-rio.br/~obmlistas/obm-l.html
=


=
Instruções para entrar na lista, sair da lista e usar a lista em
http://www.mat.puc-rio.br/~obmlistas/obm-l.html
=


[obm-l] RES: [obm-l] Divulgação: Olimpíada Brasileira de Matemática recebe inscrições até 30 de abril

2012-05-07 Por tôpico André Chaves
Querida Nelly,

Então as inscrições já acabaram. Uma pena.

Um abração,

Andrezinho.

 

 

De: owner-ob...@mat.puc-rio.br [mailto:owner-ob...@mat.puc-rio.br] Em nome
de Olimpiada Brasileira de Matematica
Enviada em: segunda-feira, 2 de abril de 2012 10:45
Para: Lista de discussao; Socios OBM; aob...@mat.puc-rio.br; Coordenadores
Assunto: [obm-l] Divulgação: Olimpíada Brasileira de Matemática recebe
inscrições até 30 de abril

 

Caros amigos, 

Estão abertas, até o dia 30 de abril, as inscrições para a 34ª Olimpíada
Brasileira de Matemática (OBM).


Cordialmente,


Nelly Carvajal

 

Olimpíada Brasileira de Matemática recebe inscrições até o dia 30 de abril

Colégios das redes pública e particular podem participar

 

 Estão abertas, até o dia 30 de abril, as inscrições para a 34ª
Olimpíada Brasileira de Matemática (OBM). A competição envolve a
participação de professores e alunos das redes pública e particular de todo
o país. As instituições interessadas devem fazer o cadastro mediante o
preenchimento da ficha de inscrição, disponível na página:
http://www.obm.org.br/ www.obm.org.br.  As inscrições são gratuitas.

A competição é uma iniciativa destinada aos alunos do 6º ao 9º ano do Ensino
Fundamental, alunos do Ensino Médio e estudantes universitários de
graduação. Para o coordenador geral da OBM, Carlos Gustavo T. de A. Moreira,
“A olimpíada desempenha um importante papel relacionado à melhoria do ensino
e à descoberta de talentos para a pesquisa em matemática e ciências afins,
além de estimular o pensamento criativo dos jovens participantes”, disse. 

Em 2011, mais de 190 mil alunos e seus professores participaram da olimpíada
que é aplicada em três fases. Este ano a prova da primeira fase será
realizada, 16 de junho, a segunda fase, 22 de setembro e a terceira e última
fase nos dias 27 e 28 de outubro. A divulgação dos resultados acontecerá
durante o mês de dezembro.

 

Premiação

Como parte da premiação serão entregues medalhas de ouro, prata e bronze,
além de certificados de menção honrosa. Os medalhistas ainda serão
convidados a participar da 16ª Semana Olímpica, evento a ser realizado em
janeiro de 2013. Além das medalhas e prêmios, os estudantes terão a
oportunidade de participar do processo de seleção para formar as equipes que
representam o Brasil nas diversas olimpíadas internacionais de Matemática. 

 

Sobre a OBM

A Olimpíada Brasileira de Matemática, competição realizada desde 1979, é um
projeto conjunto do Instituto Nacional de Matemática Pura e Aplicada (IMPA),
da Sociedade Brasileira de Matemática (SBM) e conta com o apoio do Conselho
Nacional de Desenvolvimento Científico e Tecnológico (CNPq) e do Instituto
Nacional de Ciência e Tecnologia de Matemática (INCTMat).

 

Instituto Nacional de Matemática Pura e Aplicada (IMPA)
Secretaria da Olimpíada Brasileira de Matemática (OBM) 
Estrada Dona Castorina, 110 Jd. Botânico, 
Rio de Janeiro - RJ, 22460-320, Brasil
Tel: 55-21-25295077 Fax:55-21-25295023
e-mail: o...@impa.br 
web site: www.obm.org.br   
  _  


Nenhum vírus encontrado nessa mensagem.
Verificado por AVG - www.avgbrasil.com.br
Versão: 2012.0.1913 / Banco de dados de vírus: 2425/4982 - Data de
Lançamento: 05/06/12



[obm-l] RES: [obm-l] Re: [obm-l] dúvida em uma afirmação de um vestibular da UEM

2012-05-01 Por tôpico bouskela
Bernardo,

Concordo in totum!

Essa mania de aferir conceitos através de enunciados capciosos só evidencia
a ignorância (latu sensu) dos examinadores. I.e., essa merda enche o saco!

Albert Bouskela
bousk...@gmail.com

-Mensagem original-
De: owner-ob...@mat.puc-rio.br [mailto:owner-ob...@mat.puc-rio.br] Em nome
de Bernardo Freitas Paulo da Costa
Enviada em: terça-feira, 1 de maio de 2012 08:23
Para: obm-l@mat.puc-rio.br
Assunto: [obm-l] Re: [obm-l] dúvida em uma afirmação de um vestibular da UEM

2012/5/1 Vanderlei * vanderma...@gmail.com:
 Pessoal, a afirmação a seguir é verdadeira ou falsa? Penso que seja 
 verdadeira, porém o gabarito do vestibular diz ser falsa! Preciso de
ajuda!

 Em um plano, existem duas figuras F1  e F2, cujas bases estão sobre 
 uma reta r  do plano, com a seguinte propriedade:
 toda reta paralela à reta  r que intersecta F1  e F2  determina 
 segmentos em  F1  e em  F2  de mesma medida .
 Então, a área de  F1 é igual à área de  F2.
Talvez seja uma sutileza do enunciado. O que é verdade é Se toda reta r
determina segmentos de comprimentos iguais em F1 e F2, então F1 e F2 têm a
mesma área

O que poderia acontecer no caso do enunciado (levando *muito* ao pé da
letra, e francamente o tipo de questão que me deixa desgostoso com o modo
como as pessoas encaram a matemática) seria um triângulo equilátero para F1
e dois triângulos equiláteros (em forma de
ampulheta) para F2, tocando-se por um vértice. Assim, toda reta que encontra
F1 *E* F2, determina segmentos iguais, mas algumas retas encontram só F2 (e
outras poderiam encontrar só F1... ninguém falou que as figuras eram
conexas...) e a diferença de áreas vem daí.

Esse tipo de questão, numa múltipla escolha, me parece pouco apropriado. A
mesma questão, com um enunciado totalmente claro (e não com tantas
ambigüidades possíveis) e que pedisse uma demonstração ou um contra-exemplo
(e não apenas Verdadeiro ou Falso), talvez fosse melhor. Mas continuo
achando que usar isso para determinar a qualidade dos candidatos é uma
deformação do real propósito da matemática. Não se está medido a compreensão
de um assunto, mas a pura atenção ao mínimo detalhe.

Faz sentido usar isso em sala de aula: ainda mais para explicar a
necessidade de dar um enunciado *sempre* completo na hora da verdade
(porque é a exigência deontológica da disciplina), mas também para dizer
olha, a gente quer que tal coisa seja verdade, mais ou menos assim, e a
partir daí construir o tal do enunciado completo, que não cai do céu.

Abraços,
--
Bernardo Freitas Paulo da Costa

=
Instruções para entrar na lista, sair da lista e usar a lista em
http://www.mat.puc-rio.br/~obmlistas/obm-l.html
=


=
Instruções para entrar na lista, sair da lista e usar a lista em
http://www.mat.puc-rio.br/~obmlistas/obm-l.html
=


  1   2   3   4   5   6   7   8   9   10   >